Vous êtes sur la page 1sur 120

PEDIATRICS

Section A
1. Neonatology.............................................................................................................3
2. Growth and Development ......................................................................................13
3. Nutrition in Health and Disease..............................................................................18
4. Fluid and Electrolytes.............................................................................................25
5. Disorders of Gastrointestinal System.....................................................................29
6. Immunization, Infectious Diseases and Worm Infestation ....................................36
7. Hematological Disorders........................................................................................45
8. Disorders of Cardiovascular System......................................................................51
9. Disorders of Respiratory System...........................................................................60
10. Disorders of Kidney and Urinary Tract...................................................................65
11. Disorders of Endocrine System..............................................................................73
12. Disorders of Neuromuscular System.....................................................................79
13. Metabolic Disorders...............................................................................................87
14. Genetic Disorders..................................................................................................90
15. Childhood Malignancies.........................................................................................96
16. Miscellaneous......................................................................................................101

Section B
Practice Questions.........................................................................................107
(Comprising of Questions from Recent Exams and NEET Pattern Questions)
Section A

1. NEONATOLOGY

A. Normal New Born


B. Primitive Reflexes
C. Low Birth Weight
D. Respiratory Distress
E. Congenital Diaphragmatic Hernia
F. Bronchopulmonary Dysplasia
G. Resuscitation
H. Neonatal Jaundice and Kernicterus
I. Neonate of Diabetic Mother
J. Miscellaneous
4 Jaypees Triple A

NEONATOLOGY (QUESTIONS)

A. NORMAL NEW BORN 8. Kangaroo mother care is necessary till baby attains a weight
of: (J & K PG 2011)
1. The appropriate approach to a neonate presenting with a. 1500 gms
vaginal bleeding on day 4 of life is: (AI 05) b. 2000 gms
a. Administration of vitamin K c. 2500 gms
b. Investigation bleeding for disorder d. 3000 gms Ref: OP Ghai 8/e p148
c. No specific therapy 9. Gestational age of a new born baby can be assessed by
d. Administration of 10 ml/kg of fresh frozen plasma over 4 which one of the following scoring system:
hours a. Sarnat and Sarnat (J & K PG 2010)
Ref: Ghai 8/e p172 b. Downeys
2. Neonatal MCV: (PGI Nov 09) c. Silvermans
a. 90-110 d. New Ballards Ref: OP Ghai 8/e p125
b. 110-125
c. 125-135
Ref: Ghai 8/e p322; 7/e, p296, 298 B. PRIMITIVE REFLEXES
3. A full term baby, exclusively breast fed, at the end of 1 week 10. A 3 month baby will have: (PGI Nov 09)
was passing golden yellow stools and was found to have a. Pincer grasp
adequate hydration with normal systemic examination. The b. Head control
weight of the baby was just same as it was at birth. The c. Sitting with support
pediatrician should now advise: (AIIMS May 02) d. 2 words with meaning
a. Give oral solution with breast feeding e. Rolling over Ref: Ghai 8/e, p49
b. Start top feeding
11. Moro reflex disappears at: (AI 07; PGI June 98)
c. Investigate for lactic acidosis
a. 5 months
d. Reassure the mother that nothing is abnormal
b. 3 months
Ref: Ghai 6/e, p4
c. 7 months
4. Normal finding in term neonate: (PGI Dec 02, Dec 98) d. 6 months Ref: Nelson 18/e, p2439; Ghai 8/e p142
a. Erythema toxicum
12. Persistence of Moros reflex is abnormal beyond the age of:
b. Epsteins pearl
a. 3rd month (AIIMS May 07)
c. Bilateral cryptorchidism
b. 4th month
d. Subconjunctival hemorrhages
c. 5th month
e. Erythema nodosum
d. 6th month
Ref: Ghai 8/e p138
Ref: Ghai 8/e p142; Nelson 18/e p2439
5. Most common cause of delayed puberty in males is: (AI 08)
13. All of the following reflexes are present at birth except:
a. Kallamann syndrome
a. Rooting reflex (AI 07; PGI June 98)
b. Klinefelter syndrome
b. Symmetrical tonic neck reflex
c. Constitutional
c. Asymmetrical neck reflex
d. Prader-willi syndrome
d. Crossed extensor reflex
Ref: Ghai 8/e p535; Oskis 4/e, p2083
PEDIATRICS

Ref: Nelson 18/e p2439; Ghai 8/e p142


6. The pincer grasp with the index finger and the thumb is
14. Moros reflex is abnormal after: (DNB 2007)
attained by an infant by the age of: (MP PG 2009)
a. 4 weeks
a. 3 months
b. 10 weeks
b. 6 months
c. 12 weeks
c. 9 months
d. 20 weeks Ref: OP Ghai peds 8/e p210
d. 12 months Ref: Ghai 8/e p49-55; 7/e p28
15. Persistent moros reflex at 12 weeks indicates:
7. Upper segment to lower segment ratio at birth is:
a. Normal child (Feb DP PGMEE 2009)
a. 1.3 (J & K PG 2011)
b. Brain damage
b. 1.5
c. Hungry child
c. 1.7
d. Irritable Ref: Ghai 8/e p143
d. 1 Ref: OP Ghai 8/e p11

Ans. 1. c. No specific therapy 2. a. 90-110 3. d. Reassure the... 4. a, b and d


5. c. Constitutional 6. c. 9 months 7. c. 1.7 8. c. 2500 gms
9. d. New Ballards 10. b. Head control 11. d. 6 months 12. d. 6th month
13. b. Symmetrical tonic... 14. d. 20 weeks 15. a. Normal child
Neonatology 5
16. Persistent moros reflex at 12 weeks indicates: Ref: Nelson 18/e p2423, 2603 and 17/e p2113; ROP,
a. Normal child (Feb DP PGMEE 2009) p709, 2598-2600; Ghai 8/e p666; 7/e p130
b. Brain damage 23. Loss of pulmonary surfactant in premature infant:
c. Hungry child a. Pulmonary edema (Nov 08)
d. Irritable b. Collapse of alveoli
Ref: Ghai 8/e p143 c. Elastic recoil of lungs
17. Which of the following is not present at birth? d. All of above
a. Parachute reflex (Kerala PG 08) Ref: Nelson 17/e p575, 18/e p732; Ghai 8/e p373
b. Moro reflex 24. Retrolental fibroplasias has association with:
c. Tonic neck reflex a. Prolonged labor (AIIMS May 02)
d. Rooting reflex b. Intrauterine infection
Ref: Ghai 8/e p133; Nelson, 17/e p1979, c. Meconium aspiration
Table 584-2 18/e p2439, Table 594-2 d. Low birth weight
Ref: Ghai 8/e p155; Meharban Singh 6/e p404
C. LOW BIRTH WEIGHT 25. True about LBW is: (DNB 2009)
a. Less than 1800 gms
18. All of the following are features of prematurity in a neonate, b. Less than 2000 gms
c. Less than 2500 gms
except: (AI 06)
d. Less than 3000 gms Ref: OP Ghai 8/e p310
a. No creases on sole
b. Abundant lanugo 26. Low birth weight (LBW) baby is the one whose birth weight
is: (MP PG 2010)
c. Thick ear cartilage
a. Less than 1800 gms
d. Empty scrotum
b. Less than 2000 gms
Ref: Ghai 8/e p124; 7/e p129; c. Less than 2500 gms
Nelson 18/e p703; 17/e p525 d. Less than 3000 gms
19. Small for date baby is: (PGI June 00) Ref: Ghai 8/e p124; Nelsons 18/e p702
a. < 10 percentile for the gestational age 27. A very low birth weight is a newborn whose weight, in
b. < 50 percentile for gestational age grams, is less than: (MP PG 2010)
c. < 2000 gm a. 1000
d. < 2500 gm b. 1500
Ref: Ghai 8/e p155; 7/e p129 c. 2000
d. 2500
20. A women delivered a baby of 2.2 kg weight her IMP is not Ref: Ghai 8/e p124; Nelsons 18/e p702
known. To know the maturity of baby, following are used:
a. Sole crease (PGI June 03)
b. Ear cartilage D. RESPIRATORY DISTRESS
c. Breast nodule
28. With reference to RDS, all of the following statements are
d. Anterior fontaneue
true except: (AI 02; PGI Dec 99)
e. Weight of the baby
a. Usually occurs in infants born before 34 weeks of
Ref: Ghai 8/e p155
gestation
21. Full term, small-for-date babies are at high-risk of: (AI 00)

PEDIATRICS
b. Is more common in babies born to diabetic mothers
a. Hypoglycemia c. Leads to cyanosis
b. Intraventricular hemorrhage d. Is treated by administering 100% oxygen
c. Bronchopulmonary dysplasia Ref: Ghai 8/e p169; 7/e p144-146, 6/e, p166-167;
d. Hyperthermia Nelson 18/e p731-740 and 17/e p575-578
Ref: Ghai 8/e p179; 7/e p129 29. Most common sign of acute hypoxia in neonates: (AI 09)
22. Retinopathy of prematurity is commonly predisposed by: a. Tachycardia
a. Less gestation age (AIIMS June 00) b. Bradycardia
b. Low birth weight c. Cardiac arrest tachypnea
c. O2 toxicity d. Ventricular arrhythmia
d. Carbohydrate excess Ref: Nelson 18/e p718, 719; Ghai 8/e p166

Ans. 16. a. Normal child 17. a. Parachute reflex 18. c. Thick ear cartilage 19. a. < 10 percentile for...
20. a, b and c 21. a. Hypoglycemia 22. a. Less gestation age 23. All of above
24. d. Low birth weight 25. c. Less than 2500 gms 26. c. Less than 2500 gms 27. b. 1500
28. d. Is treated by... 29. b. Bradycardia
6 Jaypees Triple A
30. Nile blue sulfate test is done to detect: (AIIMS Nov 06) c. Term gestation
a. Maturity of kidney d. Onset after 6 hours
b. Maturity of liver Ref: Ghai 8/e p169; Nelsons, 18/e p731-5
c. Maturity of lungs 37. True about RDS are all except:
d. Maturity of skin
a. Develops 6 hours after birth
Ref: Meharban Singh 6/e, p42-43; Dutta p420
b. Air bronchogram seen
31. Neonatal apnea is seen in all except: (Nov 08) c. Cyanosis seen
a. Prematurity d. Prenatal steroids given
b. Hyperglycemia Ref: Ghai 8/e p169; Nelsons, 18/e p731-5
c. Hypoglycemia
d. Hypercalcemia 38. A child 30 weeks preterm presents with moderately severe
e. Hyperthermia respiratory distress with grunting after 2 hours. Respiratory
Ref: OP Ghai 8/e p164; 7/e p146 management.
a. Nasal CPAP
32. Which of the following are signs of neonatal respiratory
b. Surfactant with mechanical ventilation
distress syndrome: (PGI June 01)
a. Intercostal retraction c. Intubation
b. RR > 60/min d. Warm oxygen
c. Absence of cyanosis Ref: Nelsons. 18/e p731-5,
d. PH < 7.2 OP Ghai 8/e p169; 7/e p144
e. A linear streak on CXR 39. In a postoperative ward newborn developed respiratory
Ref: Ghai 8/e p137; 7/e p144-146; Nelson 18/e p723; 17/e p575-577 depression in ward. It can be caused by?
33. Neonate at 2 hours age develops moderate respiratory a. Opioid
distress, in terms of increased respiratory rate (70/m), chest b. Propofol
indrawing and grunting. Which is the best management: c. Diazepam
a. Surfactant therapy and mechanical ventilation d. Ketamin
b. Intubation and mechanical ventilation (AIIMS Nov 09) Ref: Care of newborn Meharban singh,6/e p79;
c. Facemask with CPAP 8/e p168
d. 100% O2 and and head box
40. A neonate with scaphoid abdomen with respiratory distress
Ref: Ghai 8/e p169
has: (Karnataka PG 2010)
34. A 3 kg term baby delivered by cesarian section develops a. Congenital pyloric stenosis
respiratory distress soon after birth. The liquor was b. Volvulus
meconium stained. Breathing rate is 90/ minute. Correct
c. Congenitae Diaphragmatic hernia
statements: (PGI Dec 04)
d. Choanal atresia
a. Transient tachypnea of newborn
Ref: Ghai 8/e p138, 178
b. Meconium aspiration syndrome
c. Reticulonodular shadows in X-ray chest
d. Surfactant production E. CONGENITAL DIAPHRAGMATIC HERNIA
e. Oral feeding started early
Ref: Ghai 8/e p170; Nelson 18/e p742-743 and 17/e p583-584 41. A child presented with severe respiratory distress two days
35. Respiratory distress in newborn is defined when: after birth. On examination he was observed to have a
PEDIATRICS

a. Respiration rate >60/min (PGI June 04) scaphoid abdomen and decreased breath sounds on the left
b. Intercostal recession side. He was managed by prompt endotracheal intubation.
c. Aspiration> 20 ml After ET tube placement the maximal cardiac impulse
d. Hypoxemia shifted further to the right side. What should be the next
e. Reticulonodular shadow in CXR step in management? (AI 09)
Ref: Ghai 8/e p169; 7/e p144-147; a. Confirm the position of endotracheal tube by chest X-ray
Nelson 18/e p723 and 17/e p575-577 b. Remove tube and reattempt intubation
36. Newborn baby of term gestation, developed respiratory c. Nasogastric tube insertion and decompress the bowel
depression after 6 hours. What will suggest HMD? d. Chest X-ray to confirm diagnosis
a. Receipt of antenatal steroids Ref: Ghai 8/e p138, 178; 7/e p153;
b. Air bronchogram in CXR Nelson 18/e p731, 740

Ans. 30. c. Maturity of lungs 31. b, d and e 32. a, b and d 33. a. Surfactant therapy...
34. a and b 35. a and b 36. b. Air bronchogram... 37. a. Develops 6 hours...
38. a. Nasal CPAP 39. a. Opioid 40. c. Congenitae... 41. c. Nasogastric tube...
Neonatology 7
42. A child presented with respiratory distress was brought a. Insert a central venous pressure line
to emergency with bag and mask ventilation. Now child b. Bag and mask ventilation
is intubated. Chest X-ray shows right-sided deviation of c. Insert a nasogastric tube
mediastinum with scaphoid abdomen. His pulse rate is d. Ventilate with high frequency ventilator
increased. What is the next step? (AIIMS Nov 07) Ref: Nelson 18/e p746-749; 17/e p1353-1355;
a. Endotracheal intubation Schwartz 7/e p1720; Ghai 8/e p179
b. Put a nasogastric tube
c. Surgery
d. End tidal CO2 to confirm intubation
F. BRONCHOPULMONARY DYSPLASIA
Ref: Ghai 8/e p178; 7/e p153
43. A neonate having congenital diaphragmatic hernia 48. A male born at term after an uncomplicated pregnancy,
developed respiratory distress. Breath sounds were
labor and delivery develops severe respiratory distress
decreased on the left side. After bag and mask ventilation,
within a few hours of birth. Results of routine culture were
ET tube was put and the maximal cardiac impulse shifted to
negative. The chest roengogram reveals a normal heart
the right side. What should be the next step in management:
a. Confirm the position of endotracheal tube by X-ray chest shadow and fine reticulonodular infiltrates radiating from
b. Remove tube and reattempt intubation (AI 08) the hilum. ECHO findings reveal no abnormality. Family
c. Nasogastric tube insertion history reveals similar clinical course and death of a male
d. Chest X-ray female sibling at 1 month and 2 months of age respectively.
Ref: Nelson 18/e p748; Ghai 8/e p178 The most likely diagnosis is: (AI 08)
44. A Newborn weighing 1000 g is born at gestational age of a. Neonatal alveolar proteinosisy
30 weeks with respiratory distress after 2-3 hours of birth. b. Total anomalous pulmonary venous circulation (TAPVC)
What are the diagnostic possibilities: (PGI June 01) c. Meconium aspiration syndrome
a. Diaphragmatic hernia d. Diffuse herpes simplex infection
b. Cong. bronchopulmonary cysts Ref: Nelson 18/e p1820-1821; Ghai 8/e p176
c. Bronchopulmonary dysplasia 49. A newborn female child, weight 3.5 kg, delivered by
d. HMD uncomplicated delivery, developed respiratory distress
e. Pulmonary haemorrhage
immediately after birth. On chest X-ray ground glass
Ref: Ghai 8/e p179; 7/e p153;
appearance was seen. Baby put on mechanical ventilation
Nelson 18/e p731-740 and 17/e p575578
and was give surfactant but condition of baby deteriorates
45. Newborn with APGAR score of 2 at 1 min. and 6 at 5 min.
and increasing hypoxemia was present. A full term female
has respiratory distress and mediastinal shift diagnosis is:
sibling died within a week with the same complaints. ECHO
a. Congenital adenomatoid lung disease (PGI Dec 00)
is normal. Usual cultures are negative. Your diagnosis is:
b. Pneumothorax
c. Diaphragmatic hernia a. Total anomalous pulmonary vein connection
d. Transient tachypnea of newborn b. Meconium aspiration syndrome (AIIMS Nov 08)
e. HMD c. Neonatal pulmonary alveolar proteinosis
Ref: Nelson 18/e p731-740; 17/e, 575578; Ghai 8/e p176 d. Disseminated HSV infection
46. A newborn baby had normal APGAR score at birth and Ref: Nelson 18/e p1821-1822; Ghai 8/e p168; 176
developed excessive frothing and choking on attempted 50. 3.5 kg term male baby, born of uncomplicated pregnancy,
feeds. The investigation of choice is: (AIIMS May 03) developed respiratory distress at birth, did not responded to

PEDIATRICS
a. Esophagoscopy surfactant, ECHO finding revealed nothing abnormal, X-ray
b. Bronchoscopy showed ground glass appearance and culture negative.
c. MRI chest APGARS 4 and 5 at 1 and 5 minutes. History of one month
d. X-ray chest and abdomen with the red rubber catheter female sibling died before. What is the diagnosis?
passed per orally into esophagus
a. TAPVC (AIIMS June 08)
Ref: Ghai 8/e p178-179; 7/e p151;
b. Meconium aspiration
Nelson 18/e p1541 and 17/e p1219-1220
c. Neonatal pulmonary alveolar proteinosis
47. A new born baby has been referred to the casualty as a
d. Diffuse herpes simplex infection
case of congenital diaphragmatic hernia. The first clinical
Ref: Nelson 18/e p2185, Harrison 16/e p1696; Gahi 8/e p172
intervention is to: (AIIMS May 03)

Ans. 42. b. Put a nasogastric... 43. c. Nasogastric tube... 44. a, b and d 45. a, b and c
46. d. X-ray chest... 47. c. Insert a nasogastric... 48. a. Neonatal alveolar... 49. c. Neonatal...
50. c. Neonatal pulmonary...
8 Jaypees Triple A
58. The following modes of ventilation may be used for
G. RESUSCITATION weaning of patients from mechanical ventilation except:
a. Pressure support ventilation (PSV) (UP PG 2010)
51. A 5-year-old child is rushed to casualty reportedly
b. Control mechanical ventilation (CMV)
electrocuted while playing in a park. The child is apneic
c. Assist control ventilation (ACV)
and is ventilated with bag and mask. There are bums on
d. Synchronized intermittent mandatory ventilation (SIMV)
each hand. What will be the next step in the management:
Ref: Ghai 7/e p701-03; A yadav 4/e p205
(AIIMS Nov. 05, 04)
a. Check pulses 59. The first medication that may be required in the resuscitation
b. Start chest compressions of baby with prolonged birth asphyxia is: (J & K PG 2010)
c. Intubate a. Sodium bicarbonate
d. Check oxygen saturation b. Adrenalin in 1:10000 dilution
Ref: Nelson 18/e p395; 17/e p281, 286288; Ghai 8/e p125 c. Calcium gluconate
d. 25% dextrose Ref: OP Ghai 8/e p166, 126
52. A 6 months old child having severe dehydration comes to
the casualty with weak pulse and unrecordable BP Repeated
attempt in gaining IV access has failed. The next best step H. NEONATAL JAUNDICE AND
is: (AIIMS May 01)
a. Try again KERNICTERUS
b. Jugular vein catheterization
60. Drugs that can be used in kernicterus: (PGI June 09)
c. Intraosseous IV fluids
a. Barbiturates
d. Venesection Ref: Bailey and Love 24/e p290; Ghai 8/e p714
b. Benzodiazepines
53. A child presented in the casualty with fever, unconscious c. Phenytoin
and papilloedema. What next to be done: (PGI Nov 08) d. Chlorpromazine
a. Intubation e. Carbamazepine Ref: Ghai 8/e p172; 7/e p150
b. Oxygenation
61. A child has bilirubin of 4 mg. Conjugated bilirubin and
c. CT scan
alkaline phosphatase are normal, bile salts and bile in urine
d. Lumber puncture
are absent. However urobilinogen in urine is raised. What
Ref: Ghai 8/e p710; 7/e p535-536
is the likely diagnosis: (AIIMS Nov 01)
54. Best artery to palpate for pulse in infants is: (PGI Dec 00) a. Obstructive jaundice
a. Femoral artery b. Rotors syndrome
b. Radial artery c. Biliary cholestasis
c. Carotid artery d. Hemolytic jaundice
d. Brachial artery Ref: Chatterjee Shinde 4/e p593; Chandrasoma 3/e p635;
Ref: Morgans Anesthesia 3/e p936; Ghai p710; 712 Ghai 8/e p172-175
55. Which among the following is a helpful characteristic in 62. Conjugated hyperbilirubinemia in infancy seen in:
neonatal resuscitation: (Kerala PG 10) a. Choledochal cyst (PGI Dec 04)
a. APGAR Score b. Extra hepatic biliary atresia
b. Colour c. Crigler - Najjar disease
c. Cry d. Gilbert disease
d. Heart rate Ref: Ghai 8/e p126; Nelson 18/e p679 Ref: Already explained above; Ghai 8/e p173
56. A neonate on bag and mask ventilation in cardio pulmonary 63. Conjugated hyperbilirubinemia in infancy is seen in:
resuscitation. The heart rate is 60-100 per munute. Next a. Gilbert syndrome (PGI Dec 06)
treatment is: (UP PG 2010)
PEDIATRICS

b. Crigler-Najjar syndrome
a. Continue ventilation and start adranaline c. Dubin Johnson syndrome
b. Continue ventilation and start chest compressions d. Rotor syndrome
c. Continue chest compressions and ventilation e. Neonatal hepatitis
d. Stop chest compressions and continue ventilation Ref: Already explained above; Ghai 8/e p173
Ref: OP Ghai 8/e p714; 7/e p99
64. A term neonate with unconjugated hyperbilirubinemia of
57. A ratio of lung inflation to chest compression in 18 mg/dl on 20 days. All are common causes except:
cardiopulmonary resuscitation is: (UP PG 2010) a. Breast milk jaundice (AIIMS May 07)
a. 1:5 b. Congenital cholangiopathy
b. 2:5 c. G 6PD deficiency
c. 1:15 d. Hypothyroidism
d. 2:15 Ref: Ghai 8/e p714; 7/e p691-92; Nelson 18/e p389 Ref: Ghai 8/e p173-174; 7/e p147-151

Ans. 51. a. Check pulses 52. c. Intraosseous IV fluids 53. All of these 54. b. Radial artery
55. d. Heart rate 56. d. Stop chest... 57. d. 2:15 58. b. Control mechanical...
59. b. Adrenalin in 1:1000... 60. a. Barbiturates 61. d. Hemolytic jaundice 62. a and b
63. c, d and e 64. b. Congenital...
Neonatology 9
65. Late onset hemorrhagic disease of newborn is characterized 72. A term baby developed jaundice on 3rd day up to the thigh
by all of the following features except: (AI 06) with normal stool and urine. Mothers blood group is O
a. Usually occurs in cow-milk-fed babies -ve and that of babys A +ve. The cause of jaundice is:
b. Onset occurs at 4-12 weeks of age a. Rh incompatibility (PGI Dec 04; Dec 02)
c. Intracranial hemorrhage can occur b. Physiological jaundice
d. Intramuscular vitamin K prophylaxis at birth has a c. Extrahepatic biliary atresia
protective role d. Sepsis
Ref: Nelson 18/e p773-774; 17/e p606607; Ghai 8/e p148 e. Glucose-6 phosphate dehydrogenase deficiency
66. Which of the following is the principal mode of heat Ref: Ghai 8/e p173; 7/e, 1147;
exchange in an infant incubator? (AI 06) Nelson 18/e p760-761 and 17/e p594-595
a. Radiation 73. True about physiological jaundice in neonate: (PGI Dec 00)
b. Evaporation a. Occurs in first 6 hours of delivery
c. Convection b. Unconjugated hyperbilirubinemia
d. Conduction c. Neurological equel are common
Ref: Ghai 8/e p173; 7/e p150 d. Best treated by phototherapy
67. In unconjugated hyperbilirubinemia, the risk of kernicterus e. Starts on 2nd day of life Ref: Ghai 8/e p172
increases with the use of: (AI 05) 74. Bronze baby syndrome is due to: (PGI Dec 98)
a. Ceftriaxone a. Phototherapy
b. Phenobarbitone b. Wilson disease
c. Ampicillin c. Chloramphenicol toxicity
d. Sulfonamide d. Hemochromatosis
Ref: Harrsion 16/e p1818; KDT 5/e, p644; Ghai 8/e p173 Ref: Nelson 18/e p762, 763 and 17/e p598;
68. What should be measured in a newborn who presents with Ghai 8/ep172; 7/e p150
hyperbilirubinemia: (AI 00) 75. The late features of kernicterus include all except:
a. Total and direct bilirubin a. Hypotonia (DP PGMEE 2009)
b. Total bilirubin only b. Sensorineural hearing loss
c. Direct bilirubin only c. Choreoathetosis
d. Conjugated bilirubin only d. Upward gaze palsy
Ref: Meharban Singh 6/e p241; Ghai 8/e p172 Ref: Ghai 8/e p174
69. Unconjugated hyperbilirubinemia in neonate is seen in all 76. In a neonate, jaundice appears for the first time in the 2nd
of the following except: (AI 98) week. The following is not a cause: (DP PGMEE 2010)
a. Physiological jaundice a. Galactosemia
b. Dubin Johnson syndrome b. Rh incompatibility
c. Hypothyroidism c. Hypothyroidism
d. Hemolytic anemia d. Breast milk jaundice
Ref: Ghai 8/e p172; Nelson 18/e p1677; 17/e p592-596, 1321 Ref: Ghai 8/e p172; Nelson 18/e p758
70. Which mechanism in phototherapy is chiefly responsible
for reduction in serum bilirubin? (AIIMS May 05)
a. Photo-oxidation
I. NEONATE OF DIABETIC MOTHER
b. Photo-isomerization
77. Which of the following malformation in a newborn is
c. Structural isomerization
specific for maternal insulin dependent diabetes mellitus?
d. Conjugation

PEDIATRICS
a. Transposition of great arteries (AI 06)
Ref: Ghai 8/e p172; 7/e p150, 6/e p172-173;
b. Caudal regression
Nelson 18/e p762-764; 17/e p597-598
c. Holoprosencephaly
71. A full term, 80 hours old new born baby develops jaundice, d. Meningomyelocele
what should be the minimum level of serum bilirubin to Ref: Read below Ghai 8/e p179; 7/e p156
start phototherapy: (AIIMS June 99)
78. Infants of diabetic mother have the folio wings:
a. 20 mg%
a. Macrosomia (PGI June 03)
b. 12.5 mg%
b. Neural tube defect
c. 18 mg%
c. Hyperglycemia
d. 15 mg%
d. Hypocalcemia
Ref: Ghai 8/e p173; Nelson 18/e p762-763;
Ref: Nelson 18/e p783, 784; Dutta-Obs 5/e, p 303, 304;
17/e p597598; Meharban Singh 6/e p252
Ghai 8/e p179; 7/e p156

Ans. 65. a. Usually occurs... 66. c. Convection 67. a and d. 68. a. Total and direct
69. b. Dubin Johnson... 70. c. Structural... 71. a. 20 mg% 72. b. Physiological...
73. b. Unconjugated... 74. a. Phototherapy 75. a. Hypotonia 76. b. Rh incompatibility
77. b. Caudal regression 78. a, b and d
10 Jaypees Triple A
79. Long-term complication of infants born to IDDM mother 85. All can be seen in infant of diabetic mother except:
all except: (AI 95, AIIMS 98) a. Hyperbilirubinaemia (Kerala PG 08)
a. DM b. Polycythaemia
b. Ketotic hypoglycemia c. Hyperglycaemia
c. Obesity d. Hypocalcaemia
d. Blindness Ref: Ghai 8/e p182; Dutta 6/e p287
Ref: Ghai 8/e p179; 7/e p156
80. All of the following are the complications in the new born J. MISCELLANEOUS
of a diabetic mother except: (AIMS May 06)
a. Hyperbilirubinemia 86. Hypothermia in neonate is characterized by: (PGI Dec 02)
b. Hyperglycemia a. Hyperactivity
c. Hypocalcemia b. Hypoglycemia
d. Hypomagnesemia c. Apnea
Ref: Ghai 8/e p179; 7/e p156, 6/e p180; d. ed urinary output
Nelson Pediatrics 17/e p613-614 Ref: Ghai 8/e p143; 7/e p118, 115
81. Macrosomia is seen in: (PGI Nov 09) 87. The different manifestations of hypothermia are:
a. GDM a. Apnea [PGI June 06]
b. Maternal obesity b. Hypoglycemia
c. Maternal hypothyroidism c. Hyperglycemia
d. Neonatal Hyperglycemia d. Tachycardia
e. Neonatal Hypoglycemia e. Hypoxia
Ref: Dutta Obs 6/e p287; Ghai 8/e p178, 179 Ref: Ghai 8/e p143; 7/e, p115-118
82. All of the following therapies may be required in a 1 hour 88. Neonate with recurrent infection and abscess is diagnosed
old infant with severe birth asphyxia except: (AI 05) as kostmann syndrome (severe congenital neutropenia).
a. Glucose What is the treatment: (AIIMS Nov 09)
b. Dexamethasone a. Anti-thymocyte globulin + cyclosporine
c. Calcium gluconate b. Anti-thymocyte globulin + cyclosporine + gm-csf
d. Normal saline c. G-csf
Ref: Meharban Singh 6/e p106; Ghai 8/e p166 d Gm-csf
83. The most common congenital anomaly in baby born to Ref: Ghai 8/e p357; Current Diagnosis and
IDDM mother is: (AIIMS Nov 06) treatment-Pediatrics 19/e p827
a. NTD 89. True about neonatal sepsis: (PGI June 03)
b. Cardiovascular anomalies a. Meningitis commonly occur lately
c. GIT anomalies b. Jaundice predisposes
d. Pulmonary anomalies c. Fever
Ref: Ghai 8/e p396; Nelson 18/e p618, d. Jaundice is a common feature
619 and 17/e p614 Ref: Ghai 8/e p163; 7/e p136-137
84. A 3.5 kg baby born to diabetic mother develops seizures at 90. Transient tachypnea of new born (TTN) is commonly seen
PEDIATRICS

16 hours. The most likely cause is: (DNB 2010) in which of the following situations: (AIIMS May 02)
a. Hypoglycemia a. Term delivery requiring forceps
b. Hypoxia/Respiratory distress syndrome b. Term requiring ventouse
c. Hypomagnesumia c. Elective cesarean section
d. Hypocalcemia d. Normal vaginal delivery
Ref: Ghai 8/e p210; Nelson Essentials of Pediatrics Ref: Nelson 18/e p741; 17/e p583;
4/e p762-766, 783, 237, 238 Ghai 8/e p168; 7/e p146

Ans. 79. b. Ketotic ... 80. b. Hyperglycemia 81. a. GDM 82. b. Dexamethasone
83. b. Cardiovascular... 84. a. Hypoglycemia 85. c. Hyperglycaemia 86. b, c and d
87. All of the above 88. c. G-csf 89. a, c and d 90. c. Elective cesarean...
Neonatology 11
91. Characteristics radiological feature of transient tachypnea 97. All of the following groups of newborns are at an increased
of newborn is: (AIIMS May 05) risk of hypoglycemia except: (AIIMS Nov 02)
a. Reticulogranular appearance a. Birth asphyxia
b. Low volume lungs b. Respiratory distress syndrome
c. Prominent horizontal fissure c. Maternal diabetes
d. Air bronchogram d. Post-term infant
Ref: Ghai 8/e p168 Ref: Ghai 8/e p179; Nelson 18/e p785and 17/e p506-508
92. A nonventilated preterm baby in incubator is under 98. A male child of 3.8 kg born to a diabetic mother, developded
observation. Which is the best way to monitor the babys seizures 16 hours after birth. What is the probable cause:
breathing and detect apnea? (AI 07) a. Hypoglycemia (AIIMS Nov 09)
a. Infrared throraric movement study b. Hypocalcemia
b. Capnography c. Birth asphyxia
c. Nasal digital temperature monitoring d. Intracranial haemorrhage
d. Impedence technique Ref: Ghai 8/e p179; 7/e p156
Ref: Care of the newborn by Meharban 99. A newborn child RR 86 per minute. No nasal flaring, no
Singh 6/e p30, 280; Ghai 8/e p180 lower chest or xiphoid retraction, no grunt. Abdomen
93. Treatment for breath holding spells in a child is: lagged behind chest in movement. Silvermans score is:
a. Give extra care and love to the child (PGI Dec 01) a. 1
b. Inflicting painful stimulus at the beginning of the attack b. 2
c. Do not give attention to the child c. 3
d. Fulfill all the wishes of the child to prevent the attack d. 4
e. Low dose barbiturates Ref: Care of Newborn Meharban singh,6/e p262; Ghai 8/e p174
Ref: Nelson 18/e p131 and 17/e p88 100. In a preterm with PDA least likely is:
94. True about B/L CDH: (PGI June 09) a. Bounding pulses
a. Exaggerated lordosis b. NEC
b. B/L genu valgum c. CO2 washout
c. Waddling gait d. Pulmonary hemorrhage
d. Stentons line brocken Ref: Ghai 8/e p402; Nelson. Textbook of pediatrics, 18/e p737,
e. Short stature Neo Reviews 2010; 11: 495-502
Ref: Maheshwari 4/e p212, 213; 3/e p201, 202; Ghai 8/e p284 101. Lines of blaschko are:
95. A 4 kg baby born to a diabetic mother found lethargic which a. Lymphatics
of the folio whig is to be done: (PGI Dec 03) b. Blood vessel
a. Reasses the baby again after 2 hours c. Nerves
b. Give 10% dextrose IV d. Lines of development
c. Start oral feeding Ref: Harper, John. Textbood of Pediatric Dermatology, p691; Internet
d. Give injection insulin Ref: Ghai 8/e p179; 7/e p157 102. A neonate delivered at 38 weeks of gestation, birth weight
96. Administration of glucose solution is prescribed for all of of 2.2 kg develops intolerance to feeds on 2nd day. Physical
the following situations except: (AIIMS May 06) examination reveals no abnormalities. Sepsis screen in

PEDIATRICS
a. Neonates negative. What is the next step in management?
b. Child of a diabetic mother a. Wait and watch
c. History of unconsciousness b. Do a 2nd sepsis screen
d. History of hypoglycemia c. Give prophylactic antibiotics
e. Uncorrected cases may develop plagiocephaly d. X-ray abdomen
Ref: Ghai 8/e p179 Ref: Ghai 8/e p163

Ans. 91. c. Prominent hori... 92. d. Impedence technique 93. b and c 94. All of these
95. b and c 96. c. History of... 97. d. Post-term infant 98. b. Hypocalcemia
99. a. 1 100. C. CO2 washout 101. D. Lines of... 102. b. Do a 2nd sepsis...
12 Jaypees Triple A
103. Not a component of APGAR score is? 105. Which of the following is not true about late onset
a. Muscle tone (DNB 2010) Hemorrhagic disease of newborn (HDN)?
b. Color of body (AI 2006, MHPGM-CET 2010)
c. Heart rate a. Begins between 2-7 days of life
d. Respiratory rate Ref: OP Ghai 8/e p137 b. Intracranial Hemorrhage is common
c. Bilary atresia can predispose
104. The parameters used in APGAR score include all of the
d. Warfarin therapy is associated
following except: (MP PG 2010) Ref: Care of Newborn by Maherbansingh 6/e p336; Ghai 8/e p137
a. Respiratory rate
106. Bronchiolitis obliterans is caused by: (Kerala PG 08)
b. Heart rate
a. Respiratory syncytial virus
c. Muscle tone b. Adeno virus
d. Reflex stimulation c. H. influenza
Ref: Ghai 8/e p126; Nelsons 18/e p679 d. Mycoplasma
Ref: Ghai 8/e p381; Nelson textbook of Pediatrics, 17/e p1422; 18/e p1781
PEDIATRICS

Ans. 103. d. Respiratory rate 104. a. Respiratory rate 105. a. Begins between 2-7... 106. a and b
2. GROWTH AND DEVELOPMENT

A. Normal Growth and Developmental Milestones


B. Adolescent
C. Miscellaneous
14 Jaypees Triple A

GROWTH AND DEVELOPMENT (QUESTIONS)

A. NORMAL GROWTH AND d. 5 and 6 years


Ref: Ghai 8/e p42; 7/e p29-30
DEVELOPMENTAL MILESTONES
7. A normally developing 10-month-old child should be able
1. True about head circumference measurement: (PGI Dec 04) to do all of the following except: (AIIMS Nov 05; AI 06)
a. Measured in supraorbital ridge a. Stand alone
b. Measures hydrocephalus/microcephaly b. Play peek a boo
c. Serial measurement is useful c. Pick up a pellet with thumb and index finger
d. Helps in measurement of neurological development d. Build a tower of 3-4 cubes
e. Paediatric intelligence Ref: Ghai 8/e p49, 50; 7/e p28-29
Ref: Ghai 8/e p19, 27; 7/e p15-17, 6/e p4; 8. A child climbs with alternate steps, builds a tower of 8-9
Nelson 18/e p54 and 17/e p1975; Manual of Neonatal Care 5/e, p50 cubes, tells 1 but not his name and cannot say his age and
sex the probable age is: (AIIMS May 01)
2. A neonate is able to: (PGI Dec 00)
a. 36 months
a. Fix his gaze at a object 8 to 12 inches apart
b. 24 months
b. Focus on bright object
c. 30 months
c. Lift his head and chest on elbow
d. 48 months
d. Roll from side to side
Ref: Ghai 8/e p50; 7/e p30-31; Nelson 18/e p44 and 17/e p38-39
Ref: Ghai 8/e p24; 7/e p18-19; Nelson 18/e p44 and 17/e p39
9. Developmental examination should be further evaluated in
3. A 6-year-old child has an IQ 50. Which of the following
child of 12 weeks if the child: (AIIMS June 00; AI 02)
tasks the child can do: (AI 07)
a. Does not vocalize
a. Draw a triangle
b. Does not babble
b. Recognize colors
c. Does not change bright red ring from one hand to other
c. Ride a bicycle even if given in hand
d. Read a sentence Ref: Ghai 8/e p42; 7/e p562 d. Does not hold head at 90
4. A child has started mouthing objects, shows likes and Ref: Ghai 8/e p49; 7/e p26-29; Nelson 18/e p4417/e p39
dislikes has and not yet developed stanger anxiety. The age 10. 10-month-old child can not do: (PGI Dec 05; June 04)
of child is: (AI 07) a. Change cube from one hand to another
a. 3 months b. Can build a cube of 6
b. 5 months c. Can pull and stand
c. 7 months d. Can talk a sentence of 4 to 5 words
d. 9 months e. Pincer grasp
Ref: Ghai 8/e p42; 7/e p30; Nelson 18/e p47 and 17/e p36 Ref: Ghai 8/e p44; 7/e p26-29; Nelson 18/e p44 and 17/e p39
5. A two-month-old child is able to: (AI 04) 11. A 3-month-old can do: (PGI June 05; Dec 01)
a. Show a positive parachute protective reflex a. Social smile
b. Hold head steady in seated position b. Can sit without support
c. Lift head and chest off a flat surface with extended c. Transfer objects from right to left
elbows d. Hold his neck
PEDIATRICS

d. Sustain head level with the body when placed in ventral e. Can change position from prone to supine
suspension Ref: Ghai 8/e p42; 7/e p26-29, 6/e p44;
Ref: Ghai 8/e p24; 7/e p23-28; Nelson 18/e p44 and 17/e p34, 1979 Nelson 18/e p44 and 17/e p39
6. A female child has recently learned to eat with spoon 12. 2-year-old child can do: (PGI Dec 04)
without spilling; to dress and undress herself with a. Ride tricycles
supervision: and to understand that she is a girl. These b. Climb up and down stairs with one each time
skills are FIRST mastered between the ages of: c. Knows sex and age
a. 2 and 3 years (AIIMS Nov 05) d. Handles spoon well
b. 3 and 4 years e. Can read story with picture
c. 4 and 5 years Ref: Ghai 8/e p45; 7/e p26-29; Nelson 18/e p44 and 17/e p39

Ans. 1. a, b, c and d 2. b. Focus on bright... 3. b. Recognize colors 4. b. 5 months


5. d. Sustain head... 6. a. 2 and 3 years 7. d. Build a tower... 8. c. 30 months
9. a and d 10. b and d 11. a and d 12. d. Handles spoon well
Growth and Development 15
13. An 18-month-old infant can do all except: (PGI Dec 03) 20. X-ray of which of the following would be most appropriate
a. Climbing upstairs for determining bone age at 7 months?
b. Can follow mothers activities a. Shoulder (MHPGM-CET 2010)
c. Can turn 2-3 pages at a time b. Medial end of clavicle
d. Can say two or three words c. Wrist
d. Knee Ref: OP Ghai 8/e p11; 7/e p4
e. Can make tower of 8 cubes
Ref: Ghai 8/e p44; 7/e p26-29; Nelson 18/e p44 and 17/e p 39 21. Child begins to sit with support, able to transfer objects
from one hand to another hand and speak monosyllabic
14. Which of the following cannot be done by 3-year-old child:
babbles at the age of: (MHPGM-CET 2010)
a. Draw a triangle (PGI Dec 02)
a. 3 months
b. Draw a circle b. 6 months
c. Arrange 9 cubes c. 9 months
d. Go up and down stains d. 12 months
e. Stand on one foot for 5 second Ref: Ghai 8/ep49-55; Nelson paediatrics 18/e p44 Table 8-1
Ref: Ghai 8/e p44; 7/e p26-29; Nelson 18/e p44 and 17/e p39 22. A boy can climb up and down stairs, One step at a time,
15. Which of the following is the best indicator of longterm Jump, build a tower of 10 cubes, Open a door, use 3 words
nutritional status of a child? (AIIMS May 2010) in a sentences. What is his minimum age?
a. Weight a. 20 months
b. Mid arm circumference b. 24 months
c. Rate of weight gain and linear growth c. 30 months
d. Head circumference Ref: Ghai 8/e p21; 7/e p5, 6 d. 36 months Ref: Nelson 18/e p45, Ghai 8/e p49-55
23. The milestones achieved at 13 months in children are all
16. All are seen in fetal alcohol syndrome except:
except: (DP PGMEE 2009)
a. Microcephaly (AIIMS Nov 2009)
a. Index finger approach
b. Poor coordination
b. Walking
c. Overgrowth of body c. Casting
d. Flat face Ref: Internet d. Single words Ref: Ghai 8/e p49-55, Nelson 18/e p43-54
17. A normal child develops the ability to use 10 words with 24. A 5-year-old child is assessed to have developmental age of
meaning at the age of: (Karnatka PG 2011) one year. His developmental quotient would be:
a. 12 months a. 100 (DP PGMEE 2010)
b. 15 months b. 80
c. 18 months c. 60
d. 24 months Ref: OP Ghai 8/e p55; 7/e p28 d. 20 Ref: Ghai 8/e p54
18. The weight of the child at 3 years is usually ______ times the 25. A female child has recently learned to eat with spoon
birth weight. (Karnataka PG 2011) without spilling; to dress and undress herself with
a. 3 supervision: and to understand that she is girl. These skills
b. 4 are FIRST mastered between the ages:
a. 2 and 3 years (Feb DP PGMEE 2009)
c. 5
b. 3 and 4 years

PEDIATRICS
d. 7
c. 4 and 5 years
Ref: Ghai 8/e p13; Child development: Birth to adolesence,
d. 5 and 6 years
Dr Rajesh Dixit, 2006 p38; Health, Safety and
Ref: Ghai 8/e p49-55
Nutrition for the young child, Lynn R Marotz, p26
26. What milestone of 6 month old child can do?
19. Birth length is double at the age of: (UP PG 09) a. Pick up of object between index finger and thumb
a. 1 year b. Prone to supine (Raj PGI 2008)
b. 2 years c. Standing by holding the furniture
c. 3 years d. Crawling
d. 4 years Ref: Ghai 8/e p11; 7/e p6 Ref: Ghai 8/e p49-55

Ans. 13. a and e 14. a. Draw a triangle 15. c. Rate of weight... 16. c. Overgrowth of body
17. c. 18 months 18. c. 5 19. d. 4 years 20. a. Shoulder
21. b. 6 months 22. d. 36 months 23. c. Casting 24. d. 20
25. b. 3 and 4 years 26. b. Prone to supine
16 Jaypees Triple A
27. Expected three times birth weight of baby at what age: 35. First sign of puberty in girls: (PGI Dec 99, AI 08)
a. 1 year (Raj PG 2009) a. Puberchy
b. 2 year b. Thelarchy
c. 3 year c. Growth spurt
d. 5 year (Ref: Ghai 8/e p13) d. Menarche Ref: Ghai 8/e p64, 65; 7/e, p44
28. Membraneous gap seen between fetal skull bone is known 36. A 13-year-old boy has bilateral gynecomastia. His height is
as: (Kerala PG 10) 148 cm, weight 58 kg; the sexual maturity rating is stage 2.
a. Fontanelle The gynecomastia is most likely due to: (AIIMS Nov 04)
b. Suture a. Prolactinoma
c. Wormian bone b. Testicular tumor
d. Craniosynostosis c. Pubertal gynecomastia
Ref: Ghai 8/e p11; DC Dutta p84, Nelson 18/e p677 d. Chronic liver disease
29. Which among the following is order of development is not Ref: Nelson 18/e p60-62; Ghai 8/e p65
characteristic: (Kerala PG 10) 37. Which of the following is the first sign of puberty in girls?
a. Caudal-Cranial a. Pubarche (MHPGM-CET 2010)
b. Cranial -Caudal b. Thelarche
c. Generalised-Specific c. Menarche
d. Distal-Proximal Ref: OP Ghai 8/42 d. Growth spurt Ref: OP Ghai 8/e p531-63; 7/e 498
30. What is the age of child can walks up and down stairs is:
a. 2 years (UP PG 2010)
b. 3 years C. MISCELLANEOUS
c. 5 years
d. 6 years Ref: Ghai 8/e p49-55; 7/e p27 38. Which of the following is true about eruption ofteeth:
a. Premolar appear in primary dentition (PGI Dec 05)
31. The height of an average Indian child is 100 cms by this
b. Incisors appear first in secondary dentition
age: (J & K PG 2010)
c. 3rd molar is last to develop
a. 2 years
b. 4 years d. Hypothyroidism delays dentition
c. 8 years e. Canines is last to appear in primary dentition
d. 6 years Ref: OP Ghai 8/e p13 Ref: Ghai 8/e p12; CPDT 18/e p452; Nelson 18/e p47, 73
39. True about dentition: (PGI June 04)
a. Hypothyroidism causes delayed dentition
B. ADOLESCENT b. Premolar is not seen in primary dentition
c. 3rd molar is the last to appear in secondary dentition
32. The following are characteristics of autism except: (AI 06)
d. Canine is the first in primary dentition
a. Onset after 6 years of age
e. Incisor is the first in secondary dentition
b. Repetitive behavior
Ref: Ghai 8/e p12, 13; Nelson 18/e p57, 73, 74 and 17/e, p1204-1206
c. Delayed language development
d. Severe deficit in social interaction 40. What is meant by Suppositions child: (PGI Nov 08)
Ref: Ghai 8/e p65; 7/e p40; Nelson 18/e p133 and 17/e p93 a. Second born of a twin pregnancy
b. Child born out of wed lock
33. A 10-year-old child is always restless inattentive to study
and always wants to play outside. Parents are extremely c. Child rear up by a women and she claims the child
PEDIATRICS

distressed, what would you advise? (AIlMS 08) d. Heterozygous mother and father
a. Its a normal behavior e. Homozygous mother and father Ref: Parikh 6/e p5.27
b. Behavior therapy 41. Which of the following is not true about the autistic specific
c. Its a serious illness requires medical treatment disorders? (AI 2010)
d. Needs change in environment Ref: Ghai 8/e p63; 7/e p39 a. Impaired communication
34. WHO defines adolescent age between: (AI 05) b. Impaired imagination
a. 10-19 years c. Language development is delayed
b. 10-14 years d. Vision problems
c. 10-25 years Ref: Nelsons Pediatrics 17/e p93; Ahuja Psychiatry 6/e p263;
d. 9-14 years Ref: Ghai 8/e p63 OP Ghai 7/e p40

Ans. 27. a. 1 year 28. a. Fontanelle 29. a. Caudal-Cranial 30. a. 2 years


31. b. 4 years 32. a. Onset after... 33. b and d 34. a. 10-19 years
35. b. Thelarchy 36. c. Pubertal... 37. b. Thelarche 38. b, c and d
39. a, b, c and e 40. c. Child rear up... 41. d. Vision problems
Growth and Development 17
42. Stork bite lesion seen: c. 9 months
a. Sturge weber syndrome d. 1 year
b. KJippel fei! syndrome Ref: OP Ghai 8/e p53; 7/e p153
c. Biue rubber bleb nevus syndrome 45. A child is below third percentile for height. Hia growth
d. Macular staining of infants velocity is normal, but cgronogical age is more than skeletal
e. Craniofacial nevus age. The most likely diagnosis is: (DP PGMEE 2010)
Ref: Nelson 18/e p2662 a. Constitutional delay in growth
43. Aspergers syndrome is a: (JIPMER 2002) b. Genetic short stature
a. Developmental delay c. Primordial dwarfism
b. Neuromuscular disease d. Hypopituitarism
c. Degenerative disorder Ref: Ghai 8/e p37
d. Metabolic disorders 46. Perinatal Period corresponds to: (J & K PG 2011)
Ref: Ghai 8/e p45; Nelson 18/e p136 a. 28 weeks of gestation to 07 days after birth
44. Pincer grasp is achieved at: (Karnatka PG 2010) b. Period of Labour to new born period
a. 5 months c. Third trimester of pregnancy to new born period
b. 7 months d. 36 weeks of gestation to 3 days after birth
Ref: Ghai 8/e p125

PEDIATRICS

Ans. 42. e. Craniofacial nevus 43. a. Developmental delay 44. c. 9 months 45. a. Constitutional...
46. a. 28 weeks of gestation...
3. NUTRITION IN HEALTH AND
DISEASE

A. Breast Milk/Breast Feeding


B. Nutritional Status Indictors
C. Protein Energy Malnutrition
D. Rickets
E. Vitamins and their Deficiencies
F. Miscellaneous
Nutrition in Health and Disease 19

NUTRITION IN HEALTH AND DISEASE (QUESTIONS)

A. BREAST MILK/BREAST FEEDING 8. Benifits of breast milk are: (PGI June 08)
a. Better nutrition
1. The protective effects of breast milk are known to be b. Less infection
associated with: (AI 05) c. More diarrhea
a. IgM antibodies d. Less allergy
b. Lysozyme e. High sodium contents Ref: Ghai 8/e p150
c. Mast cells 9. True about cows milk are all except: (AIIMS May 07)
d. IgA antibodies a. Cows milk contains 80% whey protein not casein
Ref: Ghai 8/e p150; 7/e p123; Nelson 18/e p215 and 17/e p158 b. Cow milk has less carbohydrate than mothers milk
2. Breast milk storage in a refrigerator is upto: (PGI Dec 98) c. Has more K4 and Na+ than infant formula feeds
a. 4 hours d. Has more protein than breast milk
b. 8 hours Ref: Nutrition and child development Flizabeth 2/e p18; Ghai 8/e p148
c. 12 hours 10. Infant can breathe while sucking breast milk because of:
d. 24 hours Ref: Ghai 8/e p152 a. Small wide tongue
3. Exclusive breast feeding may be associated with the b. High place larynx
following except: (AI 98) c. Small pharynx
a. Hemolysis due to Vit K deficiency d. Short soft palate Ref: Ghai 8/e p374
b. Evening colic 11. Adequate breast feeding in baby indicates: (MP PG 2008)
c. Golden color stool a. Baby sleep 2 hour after feeds
d. Prolongation of physiological jaundice b. Does not cry frequently
Ref: Ghai 8/e p152; Nelson 18/e p217 and 17/e p158, 164 c. Urine 5-7 times/day
4. Exclusive breast feeding is at least till: (AI 98) d. Stool 4 times/day
a. 4 months Ref: Ghai 8/e p151; 7/e p452
b. 6 months 12. A drug excreted in breast milk is harmful to infant. But it is
c. 8 months us, in some situations like: (MP PG 2008)
d. 10 months Ref: Ghai 8/e p150; 7/e p122 a. Breast carcinoma
5. Recommendation of WHO for baby friendly hospital is: b. HIV therapy
a. With hold breast feed for 1 day after child birth c. Methotrexate therapy
b. Only breast feed, no other feed (AIIMS June 98) d. Morphine therapy
c. For 24 hours baby is kept with the mother Ref: Ghai 8/e p183; 7/e p159
d. Within 4 hours of birth, breast feed is started 13. After premature delivery, mothers milk is low in:
Ref: Meharban Singh 6/e p167; Ghai 8/e p155 a. Lactose (MHPGM-CET 2010)
6. True about mothers milk: (PGI Nov 09) b. Fat
a. Best even in preterm baby c. Protein
b. 30% energy from protein 10% d. Sodium Ref: OP Ghai 8/e p151; 7/e p131, 134
c. Lactoferrin promotes lactobacilli in babys gut 14. The Baby-friendly Hospital Initiative (BFHI) is not an
d. Prevents allergies in baby initiative: (AP 2010)

PEDIATRICS
e. Prevents asthma Ref: Ghai 8/e p150; 7/e p123 a. Every 4 hourly breast-feeding
7. The current recommendation for breast feeding is that: b. Give breast-feeding to infants with within 4 hrs of
a. Exclusive breast feeding should be continued till 6 month caesarean
of age followed by supplementation with additional c. Initiate Breast-feeding within a half-hour of birth
foods (AI 04) d. Encourage breast-feeding on demand
b. Exclusive breast feeding should be continued till 4 month Ref: Ghai 8/e p151
of age followed by supplementation with additional 15. The protective effects of breast milk are known to be
foods associated with: (Feb DP PGMEE 2009)
c. Colostorum is the most suitable food for a newborn baby a. IgM antibodies
but it is best avoided in first two days b. Lysozyme
d. The baby should be allowed to breast feed till one year of c. Mast cells
age Ref: Ghai 8/e p152 d. IgA antibodies Ref: Ghai 8/e p150

Ans. 1. d. IgA antibodies 2. d. 24 hours 3. b. Evening colic 4. b. 6 months


5. b > c 6. a, c, d and e 7. a. Exclusive breast... 8. a, b and d
9. a. Cows milk contains.. 10. b. High place larynx 11. c > a 12. b. HIV therapy
13. a. Lactose 14. a. Every 4 hourly... 15. d. IgA antibodies
20 Jaypees Triple A
16. Which among the following is the best method of breast 24. Short stature is seen in: (PGI June 02)
feeding in an infant of HIV mother india: (Kerala PG 10) a. Maternal deprivation syndrome
a. Breast feed till 6 months and abrupt weaning b. Hypothyroidism
b. Breast feeding for one year c. Bulimia
c. Completely avoid breast feeding as there is risk of d. Paternal smoking
transmission e. IUGR Ref: Ghai 8/e p21
d. Exclusive breast feeding as long as possible 25. Common to both acute and chronic malnutrition is:
(Ref: Ghai 8/e p183; Nelson 18/e p215, 1429) a. Weight for age (AIIMS May 07)
17. Which major nutrient is richly present in mothers hind b. Weight for height
milk: (J and K PG 2010) c. Height for age
a. Fat d. BMI
b. Protein Ref: Indian Academy of Pediatrics 3/e p127; Ghai 8/e p102
c. Carbohydrate 26. Deficit in weight for height in a 3 years old child indicates:
d. Water Ref: OP Ghai 8/e p151 a. Acute malnutrition (AIIMS Nov 05)
b. Chronic malnutrition
c. Concomitant acute and chronic
B. NUTRITIONAL STATUS INDICATORS d. Under weight
Ref: See above explanation; Ghai 8/e p98
18. Nutrition in community is assessed by all except: (AI 09)
a. Hb < 11.5 gm% in 3rd trimester pregnancy 27. Which of the following is the best indicator of long term
b. 1-4 yr mortality rate nutritional status: (Al 07)
c. Ht and wt of children a. Mid arm circumference
d. <BW < 2500 gm Ref: Nelson, 18/e p229-232; PSM p236 b. Height for age
c. Weight for age
19. Best indicator for nutritional status for a child is:
d. Weight for height Ref: Ghai 8/e p97, Park 19/e, p434
a. Mid arm circumference (AIIMS Nov 06)
b. Head circumference 28. Chronic malnutrition is judged by: (DNB 2009)
c. Rate of increase of height and weight a. Weight for age
d. Chest circumference b. Height for age
Ref: Ghai 8/e p32; 7/e p6-7; Nelson 18/e p225 and 17/e p170 c. Weight for height
d. BMI
20. Acute malnutrition is manifested by: (PGI June 05)
Ref: Ghai 8/e p254; Essentials of Paediatric Nelson 4/e p71-75, 88, 68
a. Weight for age
b. Weight for height
c. Age for height C. PROTEIN ENERGY MALNUTRITION
d. Brocas index
e. Ponderal index 29. Kwashiorkar is characterized by all of the following
Ref: Ghai 8/e p110; 7/e p62; Park 18/e p402; 19/e, p343 features except: (AI 99)
21. The following statement about Gomez classification is a. Edema
false: (Al 08) b. Patchy depigmentation of hair
a. Based on height retardation c. Fatty liver
b. Based on 50th percentile Boston standards d. Fatty infiltration of pancreas
c. Between 75 and 89% implies in malnutrition Ref: Ghai 8/e p104; 7/e p66-67; Robbins 7/e p448-449
d. This classification has progn and stip value for 30. Caloric supplementation required for a severely
hospitalization children Ref: Ghai 8/e p102
PEDIATRICS

malnourished child (per kg-body weight) is: (AI 99)


22. The most common cause of short stature is: (Al 08) a. 100 cal/kg
a. Constitutional b. 125 cal/kg
b. Systemic diseases c. 150 cal/kg
c. Hypothyroidism d. 175 cal/kg Ref: Ghai 8/e p106; 7/e p70-71
d. Growth hormone deficiency Ref: Ghai 8/e p12 31. The important fatty acid present in breast milk, which is
23. The most common cause of short stature is: (AI 07, 08) important for growth is: (AIIMS Nov 06)
a. Constitutional delay a. Docosahexaenoic acid
b. Growth hormone deficiency b. Palmitic acid
c. Cretinism c. Linoleic acid
d. Achondroplasia d. Linolenic acid
Ref: Ghai 8/e p22; Nelson 18/e p1851 and 17/e p1851 Ref: Ghai 8/e p98; Nelson 18/e p215 and 17/e p154

Ans. 16. c. Completely avoid... 17. a. Fat 18. a. Hb < 11.5 gm%... 19. c. Rate of increase...
20. b. Weight for height 21. a. Based on height... 22. a. Constitutional 23. a. Constitutional delay
24. a, b and e 25. b. Weight for height 26. a. Acute malnutrition 27. b. Height for age
28. b. Height for age 29. d. Fatty infiltration... 30. d. 175 cal/kg 31. a. Docosahexaenoic acid
Nutrition in Health and Disease 21
32. All of the following conditions are observed in Marasmus, 41. All of the following conditions are observed in marasmus,
except: (AIIMS May 05) except: (Feb DP PGMEE 2009)
a. Hepatomegaly a. Hepatomegaly-Seen in Kwashiorkor
b. Muscle wasting b. Muscle wasting
c. Low insulin levels c. Low insulin levels
d. Extreme weakness Ref: Ghai 8/e p102; 7/e p66-67 d. Extreme weakness Ref: Ghai 8/e p99
33. A child is suffering from severe PEM. Calories to be given
per kg of body weight to regain weight: (AIIMS June 99)
a. 175-200 Kcal
D. RICKETS
b. 150 Kcal
42. In Rickets seen: (PGI Dec 06)
c. 400 Kcal
a. ALP
d. 100 Kcal Ref: Ghai 8/e p108; 7/e p70
b. ALP
34. Protein efficiency ratio is: (AIIMS June 98) c. Hypo PO2 4 in blood
a. Biological value X Digestibility coefficient d. Hyper PO2 4 in blood
b. Nitrogen absorbed/Nitrogen intake e. Hyperphosphaturia Ref: Robbins 7/e p453; Ghai 8/e p113
c. Weight gain per unit of protein consumed
43. Rickets in infant present as all except: (AIIMS May 07)
d. Energy from protein/total energy from diet
a. Craniotabes
Ref: Ghai 8/e p108; PSM by Deepak Mishra 3/e p305
b. Widened Fontanelle
35. Acute complications of PEM: (PGI June 06; Dec 02) c. Rachitic Rosary
a. Hypothermia d. Bow legs Ref: Ghai 8/e p113; 7/e p82
b. Hypoglycemia
44. A 2 years child with Vit. D resistant rickets was evaluated.
c. Hypokalemia
His lab investigation reveal. Serum calcium - 9 mg/dl Serum
d. Hyponatremia
phosphate - 2.4 mg/dl Serum alkaline phosphate - 1041 IU
e. Eosinophilia Ref: Ghai 8/e p102; 7/e p68-70
Parathormone level-59 units The likely diagnosis is:
36. Acute complications of PEM: (PGI June 05) a. Distal renal tubular acidosis (AIIMS May 02)
a. Hypothermia b. Vit. D dependent rickets
b. Hypoglycemia c. Hypophosphetemic rickets
c. Hypokalemia d. Hyperparathyroidism
d. Hypermagnesemia Ref: Ghai 8/e p113-117; 7/e p83; Nelson 17/e p2345
e. Eosinophilia Ref: Ghai 8/e p102; 7/e p68-70
45. Basanti a 7 year old girl, presents with recent onset of genu
37. Kwashiorkor is characterized by all except: (UP PG 09) valgum and difficulty in walking. On X-ray examination
a. Alertness there is metaphyseal widening and osteoporosis.
b. Edema Investigations showed a serum calcium of 9 mg/dl. Serum
c. Flag sign phosphorus of 2.5mg/dl and alkaline phosphatase of 30Ka
d. Hepatomegaly Ref: Ghai 8/e p99; 7/e p67 units, the possible cause is: (AIIMS Nov 00)
38. In Kwashiorkor, the letter K is post-fixed to denote: a. Nutritional rickets
a. Weight for height (MHPGM-CET 2006, 2010) b. Hypophosphatemic rickets
b. Skin change c. Azotemic renal dystrophy
c. Edema d. Primary Hyperparathyroidism
d. Muscle wasting Ref: OP Ghai Pediatric 8/e p99 Ref: Ghai 8/e p115; 7/e p83; Nelson 18/e p254 and 17/e p2345
39. The skin changes seen in protein energy malnutrition can 46. True about vit - D deficiency rickets: (PGI Dec 04)

PEDIATRICS
be due to deficiency all of the following nutrients except: a. Vit. D3 given at a dose of 50-150 ing/dl
a. Zinc (DP PGMEE 2009) b. X-ray knee joint is diagnostic
b. Tryptophan c. Rickety rosary is tender
c. Essential fatty acids d. Increased chances of respiratory tract infection
d. Pyridoxine e. Hyponatremia Ref: Ghai 8/e p113; 7/e p82-84
Ref: Ghai 8/e p102 47. True about nutritional rickets: (PGI Dec 03)
40. Mostly death in PEM is due to all except: a. Craniotabes
a. Hypothermia (Feb DP PGMEE 2009) b. Multiple fracture
b. CCF c. Widening of wrist
c. Worm infestation d. Phosphate in serum
d. Electrolyte imbalance e. Growth retardation
Ref: Ghai 8/e p102 Ref: Ghai 8/e p113; 7/e p82-84; Nelson 18/e p253-262 and 17/e p186-188

Ans. 32. a. Hepatomegaly 33. a. 175-200 Kcal 34. c. Weight gain per 35. a, b, c and d
36. a, b and c 37. a. Alertness 38. c. Edema 39. d. Pyridoxine
40. c. Worm infestation 41. a. Hepatomegaly... 42. a, c and e 43. d. Bow legs
44. c. Hypophosphetemic... 45. b. Hypophosphatemic... 46. a and d 47. a, c, d and e
22 Jaypees Triple A
48. Familial hypoposphatemic Rickets is Characterized by: 56. Earliest sign of Rickets is: (UP PG 09)
a. Increased forehead sweating (PGI June 03) a. Craniotabes
b. Characteristically low Ca2+ b. Harrisons groove
c. Anterior fontanelle widened c. Rachitic rosary
d. Increased alk. Phosphatase d. Pigeon breast Ref: Ghai 8/e p113; 7/e p18
Ref: Ghai 8/e p113; 7/e p83; Nelson 18/e p254 and 17/e p2345-2346
49. Seen in rickets all except: (PGI June 98)
a. Cupping of metaphysis
E. VITAMINS AND THEIR DEFICIENCIES
b. Defective mineralization
57. About scurvy true all except: (PGI June 00)
c. Epiphyseal dysgenesis
a. Subperiosteal hematoma with tenderness
d. Defective osteoid formation
Ref: Ghai 8/e p113, 114; 7/e p82-84; Nelson 17/e p186-188 b. Separation of epiphysis
c. Increased alkaline phosphatase
50. All of the following are seen in rickets, except:
d. Gingival bleeding
a. Bow legs (AIIMS May 03)
Ref: Ghai 8/e p110; 7/e p90-91; Nelson 18/e p243-250 and 17/e p184-186
b. Gunstock deformity
c. Pot belly 58. Following are radiological features of scurvy except:
d. Craniotabes a. Bony thickening (PGI Dec 99)
Ref: Ghai 8/e p114; Nelson 18/e p253-262 and 17/e p186-188 b. Metaphyseal widening
c. Metaphyseal calcification
51. A common finding in osteomalacia is: (PGI June 99)
d. Epiphyseal separation
a. Low serum phosphate
Ref: Ghai 8/e p112; 7/e p90-91; Nelson 18/e p243-250 and 17/e p184-186
b. Normal level of 1, 25 di-hydroxy vit D3
c. Low serum calcium 59. All are true about scurvy except: (AIIMS May 09)
d. Increased hydroxy proline in urine a. Skeletal changes in adult occur with clinical deficiency of
Ref: Nelsons Essentials of Pediatrics 5/e, p152; Vit.C
18/e p253-262; Ghai 8/e p113 b. Defective proximal calcification is the central cause for
52. A 2-year-old boy has vitamin D resistant rickets. bone change
His investigations revealed serum Calcium-9 mg/dl, c. Cartilaginous overgrowth results in widening of
Phosphate-2.4 mg/dl, alkaline phosphatase-1041IU, normal epiphyseal plate
intact parathyroid hormone and bicarbonate 22 mEq/L. d. Bowing of legs Ref: Ghai 8/e p110; 7/e p91
Which of the following is the most probable diagnosis? 60. Pseudoparalysis in an infant is suggestive of:
a. Distal renal tubular acidosis (AIIMS May 04; Nov 02) a. Acute Rheumatic fever (PGI 06)
b. Hypophosphatemic rickets b. Vitamin B6 deficiency
c. Vitamin D dependent rickets c. Vitamin E deficiency
d. Hypoparathyroidism d. Vitamin C deficiency Ref: Ghai 8/e p112; 7/e p91
Ref: Ghai 8/e p113; 7/e p83; Nelson 17/e p2342 61. Vitamin A deficiency is characterized by: (PGI Dec 04)
53. Not seen in Rickets is: (DNB 2009) a. Bitot spot
a. Craniotabes b. Xerophthalmia
b. Bow legs c. Night blindness
c. Increased alkaline phosphatase d. Trantas spot
d. Increased acid phosphatase Ref: Ghai 8/e p110, 112
Ref: Op Ghai 8/ep230; Nelson Text book of Peds 18/e
62. Daily dose of vitamin A in a 6-12 months old child is:
54. True statement about vitamin D resistant rickets is: a. 500 microgram (AIMS June 97)
PEDIATRICS

a. X linked recessive is commonest (DNB 2011) b. 200 microgram


b. Defect in proximal tubule absorption c. 300 microgram
c. Hyperphosphatemia d. 700 microgram Ref: Ghai 8/e p120
d. No end organ resistance to 1, 25 (OH)2 D3
63. The vitamin A supplement administered in Prevention of
Ref: Ghai 8/e p238; Nelson Ped, 18/e p2720
nutritional blindness in children program contain:
55. 25 Hydroxylatio of Vit-D (Cholecalciferol) occurs in: a. 25,000 lU/ml (AI 03, AIIMS Nov 05)
a. Liver (MP PG 2009) b. 1 Lakh Ill/ml
b. Skin c. 3 Lakh TU/ml
c. Kidney d. 5 Lakh lU/ml
d. Intestine
Ref: Ghai 8/e p110; IAP 1/e p417, Nutrition and child
Ref: Ghai 8/e p112; Nelsons 18/e p2841
development KE Elezabeth 3/e p94

Ans. 48. a, c and d 49. c and d 50. b. Gunstock deformity 51. a. Low serum...
52. b. Hypophosphatemic... 53. d. Increased acid... 54. b. Defect in proximal... 55. a. Liver
56. a. Craniotabes 57. c. Increased alkaline... 58. b. Metaphyseal widening 59. b. Defective proximal...
60. d. Vitamin C deficiency 61. a, b and c 62. c. 300 microgram 63. b. 1 Lakh Ill/ml
Nutrition in Health and Disease 23
64. Which vitamin deficiency can result in Neonatal Seizures?
a. Thiamine (AIIMS Nov 09)
F. MISCELLANEOUS
b. Pyridoxine
72. A child with alopecia, hyperpigmentation psoriatic
c. Cyanocobalamin
dermatitis in genitals and mouth and hypogonadism is
d. Vitamin-C
likely to be suffering from: (AI 98)
Ref: Harrison Principles of Internal Medicine, 17/e p2502; Ghai 8/e p182
a. Cu deficiency
65. Hypervitaminosis of which of the following will cause b. Iron deficiency
bony abnormalities: (PGI Dec 06) c. Zn deficiency
a. Vit A d. Mg deficiency Ref: Ghai 8/e p121; 7/e p92
b. Vit D
73. Which of the following congenital malformation is seen
c. Vit C
in a child of a mother who is on treatment with oral
d. Vlt E
anticoagulants: (AIIMS Nov 00)
e. Vit K
a. Craniofacial malformations
Ref: Ghai 8/e p118; 7/e p79, 81, 6/e p121, Nelson 18th p245
b. Renal agenesis
66. Large doses of vitamin K in new borne cause: (DNB 2011) c. Long bone defects
a. Hemolysis d. Chondrodysplasia punctate
b. Convulsions Ref: Katzung 9/e, p997; Internet
c. Decreased albumin
74. True about Fructosan, a Prebiotic? (AI 2010)
d. Posterior fontanella buldging
a. Dietary supplement containing potentially beneficial
Ref: KD Tripathi Pharmacology, 7/e p596; Gahi 8/e p410
bacteria or yeast, with lactic acid bacteria
67. Recommended daily allowance of vitamin D in children is: b. Non-digestible food ingredient that beneficially affect the
a. 400 I.U Per day (MP PG 2010) host by-selectively stimulating the growth of bacteria or
b. 600 I.U Per day yeast
c. 800 I.U Per day c. Digestible food ingredient that beneficially affect the host
d. 1000 I.U Per day by selectively inhibiting the growth of bacteria
Ref: Ghai 8/e p112; Nelsons 18/e p257 d. Non-digestible food ingredient that selectively inhibit the
68. The recommended oral dose of vitamin A to be given in a 10 growth of bacteria or yeast
month child with deficiency on each of day 1, 2 and 14 is: 75. Zinc deficiency causes: (PGI Dec 03)
a. 50,000 IU (Karnataka PG 2011) a. Sexual infant ilism
b. 1,00,000 IU b. Loss of libido
c. 2,00,000 IU c. Poor weight gain
d. 6,00,000 IU d. Poor wound healing
Ref: OP Ghai 8/e p110; 7/e p80 Ref: Nelson 18/e p266, Ghai 8/e p122; 7/e p92
69. Recommended daily dietary allowance of vitamin A for an 76. Elemental iron and folic acid content of pediatric iron folic
infant is: (Karnataka PG 2010) acid tablet supplied under RCH program: (AI 03)
a. 100-200 mg a. 20 mg iron and 100 microgram folic acid
b. 300-400 mg b. 40 mg iron and 100 microgram folic acid
c. 500-600 mg c. 40 mg iron and 50 microgram folic acid
d. 750 mg d. 60 mg iron and 100 microgram folic acid
Ref: Ghai 8/e p110; Park 20/e p533, Table 8 Ref: Nutrition and child development by K.E
70. Hypervitaminosis is damage to: (UP PG 09) Elizabeth 3/e, p111, 112; Ghai 8/e p123; 7/e p90

PEDIATRICS
a. Lysosome 77. The protein derived from which of the following food items
b. Mycrotubules is considred as reference protein?
c. Endoplasmic reticulum a. Egg (MP PG 2009)
d. Mitochondria b. Milk
Ref: Ghai 8/e p113; 7/e p80 c. Meat
71. Which one of the following is not a pyridoxine dependent d. Soya bean Ref: Ghai 8/e p92; 7/e p57
disorder: (UPSC 07) 78. The daily caloric requirement of a child weighing 15 kg is:
a. Homocystinuria a. 750 kcal (MP PG 2009)
b. Methyl-malonic-acidemia b. 1,000 kcal
c. Cystathioninuria c. 1,250 kcal
d. Xanthurenic aciduria d. 1,500 kcal
Ref: Ghai 8/e p658; Nelson 17/e, 183 Ref: Ghai 8/e p89

Ans. 64. b. Pyridoxine 65. a and b 66. a. Hemolysis 67. a. 400 I.U Per day
68. b. 1,00,000 IU 69. b. 300-400 mg 70. a. Lysosome 71. b. Methyl-malonic-acidemia
72. c. Zn deficiency 73. d. Chondrodysplasia... 74. b. Non-digestible... 75. All of these
76. a. 20 mg iron and... 77. a. Egg 78. c. 1,250 kcal
24 Jaypees Triple A
79. Calorie requirement per day of a child weighing 15kg 80. Which of the following is not given in the early nutritional
would be: (MHPGM-CET 2010) correction of severe malnutrition: (Kerala PG 10)
a. 1150 kcal a. Zinc
b. 1250 kcal b. Iron-given after 4-6 weeks to avoid free radical injury
c. 1450 kcal c. Vit A
d. 1550 kcal d. Vitamin B Complex
Ref: OP Ghai 8/e p88; 7/e p77 Ref: Ghai 8/e p102; Nelson 18/e p232
PEDIATRICS

Ans. 79. b. 1250 kcal 80. b. Iron-given after...


4. FLUID AND ELECTROLYTES
26 Jaypees Triple A

FLUID AND ELECTROLYTES (QUESTIONS)

1. The sodium content of ReSoMal (rehydration solution for c. Glucose 20g


malnourished children) is: (AI 06) d. Sucrose 10 g
a. 90 mmol/L e. Potassium bicarbonate 2.5g
b. 60 mmol/L Ref: PSM by Deepak Mishra 7/e p374379; Ghai 8/e p294
c. 45 mmol/L 8. The composition of ORS recommended by WHO is:
d. 30 mmol/L Ref: Ghai 6/e p109; Ghai 8/e p72, 105 a. 3.5 g NaCl (PGI Dec 01)
2. A 5-year-old boy passed 18 loose stools in last 24 hours and b. 4.5 g NaCl
vomited twice in last 4 hours. He is irritable but drinking c. 2.9 g sodium-potassium citrate
fluids. The optional therapy for this child is: (AI 03) d. 2.8 g sodium bicarbonate
a. Intravenous fluids e. 1.5 g potassium chloride
b. Oral rehydration therapy Ref: PSM by Deepak Mishra 7/e p374379; Ghai p294, 295
c. Intravenous fluid initially for 4 hours followed by oral 9. True about ORS: (PGI Dec 00; Dec 06)
fluids a. Na+ = 90 mEq/L
d. Plain water ad libitum b. K+ = 30 mEq/L
Ref: Ghai 8/e p70-75, 294; 7/e p264 c. CI- - 20 mEq/L
3. Oral rehydration mixture contains glucose and sodium d. HCO3 = 40 mEq/L
because both of them: (AIIMS Nov 04) e. Glucose =111 gm
a. Are needed to maintain the plasma osmolality Ref: PSM by Deepak Mishra 7/e p374379; Ghai 8/e p294
b. Are prominent energy sources for the body 10. Kallu 2-year-child weighing 6.7 Kg presents in the casualty
c. Facilitate the transport of each other from the intestinal with history of vomiting and diarrhea for last 2 days. On
mucosa to the blood examination skin pinch over the ant. abdominal wall go
d. Are required for the activation of sodium potassium quickly to its original position. Interpretation of skin-pinch
ATPase test in this child will be:
Ref: Ghai 8/e p72, 294; 7/e p261-264 a. No dehydration (AI 02)
4. In a patient who has diarrhea and vomiting with inadequate b. Some dehydration
water intake is suffering from: (AIIMS June 00) c. Sever dehydration
a. Intracellular dehydration with hypernatremia d. Skin pinch cannot be evaluated in this child
b. Intracellular dehydration with hyponatremia Ref: Ghai 8/e p72; 7/e p264
c. Extracellular dehydration with hyponatremia 11. A girl of 8 years suffering from vomiting and diarrhea for
d. Extracellular dehydration with hypernatremia last 2 days when pinched on abdomen, skin goes within
Ref: Ghai 8/e p71, 294; 7/e p51-52 seconds, she is most likely to be suffering from:
5. WHO ORS, composition are (mmol): a. No dehydration (AIIMS June 00)
a. Glucose-111 (PGI June 04; AI 99) b. Some dehydration
b. K+ - 80 c. Severe dehydration
c. Na+ -20 d. Skin turgor cannot be commented
d. Cl- - 30 Ref: Ghai 8/e p72; 7/e p264
e. Total osmolarity -311 12. Most sensitive indicator of intravascular volume depletion
Ref: Ghai Ghai 8/e p72, 294; 7/e p263; KDT. 5/e, p 617; in infant is: (AIIMS May 2010)
PEDIATRICS

PSM by Deepak Mishra 7/e p374 379 a. Stroke volume


6. Composition of ORS, which of the following is correct: b. Heart rate
a. Na+ 90 mEq/L (PGI Dec 02) c. Cardiac output
b. HCO3- 10 mEq/L d. Blood pressure
c. K+ 20 mEq/L Ref: L Nelson Textbook of Pediatrics 18/e, p
d. Cl- 5 mEq/L 13. Most dangerous dehydration is: (PGI June 98)
e. Osmolarity 310 a. Hyponatremic
Ref: Ghai 8/e p294; 7/e p263; PSM by Deepak Mishra 7/e p374-379 b. Hypernatremic
7. WHO ORS contains: (PGI June 02) c. Isonatremic
a. Sodium chloride 2.5g d. Non-diarrheal cause
b. Potassium chloride 1.5g Ref: Ghai 8/e p73; 7/e p262

Ans. 1. c. 45 mmol/L 2. b. Oral rehydration therapy 3. c. Facilitate the... 4. c. Extracellular dehydration...


5. a and e 6. a, c and e 7. b and c 8. a, c and e
9. a and e 10. d. Skin pinch... 11. b > c 12. b. Heart rate
13. b. Hypernatremic
Fluid and Electrolytes 27
14. Hypernatremic dehydration is characterized by: d. Any age
a. Serum sodium > 150 mmol/L (PGI Dec 03) Ref: Ghai 8/e p710; Textbook of Pediatric Advanced Life Support,
b. Signs of dehydration are minimal Nelson 18/e p403 and 17/e p292; Bailey and Love 24/e p290
c. ECF volume led 21. A breast fed child presents with hypernatremia (Serum
d. Rapid correction is required sodium > 170m Eq/L). His urine sodium is 70 mEq/L. Which
e. Shift of water from ECF to ICF of the following is the most likely cause: (AIIMS Nov 00)
Ref: Ghai 8/e p72, 73; 7/e p261, 262
a. Diabetes insipidus
15. A child suffering from acute diarrhea is brought to the b. Acute necrosis
casualty and is diagnosed as having severe dehydration c. Severe dehydration
with pH of 7.23- Serum Na-125, Serum K-3, HCO3-16. The d. Excessive intake of sodium
best IV fluid of choice is:
Ref: Ghai 8/e p73; Nelson 18/e p273 and 17/e p197-198;
a. 3% Saline (AIIMS May 01)
Cloherty-Manual of Neonates Care 5/e, p104
b. N/3 Saline+ 10% dextrose
c. Normal saline 22. Oral glucose tolerance test in children done with:
d. N/3 saline + 5% dextrose a. 1.5 gm/kg glucose (PGI Nov 08)
Ref: Ghai 8/e p72; 7/e p266 b. 1.75 gm/kg glucose
16. The requirement of potassium in a child is: c. 2 gm/kg glucose
a. 1-2 mEq/kg/24 hours (AI 06) d. 2.5 gm/kg glucose
b. 4-7 mEq/kg e. 75 gm as an adult
c. 10-12 mEq/kg 23. An alert 6-month-old child is brought with vomiting and
d. 13-14 mEq/kg diarrhea. RR-45/min, HR-180/min, SBP-85 mm of Hg.
Ref: Nelson 18/e p279 and 17/e p203; Ghai 8/e p73 xtremities are cold and mottled. Capillary refilling time is 4
17. A child presents with diarrhea and peripheral circulatory secs. Diagnosis is: (AIIMS May 10)
failure. The arterial pH is 7.0, PC02 15 mm Hg, and PO2 76 a. Early compensated shock due to hypovolemia
mm Hg. What will be the most appropriate therapy: b. Early compensated shock due to supraventricular
(AIIMS Nov 05; Nov 04; May 01) tachycardia
a. Sodium bicarbonate infusion c. Late decompensated shock due to hypovolemia
b. Bolus of Ringers lactate d. Late decompensated shock due to supra ventricular
c. Bolus of hydroxyethyl starch tachycardia
d. 5% Dextrose infusion Ref: Ghai 8/e p715;1. Intermediate emergency care and
Ref: Ghai 8/e p85; 7/e p49-51, 90; transportation of the sick and injured Bv American Academy
Harrison 15/e, p286; 16/e p265;
of Orthopaedic Surgeons; 2. Ghai Essential Pediatrics
Nelson 18/e p301 and 17/e p299-301
24. The severely malnourished child is given: (DNB 2011)
18. Which of the following would be the plasma osmolality of
a. Dextrose
child with plasma Na+ 125 mEq/L, glucose of 108 mg/dl, and
b. Albumin
blood urea nitrogen (BUN) of l40 mg/dl? (AIIMS May 05)
a. 360 mOsm/kg c. Lactate
b. 306 mOsm/kg d. Thiamine
c. 312 mOsm/kg Ref: OP Ghai 8/e p345; 7/e p69
d. 318 mOsm/kg 25. The initial fluid of choice of the treatment of hypernatremic
Ref: Nelson 18/e p270 and 17/e p193; Ghai 8/e p83 dehydration is: (MP PG 2009)
19. A child had repeated vomiting and developed metabolic a. Normal saline

PEDIATRICS
alkalosis. The treatment given is: (AIIMS June 99) b. N/2 saline
a. Ringer lactate c. N/4 saline
b. IV normal saline and potassium d. D/4 saline + 5% dextrose
c. ORS Ref: Nelsons 18/e p274; Ghai 8/e p73; 7/e p25, 54
d. IV normal saline Ref: Ghai 8/e p87; 7/e p50-51 26. Hyponatremia produces confusion and rstlessness at a
20. In Pediatric advanced life support, interosseous access for serum level of: (MP PG 2009)
drug/fluid administration is recommended for pediatric age a. < 130 Meq/L
of: b. < 120 Meq/L
a. < 1 year age (AIIMS Nov 02) c. < 100 Meg/L
b. < 5 years age d. < 100 Meq/L
c. < 6 years age
Ref: Ghai 8/e p74; 7/e p51

Ans. 14. a and b 15. c. Normal saline 16. a. 1-2 mEq/kg/24 hours 17. b. Bolus of Ringers lactate
18. b. 306 mOsm/kg 19. b. IV normal saline... 20. c. < 6 years age 21. d. Excessive intake...
22. b. 1.75 gm/kg glucose 23. a. Early compensated... 24. a. Dextrose 25. a. Normal saline
26. b. < 120 Meq/L
28 Jaypees Triple A
27. Sodium content in mmol/L in WHO oral rehydration c. 80 %
solution is: (Karnataka PG 2011) d. 90 %
a. 20 Ref: Ghai 8/e p147; Nelson, 17/e p191 Fig. 45-1, 18/e p267
b. 80 32. Maintenance of fluid requirement in 23 kg girl is:
c. 90 a. 1650-ml/day (UP PG 2010)
d. 111 Ref: Ghai 8/e p294; 7/e p263, table 10.7. b. 1560-ml/day
28. Daily water requirement in child weighing 30 kgs, height c. 1120-ml/day
123 m and BSA of 1 m2 is: (DP PGMEE 2009) d. 1060-ml/day Ref: Ghai 8/e p146; 7/e p51
a. 1300 ml 33. Hyponatremia is defined as serum sodium:
b. 1700 ml a. <110 mEq/L (J & K PG 2011)
c. 2000 ml b. <120 mEq/L
d. 2500 ml Ref: Ghai 8/e p147 c. <130 mEq/L
29. Factors which decrease insensible water losses are all, d. <140 mEq/L Ref: Op Ghai 8/e p73
except: (DP PGMEE 2009) 34. A 3 years old child presents with four days history of
a. Humidified air puffiness of face, fever and tea colored urine. During the
b. Sedation course of his disease, he can have any of the complications
c. Hypothemia except: (UPSC 06)
d. Prematurity Ref: Ghai 8/e p147; Internet a. Hypokalemia
30. Hypercalciuria is said to be present in children if daily b. Hypertensive encephalopathy
calcium excretion in urine is more than: (Kerala PG 08) c. Acute renal failure
a. 1 mg/kg d. Acidosis Ref: Ghai 8/e p487; 7/e p446-447
b. 2 mg/kg 35. Baby born at 33 weeks / 1.5 kg should be started on?
c. 3 mg/kg a. Nil oral and IV fluids
d. 4 mg/kg Ref: Ghai 8/e p521; Nelson, 17/e p1824, b. Oral nasogastric tube/alternate oral route
Table 5391-1, 18/e p2184, Table 547-3 c. IV fluids and oral feeding
31. Total body water of newborn is what % of body weight? d. TPN
a. 60 % (Kerala PG 08) Ref: Nelson 18/e p706-707, O P Ghai 8/e p165; 7/e p132-33
b. 70 %
PEDIATRICS

Ans. 27. c. 90 28. b. 1700 ml 29. d. Prematurity 30. d. 4 mg/kg


31. c. 80 % 32. b. 1560-ml/day 33. c. <130 mEq/L 34. a. Hypokalemia
35. c. IV fluids and oral...
5. DISORDERS OF
GASTROINTESTINAL SYSTEM
30 Jaypees Triple A

DISORDERS OF GASTROINTESTINAL SYSTEM (QUESTIONS)

1. A newborn has dribbling after feeds. He has respiratory d. Hyalinization of the muscular coat
distress and froths at the mouth. Diagnosis is: (Al 01) Ref: Ghai 8/e p285; 7/e p256; Nelson 18/e p1565 and
a. Tracheoesophageal fistula 17/e p1239-1240; Nelson 19/e p1285
b. Tetralogy of fallot 7. Failure to pass meconium within 48 hours of birth in a
c. Respiratory distress syndrome newborn with no obvious external abnormality should lead
d. None of the above to the suspicion of: (AIIMS Nov 02)
Ref: Ghai 8/e p176; 7/e p151, Nelson 18/e p1543, 1544; a. Anal atresia
Nelson 19/e p1262-1263 b. Congenital pouch colon
2. A newborn baby had normal APGAR score at birth and c. Congenital aganglionosis
developed excessive frothing and choking on attempted d. Meconium ileus
feeds. The investigation of choice is: Ref: Ghai 8/e p285; 7/e p256; Nelson 18/e p1565-1567,
a. Esophagoscopy (AIIMS May 03) 2564 and 17/e p1239-1240; Nelson 19/e p1285
b. Bronchoscopy 8. Which one of the following is most suggestive of neonatal
c. MRI chest small bowel obstruction: (AI 03)
d. X-ray chest and abdomen with the red rubber catheter a. Generalized abdominal distension
passed per orally into esophagus b. Failure to pass meconium in the first 24 hours
Ref: Ghai 8/e p176; 7/e p151, Nelson 18/e p1543, 1544; Nelson 19/e p1263 c. Bilious vomiting
3. Most common biochemical abnormality in congenital d. Refusal of feeds
hypertrophic pyloric stenosis: (AIIMS Nov 09, 06, 02) Ref: Ghai 8/e p326; Nelson 19/e p1278
a. Hypokalemic metabolic alkalosis 9. An eight-year-old boy had abdominal pain, fever with
b. Hyperkalemic metabolic acidosis bloody diarrhea for 18 months. His height is 100 cms and
c. Hypokalemic metabolic acidosis weight is 14.5 kg. Stool culture was negative for known
d. Hyperkalemic metabolic alkalosis enteropathogens. The sigmoidoscopy was normal. During
Ref: Ghai 8/e p279; 7/e p253; Nelson 18/e p282-284, the same period, child had an episode of renal colic and
318, 1555-1557 and 17/e p1229-1230 Nelson 19/e p1275 passed urinary gravel. The mantoux test was 5 x 5 mm. The
4. Congenital hypertrophic pyloric stenosis usually presents: most probable diagnosis is: (AI 03)
a. Within 2 days after birth (AIIMS May 04) a. Ulcerative colitis
b. Around 1 week after birth b. Crohns disease
c. Around 2 weeks after birth c. Interstinal tuberculosis
d. Around 2 months after birth d. Strongyloidiasis
Ref: Ghai 8/e p279; 7/e p253; Nelson 18/e p318, 1555, 1556 and Ref: Ghai 8/e p304; Nelson 18/e p1580-1585 and 17/e p1248;
17/e p1229-1230; Nelson 19/e p1275 Nelson 19/e p1300
5. A 40-year-old male presents with recurrent bouts of 10. In which of the following conditions the lead pipe
vomiting since 9 months because of pyloric obstruction. appearance of the colon on a barium enema is seen?
The compensatory biochemical change is: (AIIMS May 01) a. Amoebiasis (AIIMS Nov 04)
a. Respiratory alkalosis b. Ulcerative colitis
b. Respiratory acidosis c. Tuberculosis of the colon
c. Paradoxical aciduria with hyponatremia and d. Crohns involvement of the colon
PEDIATRICS

hypochloremia Ref: Ghai 8/e p304; 7/e p277;


d. Metabolic acidosis Nelson 18/e p1580-1585 and 17/e p1248; Nelson 19/e p1301
Ref: Ghai 8/e p279; 7/e p253; Nelson 18/e p2301-2303, 11. Which of the followings includes inflammatory bowel
275-278 and 17/e p1229-1230 disease in children: (PGI June 05)
6. A male infant presented with distension of abdomen shortly a. Celiac disease
after birth with passing of less meconium. Subsequently a b. Tropical sprue
full-thickness biopsy of the rectum was performed. The c. Regional ileitis
rectal biopsy is likely to show: (AIIMS Nov 04, Nov 09) d. Cystic fibrosis
a. Fibrosis of submucosa e. Ulcerative colitis
b. Lack of ganglion cells Ref: Ghai 8/e p305; 7/e p277; Nelson 17/e p1248;
c. Thickened muscularis propria Nelson 19/e p1300

Ans. 1. a. Tracheoesophageal... 2. d. X-ray chest... 3. a. Hypokalemic... 4. c. Around 2 weeks...


5. c. Paradoxical aciduria... 6. b. Lack of ganglion... 7. c. Congenital... 8. c. Bilious vomiting
9. b. Crohns disease 10. d. Crohns involvement... 11. c and e
Disorders of Gastrointestinal System 31
12. A 12-yead-old girl has history of recurrent bulky stools a. Infection
and abdominal pain since 3 year of age. She has moderate b. Malnutrition
pallor and her weight and height are below the 3rd c. Diarrhea
percentile. Which of the following is the most appropriate d. Hyperglycemia
investigations to make a specific diagnosis? e. Metabolic acidosis
a. Small intestinal biopsy (AllMS Nov 04) Ref: Ghai 7/e p147-150, 6/e p666; Nelson 19/e p1376
b. Badan Sadies 19. Unconjugated hyperbilirubinemia is seen in:
c. 24 hrs fecal fat estimation a. Physiological jaundice (PGI Nov 07)
d. Urinary d-xylose test b. Breast milk jaundice
Ref: Ghai 8/e 301; Harrison 16/e p1770-1771; c. Gilbert syndrome
Nelson 19/e p1306 d. Biliary atresia
e. Rotor syndrome
13. In celiac disease all except: (PGI June 00)
Ref: Ghai 8/e p312, Harrison 16/e p1820;17/e p263; Nelson 19/e p1376
a. Gliadin is cause
20. Conjugated hyperbilirubinemia in infancy seen in:
b. Associated with HLA-B8
a. Choledochal cyst [PGI Dec. 04]
c. Decreased villi to crypt ratio
b. Extra hepatic biliary atresia
d. Increased brush border
c. Crigler-Najjar disease
Ref: Ghai 8/e p304, 301; 7/e p27;
d. Gilbert disease
Robbins 7/e p843; Nelson 19/e p1308 Ref: Ghai 8/e p315; 7/e p147150; Nelson 19/e p1376, 1377
14. The following cereals should be avoided in patients with 21. A neonate presents with jaundice and clay white stools. On
celiac diseases, except: (AIIMS Nov 03) liver biopsy giant cells are seen. Most likely diagnosis is:
a. Wheat (AI 01, AIIMS Nov 01)
b. Barely a. Physiological jaundice
c. Maize b. Neonatal hepatitis with extrabiliary atresia
d. Rye c. Neonatal hepatitis with physiological jaundice
Ref: Ghai 8/e p304, 301; Nelson 18/e p1624; d. Extrabiliary atresia
Robbins 7/e 843; Chandrasoma Taylor 3/e p591; Nelson 19/e p1310 Ref: Nelson 19/e p874; 18/e p1671-1672
15. The histological features of celiac disease include all of the 22. A neonate is being investigated for jaundice. A liver biopsy
foIlowing except: (AI02) shows features of a Giant Cell/ Neonatal hepatitis. Which
a. Crypt hyperplasia one of the following conditions usually result in this case?
b. Increase in thickness of the mucosa a. Congenital hepatic fibrosis (AIIMS Nov 04)
c. Increase in intraepithelial lymphocytes b. Hemochromatosis
d. Increase in inflammatory cells in lamina propyria c. Alpha-1-antitrypsin deficiency
Ref: Ghai 8/e p304, 301; Robbins 7/e p843; d. Glycogen storage disease Type 1
Ref: CPDT 18/e p644; Meharban Singh 6/e p257;
Nelson 18/e p1624; Nelson 19/e p1310
Nelson 19/e p1393
16. The most common genetic cause of liver disease in children
23. In neonatal cholestasis, if the serum gammaglutamyl
is:
transpeptidase is more than 600IU/L the most likely
a. Hemochromatosis (AI 02)
diagnosis is: (AIIMS Nov 02)
b. Antitrypsin deficiency a. Neonatal hepatitis (AI 04, AIIMS Nov 06)
c. Cystic fibrosis b. Choledochal cyst
d. Glycogen storage disease c. Hypothyroidism

PEDIATRICS
Ref: Ghai 8/e p289; Robbins 7/e p911; Nelson 18/e, d. Extrahepatic Biliary atresia
p1679 and 17/e p1323; Nelson 19/e p1393 Ref: Ghai 8/e p356; Surgery of Liver, bileduct and pancreas in children by
17. Unconjugated hyperbilirubinemia seen in: Edward Hoeard and Stringes 2/e, p112,
a. Dubin-Johnson syndrome (PGI Dec 08) Liver disorder in Childhood-Mowat, p83; Nelson 19/e p1385
b. Rotor syndrome 24. In a child presenting with obstructive jaundice all are seen
c. Gilberts syndrome except: (AIIMS Nov 06)
d. Breast feeding a. Gamma glutamyl transpeptidase
e. Physiological jaundice b. Alkaline phosphatase.
Ref: Ghai 8/e p312; 7/e p147-150; Nelson 19/e p1376 c. Glutamate dehydrogenase
18. A child with jaundice being given food intravenously. d. 5Nucleotidase
Which of the following can occur out of it: (PGI June 08) Ref: Ghai 8/e p324

Ans. 12. a. Small intestinal... 13. b. Associated with... 14. c. Maize 15. b. Increase in thickness...
16. b. Antitrypsin deficiency 17. c, d and e 18. a, d and e 19. a, b and c
20. a and b 21. b. Neonatal hepatitis... 22. c. Alpha-1-antitrypsin... 23. d. Extrahepatic Biliary...
24. c. Glutamate...
32 Jaypees Triple A
25. A 2-month-baby presents with history of jaundice, turmeric 31. In a child with active liver failure, the most important
colored urine and pale stools since birth. Examination prognosis factor for death is: (AIIMS May 06)
reveals liver span of l0 cm, firm in consistency and spleen a. Increasing transaminases
of 3 cm. The most specific investigation for establishing the b. Increasing bilirubin
diagnosis would be: (AI 03) c. Increasing prothrombin time
a. Liver function tests d. Gram -negative sepsis
b. Ultrasound abdomen Ref: Ghai 8/e p322; Nelson 18/e p1705;
c. Preoperative cholangiogram Nelson 19/e p1376
d. Liver biopsy 32. A 6-month-old infant presents to the diarrhea clinic unit
Ref: Nelsons 19/e p889; 18/e p1666; Meharban Singh 3/e p257 with some dehydration. The most likely organism causing
26. True about extrahepatic biliary atresia: diarrhea is: (AIIMS Nov 03)
a. Acholic stool (PGI June 03) a. Entamoeba histolytica
b. Unconjugated hyperbilirubinemia b. Rotavirus
c. Conjugated hyperbilirubinemia c. Giardia lamblia
d. Absence of nucleoid in duodenum in HIDA scan d. Shigella
e. Jaundice is presenting feature Ref: Ghai 8/e p297; 7/e p261;
Ref: Ghai 8/e p356; 7/e p293-294; Harrison 16/e p1815; Nelson 19/e p1246
Nelson 18/e p1671-1673 and 17/e p1317; 33. True about diarrhea: (PGI Dec 03)
Meharban Singh 6/e p257; Nelson 19/e p1385 a. Defined as passage of 2-3 formed stool/day
27. A 7-year-old girl from Bihar presented with three episodes b. Blood mixed with mucous stool is defined as dysentery
of massive hematemesis and melena. There is no history c. Rotavirus is the most common organism in children
of jaundice. On examination, she had a large spleen, d. Persistent diarrhea is defined if duration is more than 21
non-palpable liver and mild ascites. Portal vein was not days
visualized on ultrasonography. Liver function tests were Ref: Ghai 8/e p297; 7/e p261-263
normal and endoscopy revealed esophageal varices. The 34. Intractable diarrhea in children is caused by all except:
most likely diagnosis is: (AI 03) a. Cystic fibrosis (PGI June 00)
a. Kala azar with portal hypertension b. Giardiasis
b. Portal hypertension of unknown etiology c. Secreting tumors
c. Chronic liver disease with portal hypertension d. Milk allergy
d. Portal hypertension due to extrahepatic obstruction Ref: Ghai 8/e p297
Ref: Ghai 8/e p320; 7/e p289-290; Nelson 19/e p1415
35. Diarrhea in children is caused by: (PGI June 09)
28. Ramu, a 8 years old boy presents with upper GI bleeding. a. AIDS
On examination, he is found to have splenomegaly; there b. Nanvalle
are no signs of ascites, or hepatomegaly; esophageal varices c. Rotavirus
are found on UGId. Most likely diagnosis is: (AI 01) d. E . Coli Ref: Ghai 8/e p297-299
a. Budd chiary syndrome
36. A child is diagnosed to have anac gastroenteritis. The
b. Non cirrhotic portal fibrosis
consulting pediatricion wants to send a stool sample to a lab
c. Cirrhosis
which k 16-18 hrs. away. Which of the following medium he
d. Veno-occlusive disease
use to send this sample: (AIIMS Nov 2000)
Ref: Ghai 8/e p319, 320; Harrison 16/e p1862-1863
a. Charcol cotton bud
29. Portal hypertension in children in India is commonly due to: b. Carry Blair medium
a. Indian childhood cirrhosis [AIIMS Nov 03] c. Sterilized jar
PEDIATRICS

b. Extrahepatic portal venous obstruction d. A medium with high CO2 content


c. Idiopathic portal hypertension Ref: Micro CP Baweja 4/e p210;
d. Hepatic out flow tract obstruction Nelson 17/e p1274; 16/e p767
Ref: Ghai 8/e p320; Nelson 18/e p1709
37. One of the intestinal enzymes that is generally deficient in
30. A child is brought by mother with HO massive hematemesis children following an attack of severe infectious enteritis is:
with HO drug intake previously with NSAIDs and on Rx. a. Lactase (AI 05)
Associated with moderate splenomegaly diagnosis is: b. Trypsin
a. Esophageal varices (PGI Dec 06) c. Lipase
b. Duodenal ulcer d. Amylase
c. Drug induced gastritis Ref: Ghai 8/e p298; 7/e p269-270;
d. Peptic ulcer Nelson 18/e p1598, 1619 and 17/e p1268
Ref: Ghai 8/e p319, 320; 7/e p290; Nelson 19/e p1376

Ans. 25. c. Preoperative... 26. a, c, d and e 27. d. Portal hypertension... 28. b. Non cirrhotic...
29. b. Extrahepatic... 30. a. Esophageal varices 31. c. Increasing... 32. b. Rotavirus
33. b and c 34. b. Giardiasis 35. c and d 36. b. Carry Blair medium
37. a. Lactase
Disorders of Gastrointestinal System 33
38. A 6 months old baby with h/o bloody diarrhea of 2 days which one of the following digestive enzymes would be
duration with abdominal distension and on examination affected?
the baby screams, diagnosis is: (PGI Dec 03) a. Amylase (UP SC 07)
a. Intussusception b. Pepsin
b. HUS c. Lactose
c. Appendicitis d. Trypsin Ref: Guyton 10/e p346
d. Ac. Enterocolitis 46. Portal hypertension in children in India is commonly due to:
Ref: Ghai 8/e p287; Nelson 18/e, p1569-1570 and 17/e p1242; a. Indian childhood cirrhosis (AIIMS Nov 03)
Nelson 19/e p1288 b. Extrahepatic portal venous obstruction
39. Infant with blood in stools mass in abdomen, diagnosis is: c. Idiopathic portal hypertension
a. Intussusception (PGI June 01) d. Hepatic out flow tract obstruction 7 K
b. Volvulus Ref: Ghai 8/e p470; 7/e p319-320;
c. Idiopathic abdominal epilepsy Nelson 18/e p2082-2084 and 17/e p1670-1671
d. Hirschsprungs disease 47. To induce vomiting at home in a child who has ingested a
Ref: Ghai 8/e p287; Nelson 18/e, p1569-1570 and 17/e p1235; poison, the recommended agent of choice would be:
Nelson 19/e p1289 a. Oral rehydration solution (COMEDK 05)
40. Double bubble sign in children seen in all except: b. Mustard in warm water
a. Ladds band (PG1 Dec 04) c. Apomorphine
b. Annular pancreas d. Syrup of ipecac
c. Pancreatic pseudocyst Ref: Ghai 8/e p820; KD Tripathi 5/e p600
d. Diaphragmatic hernia 48. Baby born at 30 weeks for 18 years old primi gravida of
Ref: Ghai 8/e p179; Nelson 18/e p1559; Nelson 19/e p595 weight 2 kg which died after 48 hours. APGAR scores were
41. When severe dehydration in a neonate occurs, amount of 5 and 8 at 1 and 5 minutes. On autopsy bilateral enlarged
fluid replacement in 1st hour: kidney with multiple radially arranged cysts. Which of the
a. 20 40 mg/kg (CUPGEE 01) following finding is expected to be associated with?
b. 5 10 mg/kg a. Imperforate anus (AIIMS June 08)
c. 10 15 mg/kg b. Hepatic cyst and fibrosis
d. 15 20 mg/kg c. Absence of ureter
Ref: Ghai 8/e p73, 104; 7/e p264 d. Holoprosencephaly
42. 14-year-old girl with history of abdominal pain Ref: Ghai 8/e p316; Nelson 18/e p1706-1707; Nelson 19/e p1796, 1797
(periumbilical), postprandial, passing blood in stools, 49. Congenital Wilsons disease is characterized by:
fever, weight loss since ten months. She also has episodes of a. KF ring is present at birth (SGPGI 05)
passing blood in stools. What may be the likely diagnosis? b. May present as acute hepatitis
a. Chronic appendicitis (St. Johns 02) c. Decreased Urinary copper excretion
b. Chronic pancreatitis d. Decreased hapatic copper concentration
c. Crohns disease Ref: Ghai 7/e p641, 6/e p618; Nelson 17/e p1322
d. Bulimia 50. Childhood cholelithiasis is seen in:
Ref: Ghai 8/e p304; 7/e p277; Nelson 17/e p1252 a. Hurler Syndrome (AIIMS June 98)
43. Acquired megacolon in children most commonly due to: b. Mucopolysaccharidosis
a. Psychological problems (SGPGI 05) c. Neimann-Picks disease
b. Bad bowel habit d. Autoimmune hepatitis
c. Chagas disease Ref: Ghai 8/e p659-660; 7/e p639;

PEDIATRICS
d. Hirschusprungs disease Nelson 18/e p602 and 17/e p464; Nelson 19/e p488
Ref: Ghai 8/e p285; Bailey and Love 24/e p1156; Bailiy 51. True about Wilsons disease? (AI 2010)
44. X-ray detect congenital anorectal malformation at: a. Increase in urinary copper and increased serum
a. Immediately after birth (UP 07) ceruloplasmin and copper
b. 24-48 hours b. Increased serum ceruloplasmin levels with increased
c. 48-72 hours urinary copper
d. After 72 hours Ref: OP Ghai 6/e p178 c. Elevated hepatic copper level and increased serum
45. A young boy presents with failure to thrive. Biochemical ceruloplasmin levels
analysis of a duodenal aspirate after a meal reveals a d. Increase in urinary copper and decreased serum
deficiency of enteropeptidase (enterokinase)~ the levels of ceruloplasmin
Ref: Ghai 8/e p676; 7/e p292, 640; Harrisons 17/e p2451;
Nelson 19/e p1391

Ans. 38. a. Intussusception 39. a. Intussusception 40. d. Diaphragmatic 41. a. 20 40 mg/kg


42. c. Crohns disease 43. c. Chagas disease 44. b. 24-48 hours 45. d. Trypsin
46. b. Extrahepatic portal... 47. d. Syrup of ipecac 48. b. Hepatic cyst... 49. b. May present as...
50. c. Neimann-Picks... 51. d. Increase in urinary
34 Jaypees Triple A
52. A 12 year old girl with the mood and emotional liability c. Diabetes insipidus
has a golden brown discoloration in descement membrane. d. Mucopolysachridoses
Most likely diagnosis is: Ref: Ghai 8/e p610; Essentials of Peds, Nelson, 4/e p170,
a. Fabrys disease (AI 04) Campbell Tesxt Book Of Peds
b. Wilsons disease 60. A child si hepatitis B immunized the marker for
c. Glycogen storage disease seroconversior: (MP PG 2008)
d. Acute rheumatic fever a. HBs Ag
Ref: Ghai 8/e p676; 7/e p640; b. Anti HBc IgM
Nelson 17/e p1321-1322; Nelson 19/e p13913 c. Anti Hbs Ag
53. Congenital Wilsons disease is characterized by: d. Anti HBc IgC
a. KF ring is present at birth (SGPGI 05) Ref: Ghai 8/e p221; 7/e p193
b. May present as acute hepatitis 61. Most common bacteria causing diarrhea in children in India
c. Decreased Urinary copper excretion is: (Karnataka PG 2011)
d. Decreased hepatic copper concentration a. Enterotoxigenic e. coli
b. Enteroinvasive e. coli
Ref: OP Ghai 8/e p676; 7/e p632, 6/e p618
c. Enterohemorrhagie e. coil
54. Earliest symptoms of GERD in an infant is? d. Enteropathogenic e. coil
a. Respiratory distress Ref: OP Ghai 8/e p299; 7/e p261
b. Upper GI bleed 62. Unconjugated hyperbilirubinemia of newborn is caused by
c. Regulation all except: (Karnatka PG 2010)
d. Obstruction a. Criggler-Najjar syndrome
Ref: Ghai 8/e p279; Nelson 18/e p1547 b. Cretinism
55. Central dot sign seen in: c. Septicemia
a. Caroli disease d. Breast milk jaundice
b. Primary sclerosing cholangitis Ref: Ghai 8/e p718; Robbins Pathology, 7/e p887, Table 18-3; Nelsons
c. Liver hematoma Essentials of Pediatrics, 6/e p317; Oskis Pediatrics, 4/e p238
d. None of above 63. All of the following cause jaundice at birth or within 24
Ref: Internet hours except: (Karnataka PG 2010)
a. Physiological jaundice
56. One of the intestinal enzymes that is generally deficient in
b. Hemolytic disease of the newborn
children following an attack of severe infectious enteritis is: c. Criggler-Najjar syndrome
a. Lactase (DNB 2007) d. Intrauterine infections (TORCH)
b. Trypsin Ref: OP Ghai 8/e p172; 7/e p148
c. Lipase 64. All are true about physiological jaundice except:
d. Amylase a. 60% patients presents within 1st week (UP PG 09)
Ref: Ghai 8/e p512; Nelson, 16/e p318 b. Cord indirect bilirubin level is 1.3 mg/dL
57. Most common malignancy of GIT in childhood is: c. Resolved within one week
a. Carcinoid (DNB 2009) d. Persist beyond 10 days of jaundice
b. Lymphoma Ref: Ghai 8/e p172; 7/e p147-48
c. Adenocarcinoma 65. Budd Chiari syndrome as it is not associated with following
d. Sarcoma one: (AP 2011)
a. Leukemia
PEDIATRICS

Ref: 8/e p499; Nelson 18/e p1643


b. Sickle cell disease
58. Most common cause of gastroenteritis in infant is:
c. Polycythemia
a. Salmonella (DNB 2011)
d. Congestive heart failure
b. Rota virus Ref: Ghai 8/e p320
c. Norwalk virus
66. Newborn-frothing mouth unable to passing tube gasless
d. Coli
abdomen: (AP 2011)
Ref: Ghai 8/e p610 a. Oesophageal atresia with no Fistula
59. Hepatomegaly, Hypoglycemia and Ketosis in a 5 year old is b. Oesophageal atresia with Proximal Fistula
a feature of: (DNB 2011) c. Oesophageal atresia with distal Fistula
a. Fat storage disease d. Oesophageal atresia with double Fistula
b. Glycogen storage disease Ref: Ghai 8/e p176

Ans. 52. b. Wilsons disease 53. b. May present as... 54. a. Respiratory distress 55. a. Caroli disease
56. a. Lactase 57. b. lymphoma 58. b. Rota virus 59. b. Glycogen storage...
60. c. Anti Hbs Ag 61. a. Enterotoxigenic... 62. c. Septicemia 63. a. Physiological jaundice
64. d. Persist beyond... 65. d. Congestive heart... 66. a. Oesophageal atresia...
Disorders of Gastrointestinal System 35
67. In neonetes with duodenal atresia, all the following c. Head end elevated
statements are TRUE, except: (AP 2012) d. Thinning of infant feed formula
a. Increased incidence is seen in Downs syndrome Ref: Nelson, 17/e p1223; Bailey and Love, 24/e end
b. Incidence of other defects is reported 72. Which of the following is not a feature of infant colic
c. Prenatal diagnosis by ultrasound is not possible syndrome: (Kerala PG 09)
d. The operative treatment of choice is duodeno-duode- a. Vomiting
nostomy b. Paroxysmal Symptoms
Ref: Ghai 8/e p309 c. Abdominal pain
68. Regarding splenomegaly, one of the following statements d. Continuous severe cry
is false: (AP 2012) Ref: Ghai 8/e p278; Nelson 18/e p222
a. Spleen tip may be palpable in 15 percent of normal 73. Two month old baby presented with non bilous vomiting
neonates and 10 percent of normal children and a palpable epigastric lump. Which among the following
b. To become palpable below the costal margin on will be investigation of choice: (Kerala PG 10)
abdominal examination, it must be twice or thrice its a. USG abdomen
normal size b. X ray abdomen
c. Splenomegaly is a characteristics finding in aplastic c. Upper GI Series
anemia and idiopathic thrombocytopenic purpura d. CT abdomen
d. Childhood osteopetrosis and prehepatic portal Ref: Ghai 8/e p279; Nelson 18/e p1555
hypertension have significantly large spleen on
74. Hutchinson teeth are seen in: (UP PG 2010)
examination Ref: Ghai 8/e p310
a. Toxoplasma
69. Which one of the following hepatitis viruses have b. Congenital syphilis
significant prenatal transmission? (Feb DP PGMEE 2009) c. Rubella
a. Hepatitis E virus d. CMV-infection Ref: Nelson 18/e p1265
b. Hepatitis C virus
75. Persistent diarrhea is defined as diarrhea that begins as an
c. Hepatitis B virus
acute episode and lasts for at least: (J & K PG 2011)
d. Hepatitis A virus
a. 7 days
Ref: Ghai 8/e p183
b. 10 days
70. A Child is 18 month old presented with vomiting without c. 14 days
bile the diagnosis is: (Raj PG 2008) d. 21 days Ref: OP Ghai 8/e p297
a. Pyloric stenosis
76. The causes of chronic diarrhea in children may be because
b. Duodenal atresia
of etiological factors given below except: (J & K PG 2010)
c. Congenital megacolon
a. Giardiasis
d. All of the above Ref: Ghai 8/e p279
b. Rotavirus
71. All are true of GERD except: (Kerala PG 08) c. Lactose intolerance
a. Antacid d. Celiac diseases
b. Prone end elevated Ref: OP Ghai 8/e p198

PEDIATRICS

Ans. 67. c. Prenatal diagnosis... 68. c. Splenomegaly... 69. c. Hepatitis B virus 70. a. Pyloric stenosis
71. b and d 72. c. Abdominal pain 73. a. USG abdomen 74. b. Congenital syphilis
75. d. 21 days 76. b. Rotavirus
6. IMMUNIZATION, INFECTIOUS
DISEASES AND WORM INFESTATION
Immunization, Infectious Diseases and Worm Infestation 37

IMMUNIZATION, INFECTIOUS DISEASES... (QUESTIONS)

1. Which of the following is not a common manifestations of 8. True about measles: (PGI Dec 05)
congenital Rubella: (AI 02) a. Rash appears first on leg
a. Deafness b. Koplik spots are seen in retina
b. PDA c. Long-term complication follows in the form of SSPE
c. Aortic stenosis d. Caused by RNA virus
d. Mental retardation e. IP is 2 - 3 days
Ref: Ghai 8/e p213; Nelson 18/e p1340 and 17/e Ref: Ghai 8/e p213-215; 7/e p185; Harrison 16/e p485; Nelson 19/e p1072
p569, 1033; Nelson 19/e p1077 9. SSPE (subacute sclerosing panencephalitis) is associated
2. All are true about congenital rubella except: with: (PGI June 01)
a. IgG persists for more than 6 months a. Mumps
b. IgM antibody is present at birth (AIIMS Nov 08) b. Chicken pox
c. Most common anomalies are hearing and heart defects c. Herpes
d. Increased congenital malformation if infection after 16 d. Measles
weeks Ref: Ghai 8/e p214; 7/e p185; Harrison 16/e p485; Nelson 19/e p1072
Ref: Ghai 8/e p213; Nelson 18/e p1340; Nelson 19/e p1076-1077 10. With reference to mumps which of the following is true?
3. Congenital rubella manifestations are all except: a. Meningoencephalitis can precede parotitis (AI 06)
a. Rash appears first on trunk (PGI June 00) b. Salivary gland involvement is limited to the parotids
b. Pre-auricular lymph nodes c. The patient is not infectious prior to clinical parotid
c. Arthralgia enlargement
d. Retinopathy d. Mumps orchitis frequency leads to infertility
Ref: Ghai 8/e p213; Nelson 18/e p1337-1371 and 17/e Ref: Ghai 8/e p213; 7/e p187-188; Nelson 19/e p1080
p10321034; Nelson 19/e p1076 11. Which of the following is true of mumps? (AIIMS May 05)
4. Symptomatic neonatal CNS involvement is most commonly a. Salivary gland involvement is limited to the parotids
seen in which group of congenital intrauterine infections: b. The patient is not infectious prior to clinical parotid
a. CMV and Toxoplasmosis (AI 09) enlargement
b. Rubella and Toxoplasmosis c. Menigoencephalitis can precede parotitis
c. Rubella and HSV d. Mumps orchitis frequently leads to infertility
d. CMV and Syphillis Ref: Ghai 8/e p213; 6/e p209-210; Nelson 18/e p395-405;
Ref: Ghai, 8/e p214; Nelson 18/e p1340; Nelson 19/e p1077 Nelson 19/e p1080
5. Congenital Rubella Syndrome in associated with anomalies? 12. An 8-years-old female child following URTI developed
a. VSD (PGI Nov 09) maculopapular rash on the jaw spreading onto the trunk
b. ASD which cleared on the 3rd day without desquamation and
c. PDA tender post auricular and suboccipital lymphadenopathy.
d. Epstenis The diagnosis is:
e. Myocarditis a. Kawasaki disease (AIIMS May 01)
Ref: Ghai 8/e p213; Robbins Pathology 7/e p473 Nelson 19/e p1077 b. Erythema infectiosum
6. True about chicken pox: (PGI June 01) c. Rubella
d. Measles

PEDIATRICS
a. I.P. 2-3 days
b. Scabs are infective Ref: Ghai 8/e p213, 214; Nelson 18/e p1337-1341 and 17/e p1032;
c. Centrifugal rash Nelson 19/e p1076
d. Rash appears on first day 13. A patient had fever and coryza for last 3 days developed
e. Rash can occur in axilla maculopapular erythematous rash with which lasted
Ref: Ghai 8/e p215; Nelsons 18/e p1767-1369; Nelson 19/e p1105 for 48 hours and disappeared without leaving behind
7. M/C complication of chicken pox in children: pigmentation is most commonly due to: (AIIMS June 00)
a. Encephalitis (PGI June 00) a. Measles
b. Secondary bacterial infection b. Typhoid
c. Pneumonia c. Roseola infantum
d. Otitis media d. Fifth disease
Ref: Ghai 8/e p215; 7/e p186; Harrison 16/e p1043; Nelson 19/e p1109 Ref: Ghai 8/e p213, 207, 229; Nelson 19/e p1118

Ans. 1. c. Aortic stenosis 2. d. Increased congenital... 3. a, b and c 4. c. Rubella and HSV


5. a, b and c 6. d and e 7. b. Secondary... 8. c and d
9. d. Measles 10. a. Meningoencephalitis... 11. c. Menigoencephalitis... 12. c. Rubella
13. c. Roseola infantum
38 Jaypees Triple A
14. Child (girl) is suffering from varicella (fever rash). And Ref: Ghai 8/e p250; 7/e p166; Nelson 17/e p971-972;
childs aunt is pregnant. When is it earliest that the child Nelson 19/e p1007, 1011
can meet her aunt: (PGI Dec 06) 21. Drugs included in ATT for children:
a. When the lesions have crusted a. Streptomycin (PGI June 06)
b. Immediately b. Ethionamide
c. Anytime as the child is aunts favourite c. Ethambutol
d. After the delivery of the baby d. Pyrazinamide
Ref: Ghai 8/e p215; 7/e p186; Nelson 19/e p1105 e. Ofloxacin
15. Which of the following is true about Roseola infantum: Ref: Ghai 8/e p250, 251; 7/e p210-212]
a. Defervescence follows the rash (PGI Dec 05) 22. First line ATT in children: (PGI June 05)
b. Caused by HHV 6 and 7 a. Streptomycin
c. Slapped cheek appearance is seen b. Pyrazinamide
d. Otitis media is common complication c. Ethionamide
e. Rash appears first on face and neck d. Ethambutol
Ref: Ghai 8/e p213, 207, 229; Nelson 19/e p1117 e. Ofloxacin Ref: Ghai 8/e p250; 7/e p217
16. Which of the following is true about erythema infectiosum: 23. 20 years old Rajesh presence with fever and chronic
a. Slapped cheek appearance seen (PGI Dec 05) cough ESR 35 mm sputum cytology is negative for AFB
b. Caused by parvovirus and tuberculin test show indurations of 19 X 23 mm. The
c. Defervescence before rash probable diagnosis is: (AIIMS Nov 01)
d. Rash appears on head and neck a. Fungal pneumonia
Ref: Ghai 8/e p213, 214; Nelson 19/e p1117 b. Viral pneumonia
17. True about Roseola infantum: (PGM June 03) c. Pulmonary TB
a. Also called 5th disease d. Bacterial pneumonia
b. Causedby HHV 6 and 7 Ref: Ghai 8/e p250; CMDT 2002/e p310, API 6/e p244
c. Rash appear in trunk 24. Pertusis affects which age: (PGI June 2000)
d. During defeverescence, rash appears a. 2-3
Ref: Ghai 8/e p213; See previous questions; Nelson 19/e p1117 b. <5
18. A 3-month-old male infant developed otitis media for c. 5-7
which he was given a course of Cotrimoxazole. A few days d. > 10 years
later, he developed extensive peeling of the skin; there were Ref: Ghai 8/e p240; 7/e, 221; Nelson 19/e p945
no mucosal lesions and the baby was not toxic. The most 25. All of the following strategies are effective in preventing
likely diagnosis is: mother to child transmission of HIV, except:
a. Toxic epidermal necrolysis (AIIMS May 04) a. Zidovudine to mother and baby (AIIMS Nov 03)
b. Staphylococcal scalded skin syndrome b. Vaginal cleansing before delivery
c. Steven Johnson syndrome c. Stopping breast feeding
d. Infantile pemphigus d. Elective cesarean section
Ref: Pasricha 4/e p162, 163, Ghai 8/e p687; Nelson 19/e p2302 Ref: Ghai 8/e p229; 7/e p204-207; Harrison 16/e p1082;
19. A 10-month-old child presents with two weeks history of Nelson 19/e p1159
fever, vomiting and alteration of sensorium cranial CT scan 26. All of the following methods are used for the diagnosis of
reveals basal exudates and hydrocephalus, the most likely HIV infection in a 2-months old child, except:
etiological agent is: (AI 04) a. DNA-PCR (AIIMS May 03)
a. Mycobacterium tuberculosis b. Viral culture
PEDIATRICS

b. Cryptococcus neoformans c. HIV ELISA


c. Listeria monocytogenes d. p24 antigen assay
d. Streptococcus pneumonia Ref: Ghai 8/e p229; Nelson 18/e p1492, 859,
Ref: Ghai 8/e p250; 7/e p539; 1435-1436 and 17/e p1116; Nelson 19/e p1166
Nelson 18/e p1240-1254 and 17/e p966; Nelson 19/e p1005
27. Which drug is given to prevent HIV transmission from
20. Management of a newborn when mother has active mother to child: (AIIMS Nov 06)
tuberculosis and is taking ATT: (AI 00) a. Nevirapine
a. BCG + Rifampicin + INH + Breast Feeding b. Lamivudnie
b. BCG +Isolation of baby c. Stavudine
c. BCG +INH for 6 week + Breast Feeding d. Abacavir
d. BCG + INH +withhold Breast Feeding Ref: Ghai 8/e p230, Nelson 17/e, p1120

Ans. 14. a. When the lesions... 15. b. Caused by HHV... 16. a and b 17. b, c and d
18. b. Staphylococcal... 19. a. Mycobacterium... 20. c. BCG +INH... 21. All of these
22. a, b and d 23. c. Pulmonary TB 24. b. <5 25. b. Vaginal cleansing...
26. c. HIV ELISA 27. a. Nevirapine
Immunization, Infectious Diseases and Worm Infestation 39
28. Which one of the following hepatitis viruses have c. IM injections and increased muscular activity increases
significant perinatal transmission: (A/ 03) the risk of paralytic polio
a. Hepatitis E virus d. Pulse polio immunization is indicated in less than 3 years
b. Hepatitis C virus of age
c. Hepatitis B virus Ref: OP Ghai 8/e p215; 7/e p190;
d. Hepatitis A virus SPM, park 19/e, p 168; Nelson 19/e p1087
Ref: Ghai 8/e p222; Nelson 18/e p1680; 34. Kenny Packs were used in the treatment of:
Nelson 19/e p1396, 1397 a. Poliomyelitis (AIIMS 03)
29. A 45 day-old infant developed icterus and two days later b. Muscular dystrophy
symptoms and signs of acute liver failure appeared. Child c. Polyneuropathies
was found to be positive for HBsAg. The mother was also d. Nerve injury
HBsAg carrier. The mothers hepatitis B serological profile Ref: Ghai 8/e p215; 7/e p190
is likely to be: (AI 03) 35. Acute flaccid paralysis is reported in a child aged:
a. HBsAg positive only a. 0-3 years (AI 02)
b. HBsAg and HBeAg positivity b. 0-5 years
c. HBsAg and anti-HBe antibody positivity c. 0-15 years
d. Mother infected with mutant HBV d. 0-25 years
Ref: Ghai 8/e p223; 7/e p193-194; Harrison 16/e p1830; Ref: Ghai 8/e p592; 7/e p190
Nelson 18/e p1665, 1699, 1700 and 17/e p1327; Nelson 19/e p1398
36. An 8-year-old boy presented with fever and bilateral
30. A 2-month-old baby with acute viral hepatitis like illness cervical lymphadenopathy with prior history of sore throat.
slips into encephalopathy after 48 hours. The mother is There was no hepatomegaly. The peripheral blood smear
a known hepatitis B carrier. Mothers hepatitis B virus shows > 20% lympho-plasmacyteoid cells. The most likely
serological profile is most likely to be: (AI 03) diagnosis is: (AI 02)
a. HBsAg positive only a. Influenza
b. HBsAg and HBeAg positive b. Tuberculosis
c. HBsAg and HBe antibody positive c. Infectious mononucleosis
d. HBV DNA positive d. Acute lymphoblastic leukemia
Ref: Ghai 8/e p222; 7/e p193-194; Harrison 16/e p1830; Ref: Ghai 8/e p215; 7/e p187; Harrison 16/e p1045-1048;
Nelson 18/e p1665-1700 and 17/e p1327; Nelson 19/e p1111
31. A 5-year-old boy is detected to be HBs Ag positive on two 37. Which of the following does not establish a diagnosis of
separate occasions during a screening program for hepatitis congenital CMV infection in a neonate? (AIIMS May 06)
b. He is otherwise asymptomatic: a. Urine culture of CMV
a. Obtain HBeAg and anti-HBe levels [AI 03] b. IgG CMV antibodies in blood
b. Obtain anti HBs levels c. Intranuclear inclusion bodies in hepatocytes
c. Repeat HBsAg d. CMV viral DNA in blood by polymerase chain reaction
d. Repeat another course of Hepatitis B vaccine Ref: Ghai 8/e p272; Nelson 18/e p1378 and 17/e p1068;
Ref: Ghai 8/e p221; Nelson 18/e p1685; 17/e p1328; Harrison 16/e p1051; Nelson 19/e p1111
Nelson 19/e p1399
38. Dengue shock syndrome is characterized by the following
32. A 30 years old lady delivered a healthy baby at 37 weeks except: (AIIMS May 05)
of gestation. She was a known case of chronic hepatitis B a. Hepatomegaly
infection. She was positive for HBsAG but negative for b. Pleural effusion
HBeAG. Which of the following is the most appropriate c. Thrombocytopenia

PEDIATRICS
treatment for the baby: (AIIMS Nov 05) d. Decreased haemoglobin
a. Both active and passive immunization soon after birth Ref: Ghai 8/e p200; 7/e p196; Nelson 18/e p1412-1414 and
b. Passive immunization soon after birth and active 17/e p1092-1093; Harrison 16/e p1173; Nelson 19/e p1148
immunization at 1 year of age
39. After 5 days of birth, baby developed poor feeding,
c. Only passive immunization soon after birth
convulsions, fever with low protein, low sugar and high
d. Only active immunization soon after birth
chloride (in CSF) is most likely due to:
Ref: Ghai 8/e p222; Nelson 18/e p1685 and 17/e
a. Listeria monocytogenes (AIIMS June 00)
p1328-1329; Nelson 19/e p1399
b. Mycoplasma pneumoniae
33. Regarding polio which is true: (AIIMS Nov 07) c. TB
a. Most of the cases are symptomatic (May 08) d. Leptospira
b. Spastic paralysis is seen Ref: Ghai 8/e p262; Nelson 18/e p1157-1159 and 17/e p2039;
Nelson 19/e p929

Ans. 28. c. Hepatitis B virus 29. b. HBsAg and HBeAg... 30. b. HBsAg and HBeAg... 31. b. Obtain anti HBs...
32. a. Both active... 33. c. IM injections... 34. a. Poliomyelitis 35. c. 0-15 years
36. c. Infectious... 37. b. IgG CMV antibodies... 38. d. Decreased haemoglobin 39. a. Listeria monocytogenes
40 Jaypees Triple A
40. True about tetanus: (PGI June 01) 47. A child presented with fever for 2 days, altered sensorium
a. Tetanus bacilli can spread through blood and purpuric rashes. His blood pressure is 90/60 mmHg.
b. Spread along the nerves only Treatment of choice is:
c. Requires oxygen a. IV Quinine (AIIMS May 02)
d. IP is variable b. IV Artesunate
e. Causes deep wound invasion c. IV Penicillin
Ref: Harrison Ghai 8/e p243; 16/e p840841; Nelson 19/e p992 d. Chloroquine
41. A child with fever and sore throat developed acute cervical Ref: Ghai 8/e p223; 7/e p538-539;
lymphadenopathy most likely investigation to be done is: Nelson 18/e p1166, 2514-2519 and 17/e p2024; Nelson 19/e p932
a. Open biopsy of node (AIIMS June 00) 48. Vaccine with best efficacy: (AI 09)
b. Radical neck dissection a. TT
c. Neck X-ray b. DPT
d. Complete hemogram c. Measles
Ref: Ghai 8/e p241; Nelson 18/e p917, 2093-2095, d. Typhoid Ref: Ghai 8/e p215; 7/e p171; Nelson 19/e p881
2106 and 17/e p1062-1066, 1678; Nelson 19/e p992, 993 49. A 4-year-old child presented in OPD with vaccination
42. In a 6 months old baby, floppy infant syndrome is seen history of receiving one dose of OPV and DPT at 2 months
commonly due to infection with: of age. Correct statements are: (PGI Dec 04)
a. Clostridium welchii (AIIMS June 00) a. BCG should not be given
b. Clostridium tetani b. DPT and OPV should be repeated.
c. Clostridium Botulinum c. Measles should be given
d. Clostridium septicum d. Hepatitis B vaccine advised
Ref: Ghai 8/e p243; Nelson 18/e p1224-1227 and 17/e p947948; e. Hemophilus vaccine not given
Nelson 19/e p989 Ref: Ghai 8/e p215; Nelson 18/e p1013, 1062 and 17/e p1178
43. Integrated management of childhood illness includes 50. Which vaccine is contraindicated in child with history of
except: (Al 09) convulsions? (AIIMS June 08)
a. ARI a. DPT
b. Malaria b. Measles
c. Diarrhea c. Typhoid
d. TB d. BCG
Ref: Ghai 8/e p751-756; 7/e p735; Nelson, 18th/e, p1613; Ref: Ghai 8/e p205; 7/e p167-169;
Nelson 19/e p1334 IAP guide book on immunization, 3/e p14
44. Resistant plasmodium falciparum malaria in the pediatric 51. 18 months old child, who has received one dose of DPT
age group should be treated by: and OPV at 2 months of age. What will be your next
a. Chloroquine (AI 08) immunization plan? (AIIM Nov 07)
b. Tetracycline a. Restart immunization schedule, as per age
c. Clindamycin b. Measles, BCG, booster dose of DPT and OPV
d. Doxycycline c. Measles, booster dose of DPT and OPV
Ref: Ghai 8/e p260; Nelsons 18/e p1421; Nelson 19/e p1203 d. BCG, 2nd dose of DPT and OPV
45. A child presents with generalized petechiae. CSF shows Ref: Ghai 8/e p205; IAP guide book on immunization, 3/e p41,
gram negative diplococci. Treatment of choice: S.K. Mittal and Kukreja book of immunization, 2/e, p5
a. IV ceftriaxone (PGI Nov 07) 52. A premature baby of 34 weeks was delivered. The baby had
b. IV cefotaxime a bullous lesion on the body and X-ray showed periostitis
PEDIATRICS

c. IV penicillin G what will be the next diagnostic procedure: (AI 07)


d. IV ampicillin a. VDRL of mother and baby
e. IV ofloxacin b. ELISA for HIV
Ref: Ghai 8/e p243; 7/e p539; Harrison 16/e, 2476, 17/e, 913 c. PCR for Herpes
46. All are true about septic shock in children except? d. HBsAg for mother
a. 1st response is CO. due to vasodilaiaiion Ref: Ghai 8/e p272; Nelsons 18/e p1264-1265
b. Hypotension is a late sign (PGI Nov 09) 53. Congenital syphilis can be best diagnosed by:
c. Heart rate remains same a. IgM FTA- ABS
d. Periph vascular resistance to compensate b. IgG FTA-ABS [AI 01]
e. Endotheliurn release vasodl factors c. VDRL
Ref: Ghai 8/e p715; Robbins 7/e p139 d. TPI
Ref: Ghai 8/e p272; Harrison 16/e p982

Ans. 40. b and d 41. d. Complete hemogram 42. c. Clostridium Botulinum 43. d. TB
44. c. Clindamycin 45. c. IV penicillin G 46. c and d 47. c. IV Penicillin
48. c. Measles 49. b, c and d 50. a. DPT 51. d. BCG, 2nd dose
52. a. VDRL of mother... 53. a. IgM FTA- ABS
Immunization, Infectious Diseases and Worm Infestation 41
54. Symptomatic neonatal CNS involvement is most commonly c. R3 type
seen in which group of congenital intrauterine infection: d. R4 type Ref: Ghai 8/e p260; 7/e p227
a. CMV and toxoplasmosis (AI 08) 63. Which one of the following antibacterial antibiotics are not
b. Rubella and toxoplasmosis recommend for lactating mothers:
c. Rubella and HSV a. Cephaslosporins (COMED 06)
d. CMV and syphilis b. Anti tubercular drugs
Ref: Ghai 8/e p274; Neurology Valpe 3/e, p63 c. Quinolones
55. Cysticercosis present as: (PGI Dec 03) d. Aminoglycoside
a. Seizures Ref: KD Tripathi 5/e p534; Ghai 7/e p724, 6/e p681
b. Neuropathy 64. HIV associated nephropathy is severe form of:
c. Encephalitis a. Focal segmental glomerulosclerosis
d. Muscular hypertrophy Ref: Ghai 8/e p570 b. Membranous nephropathy (JIPMER 04)
56. Which vaccines are not given in a 8 yrs old unimmunized c. Membrano-proliferative glomerulonephritis
child: (PGI Dec 06) d. Focal segmental glomerulonephritis
a. Pertusis Ref: Ghai 8/e p229; Robbin 7/e p984
b. Salk vaccine 65. Which one of the following is a conjugated vaccine:
c. Measles a. Hepatitis B (UPSC 04)
d. BCG b. Rubella
e. DT Ref: Ghai 8/e p205; See above explanation c. Hemophilius influenza
57. The following age group is most severely affected by d. Pertussis Ref: Ghai 8/e p190
Rubella infection: (JIPMER 02) 66. 12.6 months old baby was brought with c/o difficulty in
a. Females aged 25-35 years feeding. The child was found to be hypnotic with a week
b. Young girls gag. The child is on breast milk and mother also gives
c. Adolescent girls honey to the child during periods of excessive crying. The
d. Unborn child Ref: Ghai 8/e p215; 7/e p1 causative agent is:
58. Roseola infantum can be caused by: (Kerala 04) a. Gram-positive aerobic occurs (J & K 05)
a. Herpes virus 6 b. Gram positive anaerobic spre-suffering bacillus
b. Parvovirus B 19 c. Toxin produced by gram-positive anaerobic bacillus
c. Echovirus 19 d. Echovirus
d. All of the above Ref: Ghai 8/e p213; 7/e p186 67. A 32 weeks premature infant, 900 gm weight on the third
59. The etiological agent for roseola infantum is: days. The serum bilirubin is 13 mg%. The treatment of
a. Parvovirus (JIPMER 02) choice is: (UP 07)
b. Human herpes virus 6 a. Exchange transfusion
c. CMV b. Phototherapy
d. EBV Ref: Ghai 8/e p213, 214; 7/e p186 c. Wait and watch therapy
60. Which positive test done not necessarily indicate d. Pharmacologic therapy
clostridium botulinum HIV infection in a newborn? Ref: OP Ghai 8/e p175; 7/e p191-195
a. ELISA for HIV 1gG antibody (Karnataka 04) 68. Differential diagnosis in case of fever with vesicular rash
b. p24 antigen for two days are all except:
c. Virus culture a. Candida albicans (UP 07)
d. ELISA for HIV IgM antibody Ref: Ghai 8/e p229; 7/e p203 b. Infectious mononucleosis

PEDIATRICS
61. 10 years old child with 10 days continuos fever with soft, c. Klebsiella pneumonia
enlarged spleen, diagnosis is: d. Influenza Ref: Ghai 8/e p216; Harrisons 16/e, p110-114
a. Enteric fever (JIPMER 02) 69. For the prevention of parent to child transmission of HIV,
b. Malarial the NACOs recommendation is to give: (UPSC 07)
c. Hodgkins disease a. Niverapine 200 mg in active labor to mother
d. Meningitis Ref: Ghai 8/e p240; 7/e p222 b. Niverapine 200 mg, four hours after rupture of
62. Patient was given chloroquine and doxycycline for 7 days. membranes, to mother
Patients fever decreases in 4 days, but, peripheral smear c. Niverapine 200 mg in active labor to mother and syrup
showed occasional gametocytes of plasmodium falciparum. niverapine 2 mglkg body weight to newborn within 72
This type of drug resistance is: (CMC 05) hours of delivery
a. R type d. Syrup niverapine 2 mg/kg body weight to newborn
b. R2type within 72 hours of birth Ref: OP Ghai 8/e p229; 7/e p207

Ans. 54. c. Rubella and HSV 55. a and c 56. a, b and d 57. d. Unborn child
58. a. Herpes virus 6 59. b. Human herpes... 60. a. ELISA for HIV... 61. a. Enteric fever
62. b. R2type 63. c. Quinolones 64. a. Focal segmental... 65. c. Hemophilius influenza
66. c. Toxin produced... 67. a. Exchange transfusion 68. b. Infectious mononucleosis 69. c. Niverapine 200 mg...
42 Jaypees Triple A
70. 2-years-old child with head circumference of 55 cm, is likely c. 2 nd trimester
to have had intrauterine infection due to: (Corned 07) d. Third trimester
a. Rubella Ref: Ghai 8/e p210; Esssentials of Pediatrics, Nelson 4/e p242, 465, 466
b. Toxoplasmosis 79. Not a cause of pneumonia is: (DNB 2011)
c. Herpes a. Measles
d. Hepatitis B Ref: Ghai 8/e p272; Nelson 17/e p1989 b. Mumps
71. Infant with cystic fibrosis (CF) are likely to develop: c. RSV
a. Meconium ileus (MAHE 07) d. Influenza Ref: Ghai 8/e p376; Nelson Ped, 18/e p2366
b. Loose motions 80. What is the minium interval required between the
c. Vomiting administration of two live vaccines? (MP PG 2010)
d. Constipation Ref: OP Ghai 8/e p393; 7/e p369 a. 8 Weeks
72. Pneumatoceles often develop in children after pneumonia b. 6 Weeks
due to the following organism: c. 4 Week
a. Klebsiella (Corned 07) d. 2 Weels Ref: Ghai 8/e p203; Nelsons 18/e p1078
b. Streptococcus 81. Oral typhoid vaccine is not recommended for children in
c. Staphylococcus aureus the age group of: (MP PG 2009)
d. Hemophilus influenzae Ref: OP Ghai 8/e p374; 7/e p352 a. < 2 years
73. An infant develops cough and fever. The X-ray examination b. < 4 years
is suggestive of bronchopneumonia. All of the following c. > 6 years of age
viruses can be the causative agent except: (AI 04) d. < 8 years
a. Parainfluenza viruses Ref: Ghai 8/e p197; Nelsons 18/e p174
b. Influenza virus A 82. The most effective and preferred scabicidal drug for
c. Respiratory syncytial virus treatment of scabies in children is: (MP PG 2008)
d. Mumps virus a. Permethrin
Ref: Ghai 8/e p376; 7/e p187-188; Nelson 18/e p797-798, 394, b. Cromatilion
1795-1799 and 17/e p1433; Nelson 19/e p1475 c. Hexachlorobenzene
74. 2-month-old girl present with verrucous plague on the d. Ivermectin
trunk: What is your most probable diagnosis? Ref: KDTs Pharma 6/e p865; Nelsons 18/e p2757
a. Incontinentia pigmentosa (AIIMS Nov 08) 83. Black water fever is a complication associated with:
b. Darier disease a. Filaria (MP PG 2008)
c. Cogenital nevus b. Leptospirosis
d. Ichthyosis c. Kalazar
Ref: Ghai 8/e p675; Nelson 18/e p2681; Nelson 19/e p2052, 2053 d. Malaria
75. Which is true about BCG? Ref: Ghai 8/e p260; Nelsons 18/e p1481; 17/e p1142
a. Distilled water is used as diluents 84. Typhoid Vi polysaccharide vaccine is usually administered
b. Site for injection is cleaned with spirit in children above the age of: (Karnataka PG 2011)
c. Mantoux test positive in 6 weeks a. 6 months
d. WHO recommends Danish 1331 for vaccine production b. 1 year
Ref: 171 Park 21/e, p176 c. 2 years
76. Live attenuated bacterial vaccine is: (DNB 2009) d. 1 year 6 months
a. Tetanus Ref: OP Ghai 8/e p190-203; 7/e p174
b. Hepatitis B 85. Not immunized previously in a 8 years male child which of
PEDIATRICS

c. Sabin the following vaccine is not given to that children:


d. BCG Ref: OP Ghai 8/e p310 a. Inactive polio vaccine (UP PG 09)
77. Congenital Rubella causes all the following features except: b. Hepatitis B vaccine
a. Cataract (DNB 2011) c. Acellular pertusis vaccine
b. Deafness d. Measles vaccine Ref: Ghai 8/e p203
c. Cardiac defects 86. DPT is contraindicated in: (UP PG 09)
d. Hydrocephalus a. Family history of convulsion
Ref: Ghai 8/e p210; Esssentials of Pediatrics, Nelson 4/e p242, 243 b. Acute respiratory tract infection
78. Vertical transmission of Toxoplasmosis is more severe in: c. Progressive neurological illness
a. During labour (DNB 2011) d. Mild diarrhea
b. Ist trimester Ref: Ghai 8/e p192; 7/e p168-69

Ans. 70. b. Toxoplasmosis 71. a. Meconium ileus 72. a and c 73. d. Mumps virus
74. a. Incontinentia... 75. d. WHO recommends... 76. d. BCG 77. d. Hydrocephalus
78. b. Ist trimester 79. b. Mumps 80. c. 4 Week 81. c. > 6 years of age
82. a. Permethrin 83. d. Malaria 84. c. 2 years 85. c. Acellular pertusis...
86. c. Progressive...
Immunization, Infectious Diseases and Worm Infestation 43
87. Treatment of c. difficle infection: (MHPGM-CET 2010) 95. Which one of the following is NOT a features of congenital
a. Metronidazole rubella syndrome (AP 2012)
b. Bacitracin a. Deafness
c. Nitazoxanilide b. Microphthalmia
d. All of the above c. Hepatitis
Ref: Ghai 8/e p299; Harrisons 17/e 820; d. Cataracts Ref: Ghai 8/e p272
KDT 6/e p731; Table 123-2 96. All are true regarding typhoid vaccines except:
88. Incubation period of mumps? (MHPGM-CET 2007, 2010) a. Heat killed whole cell vaccine causes significant local
a. 5-7 days pain and systemic malaise (DP PGMEE 2009)
b. 8-15 days b. Vi polysaccharide is non-immunogenic in children less
c. 14-24 days than two years
d. 24-30 days c. Ty21 a vaccine can be given intramuscularly and
Ref: Ghai 8/e p217; Nelson paediatrics 18/e p1341 subcutaneously
89. Single-dose mebendazole is effective in treatment of (repeat d. Vi polysaccharide vaccine is to be repeated every 3 years
dose 14 days later): (MHPGM-CET 2008, 2010) Ref: Ghai 8/e p197
a. Enterobius vermicularis 97. The 23 valent pneumococcal vaccine is recommended in all
b. Ankylostoma duodenale except: (DP PGMEE 2009)
c. Ascaris lumbricoides a. Cerebrospinal fluid leak
d. All of the above b. Chronic cardiac disease
Ref: Nelson paediatrics 18/e p1450; c. Children less than 2 years
KDT Pharmacology 6/e p809 d. Nephritic syndrome
90. Clinical Manifestations of Congenital Rubella Syndrome in Ref: Ghai 8/e p199
children following Maternal Rubella include all except: 98. Antibodies to one of the following infection is not
a. Nerve deafness (AP 2010) transmitted to child: (DP PGMEE 2010)
b. Paychomotor retardation a. Measles
c. Patent Ductus Arteriosus b. Pertusis
d. Aortic stenosis Ref: Ghai 8/e p272 c. Diphtheria
91. A 6 month old infant presents with an early feature of d. Polio Ref: Ghai 8/e p243
vomiting, more severe diarrhea, mild to moderate fever and 99. All of the following are features of mumps except:
dehydration. The possible infection is due to: (AP 2010) a. Caused by paramyxovirus (Feb DP PGMEE 2009)
a. Norwalk virus b. Aseptic meningitis is a complication in children
b. Rota virus c. Orchitis is a complication in adults
c. Shigella d. Incubation period is less than 2 weeks
d. e. coli Ref: Ghai 8/e p299 Ref: Ghai 8/e p217
92. Baby developed signs and symptoms of congential syphilis 100. A child 18 month old with 2 month history of bloody
after two weeks. This is supported by the following law: Dysentery the probable cause is: (Raj PGI 2008)
a. Kassowitzs law (AP 2011) a. Adenovirus
b. Profeta law b. Rotavirus
c. Colles law c. Campylobacter
d. Didays law d. Norwalk
Ref: Ghai 8/e p272 Ref: Ghai 8/e p296

PEDIATRICS
93. Reactive arthistis, erythema nodosum, haemolytic anaemia 101. Which of the following is best used in the diagnosis of
and dysentery occur more often in enteric infections with: congenital syphilis: (Raj PGI 2009)
a. Entamoeba histolytica (AP 2012) a. FTA-ABS
b. Yersinia enterocolitica b. TPHA
c. Campylobacter jejuni c. IgM-FTAABS
d. Cryptosporidum parvum Ref: Ghai 8/e p296 d. TPI Ref: Ghai 8/e p272
94. Which one of the following investigative features is NOT 102. Which of the following is contraindicated in a 4 year old
indicative of severe malaria: (AP 2012) child with immunosuppression? (Kerala PG 09)
a. Hypoglycemia a. Acellular pertusis
b. Lactic acidosis b. Measles
c. Haemoglobinuria c. Tetanus
d. Macrocytic RBCs Ref: Ghai 8/e p260 d. Varicella Ref: Ghai 8/e p189; Nelson 18/e p1371

Ans. 87. d. All of the above 88. c. 14-24 days 89. a. Enterobius vermicularis 90. d. Aortic stenosis
91. b. Rota virus 92. b. Profeta law 93. c. Campylobacter jejuni 94. d. Macrocytic RBCs
95. c. Hepatitis 96. c. Ty21 a vaccine... 97. c. Children less than... 98. b. Pertusis
99. d. Incubation period... 100. c. Campylobacter 101. c. IgM-FTAABS 102. b and d
44 Jaypees Triple A
103. All the following regarding conjugate pneumococcal c. Booster is not required
vaccine are true except: (Kerala PG 10) d. Incidence of invasive infections are not reduced following
a. Provoke antibody response in 90% of infants and reduces administration
the incidence of nasopharyngeal carriage of vaccine Ref: Ghai 8/e p199; Nelson 18/e p1134
serotypes 105. Which among the following is true regarding exanthem:
b. Enhanced responses after booster doses given at 12-15 a. Roseola infantum is fifth disease (Kerala PG 10)
months of age b. Koplick spot cannot occur before rash
c. It helps to reduce the incidence of otitis media in infants c. Forreschmier spot is petechial spot in soft palate in
d. Cannot be given in children < 2 years of age rubella
Ref: Ghai 8/e p199; Nelson 18/e p1134 d. Symptoms of vaccine (Breakthrough) chickenpox is
104. Which among the following is difference of conjugate similar to unvaccinated type
pneu-mococcal vaccine, when compared to polysaccharide Ref: Ghai 8/ep213; Nelson 18/e p1339, 1317, 1357
pneumococcal vaccine: (Kerala PG 10)
a. Can be given in less than 2 years
b. Contains more serotypes
PEDIATRICS

Ans. 103. d. Cannot be given... 104. a. Can be given 105. c. Forreschmier...


7. HEMATOLOGICAL DISORDERS
46 Jaypees Triple A

HEMATOLOGICAL DISORDERS (QUESTIONS)

1. Which organ is the primary site of hematopoiesis in the 7. All of the following are true of thalassemia major, except:
fetus before midpregnancy: (AIIMS May 06) a. Splenomegaly (AIIMS May 06)
a. Bone b. Target cells on peripheral smear
b. Liver c. Microcytic hypochromic anemia
c. Spleen d. Increased osmotic fragility
d. Lung Ref: Ghai 8/e p423; 7/e p308-309; Nelson 19/e p1674
Ref: Ghai 8/e p623; Robbins 7/e p620, Nelson 18/e p1997; 8. Which of the following aemoglobin (Hb. Estimation will
Nelson 19/e p1650, 1651 be diagnostically helpful in a case of beta thalassemia trait?
2. When does switchover from fetal to adult hemoglobing a. Hb-F (AIIMS May 06)
synthesis begin: b. Hb1-C
a. 14 weeks gestation (AIIMS Nov 04, Nov 05, May 05) c. Hb-A2
b. 30 weeks gestation d. Hb-H
c. 36 weeks gestation Ref: Ghai 8/e p425; 7/e p307-308; Nelson 18/e p2060-2061; 17/e p1633;
d. 7-10 days postnatal Robbins 7/e p635; Nelson 19/e p1676
Ref: Ghai 8/e p419; Nelson 18/e p2001; Nelson 19/e p1662 9. A child died soon after birth. On examination there was
3. A 5-years-old girl came with history of progressively hepatosplenomegaly and edema all over body. Most
increasing pallor since birth and hepatosplenomegaly. probable diagnosis in:
Which of the following is the most relevant test for a. b-thalassemia (AIIMS May 02)
achieving diagnosis: (AI 04) b. a-thalassemia
a. Hb electrophoresis c. Hereditary spherocytosis
b. Peripheral smear examination d. ABO incompatibility/sickle cell anemia
c. Osmotic fragility test Ref: Ghai 8/e p423, 424; 7/e p307-309; Nelson 18/e p2032-2033 and 17/e
d. Bone marrow examination p1633; Nelson 19/e p1676
Ref: Ghai 8/e p456; Nelson 18/e p2035, 2036; Nelson 19/e p1674, 1675 10. Thalassemia occurs due to which mutations:
4. Barts hydrops fetalis is lethal because: (AI 05) a. Missense (PGI Dec 2000)
a. Hb Barts cannot bind oxygen b. Splicing
b. The excess a-globin form insoluble precipitates c. Transition
c. Hb Barts cannot release oxygen to fetal tissues d. Frame-shift
d. Microcytic red cells become trapped in the placenta e. Truncation
Ref: Ghai 8/e p422; 7/e p307-308; Robbins 7/e p636; Nelson 18/e p2016, Ref: Ghai 8/e p425; Harper 27/e, p415, Robbins 7/e, p632,
2033-2037 and 17/e p1633; Nelson 19/e p1675 Nelson 18/e p2035; Nelson 19/e p1674
5. Diagnosis of beta thalassemia is established by: 11. Thalasemia trait; true about: (PGI June 04)
a. NESTROFT Test (AI 05) a. HbF
b. Hb A1, C estimation b. HbA2
c. Hb electrophoresis c. Microcytosis
d. Target cells in peripheral smear d. Severe anemia
Ref: Ghai 8/e p425; 7/e p308-309; Nelson 18/e p2033-2037 and Ref: Ghai 8/e p424, 425; Nelson 18/e p2035, 2036, Robbins 7/e p634-635,
17/e p1633; Robbins 7/e p635; Nelson 19/e p1675 Harrison 17/e, p640, 641; Nelson 19/e p1676
PEDIATRICS

6. A child aged 2 years presents with nonspecific symptoms 12. The primary defet which leads to sickle cell anemia is:
suggestive of anemia. On peripheral blood smear target a. An abnormality in porphyrin part of hemoglobin (AI 03)
cells are seen. He has hypochromic microcytic picture and b. Replacement of glutamate by valine in chain of HbA
Hb of 6 gm%. He also has a positive family history. Next c. A nonsence mutation in the (3-chain of HbA)
investigation of choice is: (AI 01) d. Substitution of valine by glutamate in the achain of HbA
a. Hb electrophoresis Ref: Ghai 8/e p452; Nelson 18/e p2026, Robbins 7/e p628;
b. Coombs test Nelson 19/e p1663
c. Liver function tests 13. Mutation leading to sickle cell anemia:
d. Osmatic fragility test a. Crossover mutation (PGI June 01)
Ref: Ghai 8/e p423; 7/e p308-309; Nelson 18/e p2034-2036 and b. Frame shift
17/e p1633; Robbins 7/e p635; Nelson 19/e p1675 c. Deletion

Ans. 1. b. Liver 2. c. 36 weeks gestation 3. a. Hb electrophoresis 4. c. Hb Barts cannot...


5. c. Hb electrophoresis 6. a. Hb electrophoresis 7. d. Increased osmotic... 8. c. Hb-A2
9. b. a-thalassemia 10. b and d 11. a, b and c 12. b. Replacement...
13. e. Pointmutation
Hematological Disorders 47
d. Nondysfunction c. FactorXIIdeficiency
e. Pointmutation d. Mild aemophilia A
Ref: Ghai 8/e p450, 452; Already explained in Previous Answer Ref: Read below; Nelson 19/e p1698; Internet
14. Regarding G6PD deficiency true are: 20. A 10-years-old girl presents with swelling of one knee joint.
a. Autosomal dominant (PGI June 06) All of the following conditions can be considered in the
b. Bite cell (+) differential diagnosis except:
c. Protects against kala azar a. Tuberculosis (AI 03; AIIMS Nov 01)
d. Enzyme level directly proportional to age of RBC b. Juvenile rheumatoid arthritis
e. Sex preponderance c. Hemophilia
Ref: Ghai 8/e p437; Nelson 18/e p2041; Nelson 19/e p1673 d. Villonodular synovitis
15. True about iron deficiency anemia: (PGI June 05) Ref: Ghai 8/e p462; 7/e p320-321; Robbins 7/e p1310;
a. Microcytic hypochromic anemia Harrison 16/e p2063; Nelson 19/e p1699
b. Decreased TIBC 21. Characteristic lab findings of haemophilia A are:
c. Increased ferritin a. Increase PT (PGI June 06)
d. Bone marrow iron decreased earlier than serum iron b. Increase aPTT
Ref: Robbins 7/e p645; Ghai 8/e p406; Nelson 19/e p1656 c. X-linked recessive
16. The earliest indicator of response after starting iron in a 6 d. Presence of 30% of factor level express the disease
years old girl with iron deficiency is: (AI 06) e. Increased bleeding time
a. Increased reticulocyte count Ref: Ghai 8/e p465; Nelson 18/e p2066-2069; Nelson 19/e p1700
b. Increased aemoglobin 22. Persistent bleeding from umbilical stump, the most
c. Increased ferritin probable diagnosis is: (AIIMS Nov 06)
d. Increased serum iron a. Factor XIII deficiency
Ref: Ghai 8/e p405, 406; 7/e p300-301; Nelson 18/e p2016-2017 and b. Glanzmans thrombasthenia
17/e p1615-1616; Nelson 19/e p1656 c. VWD
17. A nine months old boy of Sindhi parents presented to d. Bernard soulier syndrome
you with complaints of progressive lethargy, irritability Ref: Nelson 18/e p2060-2061 and 17/e p1661;
and pallor since 6 months of age. Examination revealed Wintrob 11/e, p1604; Nelson 19/e p1703; Internet
severe pallor. Investigation showed Hb-3.8 mg%; MCV-58 23. A 15 years old female presented to the emergency
fl; MCH-19.4 pg/cell. Blood film shows osmotic fragility is department with history of recurrent epistaxis, hematuria
normal (target cells and normoblasts). X-ray skull shows and hematochezia. There was a history of profuse bleeding
expansion of erythmoid marrow. Which of the following is from the umbilicus stump at birth. Previous investigations
the most likely diagnosis? (AI 04) revealed normal prothrombin time, activated partial
a. Iron deficiency anemia thromboplastin time, thrombin time and fibrinogen levels.
b. Acute lymphoblastic anemia Her platelet counts as well as platelet function tests were
c. Hemoglobin D disease normal but urea clot lysis test was positive. Which one of
d. Hereditary spherocytosis the following clotting factor is most likely to be deficient?
Ref: Ghai 8/e p405; Nelson 18/e p1528, 2014-2017 and 17/e p1629; a. Factor X (AIIMS May 06)
Nelson 19/e p1656 b. Factor XI
18. True about acute ITP: (PGI June 05) c. Factor XII
a. More common in female d. Factor XIII
b. Specific anti platelet antibodies detected Ref: Ghai 7/e p317-318, 6/e p322; Nelson 18/e p2060-2061 and
c. Viral infection predisposes as seen after vaccination 17/e p1660-1661; Harrison 16/e p682-683; Nelson 19/e p1704; Internet

PEDIATRICS
d. 80% cases transforms to chronic 24. A newborn baby presented with profuse bleeding from
e. More common in children umbilical stump after birth. Probable diagnosis is:
Ref: Ghai 8/e p642; Nelson 18/e p2082, 2084, 2087 and a. Factor XIII deficiency (AIIMS May 07, Nov 06)
17/e p1670-1671; Nelson 19/e p1715 b. VWF deficiency
19. A child underwent a tonsillectomy at 6 years of age with no c. Factor XII deficiency
complications. He underwent a preoperative screening for d. Glanzmann thrombosthenia
bleeding at the age of 12 years before an elective laparotomy, Ref: J.b. Henry clinical diagnosis and management by laboratory
and was found to have a prolonged partial thromboplastin methods 20/e, p650; Nelson 19/e p1704; Internet
time but normal prothrombin time. There was no family 25. Late onset hemorrhagic disease of newborn is characterized
history of bleeding. The patient is likely to have: by all of the following features except: (Al 06)
a. Acquired VitK deficiency (AIIMS Nov 04) a. Usually occurs in cow-milk fed babies
b. Acquired liver disease b. Onset occurs at 4-12 week of age

Ans. 14. b and e 15. a and d 16. a. Increased... 17. a. Iron deficiency...
18. a, b, c and e 19. c. Factor XII deficiency 20. c. Hemophilia 21. b and c
22. a. Factor XIII... 23. d. Factor XIII 24. a. Factor XIII deficiency 25. a. Usually occurs...
48 Jaypees Triple A
c. Intracranial aemorrhage can occur 33. Which one of the following is not a feature of combined
d. Intramuscular vitamin K prophylaxis at birth has a immunodeficiency in children:
protective role a. Decreased total serum immunoglobulin level
Ref: Ghai 8/e p465; Nelson 18/e p773; Nelson 19/e p620, 621 b. Neutropenia and eosinophilia (UPSC 07)
26. The coagulation profile in a 13-years old girl with c. Defective T-cell function
menorrhagia having von Willebrands disease is: d. Recurrent pulmonary infection
a. Isolated prolonged PTT with a normal PT Ref: O.P Ghai 7/e p57, 58, 6/e p187; Harrison 16/e p1942;
b. Isolated prolonged PT with a normal PTT Nelson 17th/e p697
c. Prolongation of both PT and PTT 34. Thrombocytopenia in a newborn baby can be caused by all
d. Prolongation of thrombin time (AI 05) except: (AIIMS Nov 99)
Ref: Robbins 7/e p654-655; Ghai 8/e p469; Nelson 18/e a. ABO incompatibility
p2060, 2061 and 17/e p1662; Nelson 19/e p1704-1706 b. Isoimmune thrombocytopenia
27. Which of the following is generally not seen in idiopathic c. Autoimmune thrombocytopenia
thrombocytopenic purpura (ITP)? d. SLE Ref: Nelsons Essentials of Pediatrics 5/e, p716
a. More common in females (AIIMS Nov 04) 35. A child is brought with drowsiness, decreased deep tendon
b. Petechiae, ecchymosis and bleeding reflexes and seizures. On examination the child has a line
c. Palpable splenomegaly on gums. There is history of constipation. Which will be
d. Increased megakaryocytes in bone marrow most appropriate drug that should be used in this child:
Ref: Robbins 7/e p654655; Ghai 8/e p462; a. EDTA (AI 07)
Nelson 18/e p2060, 2061 and 17/e p1662; Nelson 19/e p1715 b. DMSA
28. Iron requirement is determined from the equation: c. BAL
a. 3 x wt. (kg) x Hb deficit (gm/dl) (AMU 05) d. Penicillamine
b. 3.3 x wt. (kg) x Hb deficit (gm/dl) Ref: Ghai 8/e p696; KDT 6/e p645, Nelson 18/e p2916;
c. 4 x wt. (kg) x Hb deficit (gm/dl) Nelson 19/e p2452
d. 4.3 x wt. (kg) x Hb deficit (gm/dl) 36. Treatment of choice for thalassemia major is: (DNB 2007)
Ref: OP Ghai 8/e p406; 7/e p301 a. Blood transfusion and iron therapy
29. Fanconis anemia is a: (Jipmer 03) b. Folic acid and desferrioxamine
a. Constitutional anemia c. Blood transfusion and desferrioxamine
b. Hemolytic anemia d. Iron, blood transfusion and desferrioxamine
c. Iron deficiency anemia Ref: Ghai 8/e p289; Nelson, 18/e p2032
d. Auto - immune anemia 37. A newborn baby presented with profuse bleeding from
Ref: Nelson 17/e p1642; Internet umbilical stump after birth. Probable diagnosis is:
30. Quinine induced thrombocytopenia is: a. Factor XIII deficiency (DNB 2009)
a. Antibody mediate (Jipmer 04) b. VWF deficiency
b. Dose related toxicity c. Factor XII deficiency
c. Idiosyncratic reaction d. Glanzmann thrombosthenia
d. Inhibits production of platelets Ref: 8/e p613; J.b. Henty, Clinical diagnosis and
Ref: Harrison 16/e p674675; Internet management by laboratory Methods, 20/e p650
31. Iron supplementation in a healthy term breast fed baby 38. Non immune Hydrops is seen in: (DNB 2010)
should be started at the age of: a. Hb Barts Thalessemia
a. 2 weeks (COMED 06) b. Rh Incompatibility
b. 4 weeks c. Syphilis
PEDIATRICS

c. 8 weeks d. EBV infection


d. 6 weeks Ref: Ghai 8/e p680; Nelson 18/e p2375, 2379
Ref: Ghai 8/e p352; Park 18/e p405 39. In children with idiopathic thropmbocytopenic purpura
32. A four-years-old child presents with mild fever, malaise, (ITP), the thrombocytopenia is considered as severe and
prupura, arithritis, abdominal pain and microscopic unsafe at a level below: (MP PGI 2008)
hematuria. What would be the most likely diagnosis: a. 10,000/m2
a. Thrombasthenia (UPSC 07) b. 30,000/mm3
b. Idiopathic thrombocytopenic c. 20,000/mm3
c. Systemic lupus erythematosus d. 40,000/mm
d. Henoch-Schonlein purpura Ref: Ghai 8/e p351; Nelsons 18/e p2082-2084;
Ref: OP Ghai 8/e p623 IAPs emergencies 2/e p273

Ans. 26. a. Isolated prolonged... 27. c. Palpable splenomegaly 28. c. 4 x wt. (kg)... 29. a. Constitutional anemia
30. a. Antibody mediate 31. d. 6 weeks 32. d. Henoch-Schonlein... 33. b. Neutropenia...
34. a. ABO incompatibility 35. a. EDTA 36. d. Iron, blood... 37. a. Factor XIII deficiency
38. a. Hb Barts... 39. a. 10,000/m2
Hematological Disorders 49
40. Splenectomy eliminates most of the hemolysis associated a. Beta Thalassemia minor and major
with: b. Copper deficiency induced anaemia
a. Sickle cell anemia (MP PGI 2008) c. Lead poisoning
b. Thallassemia d. X-linked receive sideroblastic anaemia
c. Hereditary spherocytosis Ref: Ghai 8/e p341
d. Glucose6 phosphate dehydrogenase (G-6-PD) deficiency 49. The coagulation profile in a 13-year old girl with
Ref: Ghai 8/e p347; Bailey and Loves 25/e p1106 menorrhagia having von willebrands disease is:
41. Optimal oral dose of elemental iron to treat iron deficiency a. Isolated prolonged PTT with a normal PT
anemia is: (MP PGI 2008) b. Isolated prolonged PT with a normal PTT
a. 1-2 mg/kg/day c. Prolongation of both PT and PTT (DP PGMEE 2010)
b. 3-6 mg/kg/day d. Prolongation of thrombin time
c. 7-10 mg/kg/day Ref: Ghai 8/e p349; Nelson 18/e p2073, Harrison 16/e p676
d. 11-14 mg/kg/day 50. An asymptomatic growth retarded infant is born by
Ref: Ghai 8/e p334; Nelsons 18/e p2016 and KDTs 6/e p586 caesarean section for maternal hypertension. He is noted to
42. Drug which may lead to hemolysis in a child with G6PD have low platelet count. What is the most likely diagnosis?
deficiency is: (Karnataka PGI 2011) a. Placental insufficiency (DP PGMEE 2010)
a. Penicillin b. Gram-negative septicemia
b. Primaquine c. Auto-immune thrombocytopenia
c. Ceftriaxone d. Glanzmans thromboesthenia
d. Erythromycin Ref: Ghai 8/e p339; Harrisons 17/e p657 Ref: Ghai 8/e p352; Still searching
43. Vital pneumonia is caused by all except: (UP PGI 09) 51. Aplastic anemia is associated with: (Raj PG 2008)
a. Mumps a. CD 33
b. Measles b. CD 34
c. Herpes c. CD 35
d. Cytomegala virus Ref: Ghai 8/e p577; 7/e p352,355; (UP-03) d. CD 36 Ref: Ghai 8/e p345
44. Uveo-parotitis is seen in: (UP PGI 09) 52. Regarding Kawasaki disease all are true except:
a. Mumps a. Cervical lymph node enlargement is the least common
b. Measles among diagnostic criteria (Kerala PG 09)
c. Sarcoidosis b. Conjunctival ingestion is last to disappear among the
d. Cystic fibrosis diagnostic criteria
Ref: Ghai 8/e p630; Basic pathology Robbins 8/e p738 c. Coronary artery involvement is less common in infants
45. Constitutional pancytopenia can be seen in following d. Atypical Kawasaki is more prone for cardiac involvement
except: (MHPGM-CET 2010) Ref: Ghai 8/e p631; Nelson 18/e p1039
a. Fanconis anemia 53. True statement about sickle cell anaemia: (Kerala PG 09)
b. Diamond-Blackfan syndrome a. Hydroxyurea treatment do not decrease painful crisis
c. Dyskeratosis congenita b. Ultimate height is achieved
d. Schwachman Diamond syndrome c. Hemorrhagic stroke in infants seen
Ref: Ghai 8/e p347; Nelson paediatrics 18/e p2047; Table 468-1 d. Most common surgery is cholecystectomy
46. Fetal Hb is: (AP 2011) Ref: Ghai 8/e p344; Nelson 18/e p2030
a. Two Alpha & two beta 54. Which among the following diagnostic criterion is most
b. Two Alpha & two gamma characteristic of Kawasakis disease: (Kerala PG 10)

PEDIATRICS
c. Two Alpha & two delta a. Periungual desquamation
d. Two Alpha & two epsilon Ref: Ghai 8/e p330 b. Generalised lymphadenopathy
47. A 3-years-old boy with a week history of fever has diffuse c. Splenomegaly
faint rashes, erythema of palms and soles, enlarged d. Exudative conjunctivitis
unilateral cervical node and strawberry tongue. The Ref: Ghai 8/e p631; Nelson 18/e p1039
diagnosis is: (AP 2012) 55. Typically Peribronchovascular and Subpleural nodules on
a. Scarlet fever HRCT Thorax are seen in: (J & K PG 2010)
b. Erythema infectiosum a. Emphysema
c. Infectious mononucleosis b. Endobronchial spread of tuberculosis
d. Kawasakis disease Ref: Ghai 8/e p631 c. Sarcidosis
48. Basophilic stippling of red cells in a child with hypochromic, d. Interstital Pulmonary Fibrosis
microcytic anaemia is indicative of: (AP 2012) Ref: Ghai 8/e p373

Ans. 40. c. Hereditary... 41. b. 3-6 mg/kg/day 42. b. Primaquine 43. a. Mumps
44. c. Sarcoidosis 45. b. Diamond-Blackfan... 46. b. Two Alpha... 47. d. Kawasakis disease
48. a. Beta Thalassemia... 49. a. Isolated prolonged... 50. a. Placental insufficiency 51. a. CD 33
52. c. Coronary artery... 53. d. Most common.... 54. a. Periungual... 55. c. Sarcidosis...
50 Jaypees Triple A
56. The commonest blood transfusion dependant anemia in c. RSV
children in India is: (J & K PG 2010) d. Pneumocystitis carnii
a. Thalassemia major Ref: Ghai 8/e p377
b. Thalassemia minor 58. The most common leukocytoclastic vasculitis affecting
c. Iron deficiency anemia children is: (AI 05)
d. Megalo blastic anemia a. Takayasu disease
Ref: OP Ghai 8/e p341 b. Mucocutaneous lymph node syndrome (Kawasaki
57. Pneumatoceles are characteristic X ray findings in disease)
pneumonia caused by the following organism: c. Henoch Schonelin purpura
a. Streptococcus pneumonia (J & K PG 2010) d. Polyarteritis nodosa
b. Staphylococcus and Klebsiella Ref: Ghai 8/e p632; 7/e p607; Harrison 16/e p2010;
Robbins illustrated 7/e p539-541; Nelson 19/e p1789
PEDIATRICS

Ans. 56. a. Thalassemia major 57. b. Staphylococcus... 58. c. Henoch Schonelin...


8. DISORDERS OF
CARDIOVASCULAR SYSTEM
52 Jaypees Triple A

DISORDERS OF CARDIOVASCULAR SYSTEM (QUESTIONS)

1. Oxygenated blood to the fetus is carried by: 8. In which of the following conditions left atrium is not
a. Umbilical artery enlarged: (AI 06)
b. Umbilical vein (PGI June 07) a. Ventricular septal defect
c. SVC b. Atrial septal defect
d. Pulmonary artery Ref: Ghai 8/e p396; Nelson 19/e p1529 c. Aortopulmonary window
2. Which of the following malformation in a newborn is d. Patent ductus arteriosus
specific for maternal insulin dependent diabetes mellitus? Ref: Ghai Ghai 8/e p402-403; 7/e p402; Nelson 18/e p1888-1889 and
a. Transposition of great arteries (AI 06) 17/e p1514; Nelson 19/e p1554, 1555
b. Caudal regression 9. All of the following are true about ASD except:
c. Holoprosencephaly a. Right atrial hypertrophy (AI 01)
d. Meningomyelocele b. Left atrial hypertrophy
Ref: Ghai 8/e p400; Current Diagnosis and treatment Obstetrics and c. Right ventricular hypertrophy
Gynaecology 10/e p312; Dutta 6/e p287 d. Pulmonary hypertension
3. Which of the following syndromes is best associated with Ref: Ghai 8/e p402; 7/e p401-402; Nelson 18/e p1883, 1884;
congenital heart disease? Nelson 19/e p1554
a. Lesch-Nyhan syndrome (AIIMS May 05) 10. A young female presents with history of dyspnea on
b. Rasmussen syndrome exertion. On examination, she has wide, fixed split S2 with
c. Holt Oram syndrome ejection systolic murmur (m/VI) in left second intercostal
d. Leopard syndrome space. Her ECG shows left axis deviation. The most
Ref: Ghai 8/e p402; 7/e p400; Nelson 18/e p1883, 1885 and 17/e p1501; probable diagnosis is:
Harrison 16/e p1382; Nelson 19/e p1531 a. Total anomalous pulmonary venous drainge
4. ASD is seen in all except: (PGI June 01) b. Tricuspid atresia (AIIMS May 2003)
a. Turners syndrome c. Ostium prirnum atrial septal defect
b. Ellis van creveld syndrome d. Ventricular septal defect with pulmonary arterial
c. Downs syndrome hypertension
d. Halt-oram syndrome Ref: Ghai 8/e p402; 7/e p402; Nelson 18/e p1883, 1884
e. TAR syndrome 11. A patient presents with LVH. And pulmonary complications.
Ref: Ghai 8/e p403; 7/e p400; Nelson 8/e, p2386, 2389 and 17/e p1501; ECG, shows left axis deviation. Most likely diagnosis is:
Harrison 16/e p1382; Nelson 19/e p1531 a. TOP (AI 01)
5. A child after 4 weeks of birth acyanotic, ejection systolic b. Tricuspid atresia
murmur detected causes are: c. TAPVC
a. VSO (PGI June 08) d. VSD
b. PDA Ref: Ghai 8/e p429; 7/e p403-404; Nelson 18/e p1913, 1914;
c. TOF Nelson 19/e p1556, 1557
d. Coarctation of aorta 12. A child with large perimembranous VSD has congestive
e. Tricurpid stenosis heart failure. What may be the cause of improvement of
Ref: Ghai 8/e p413; 7/e p419; Nelson 19/e p1568 cardiac failure in the patient?
6. Cyanosis is seen in: (PGI Dec 01) a. Aortic regurgitation (AIIMS Nov 01)
PEDIATRICS

a. Persistent ductus arteriosus b. Vascular changes in pulmonary circulation


b. Tricuspid atresia c. Infective endocarditis
c. Ostium premium ASD d. Closure of VSD spontaneously
d. Eisenmenger complex Ref: Ghai 8/e p396; 7/e p403-404;
e. Tetralogy of Fallot Ref: Ghai 8/e p420 Nelson 8/e, p1888-1891 and 17/e p1508; Nelson 19/e p1557
7. Which one of the following does not produce cyanosis in 13. A child with VSD presents with development of cyanosis
the first year of life: (AIIMS May 03) because of Eisenmenger physiology. What is the correct
a. Atrial septal defect sequence of events which leads to this change:
b. Hypoplastic left heart syndrome (AIIMS Nov 00)
c. Truncus arteriosus a. Left to right shunt, pulmonary hypertension, right
d. Double outlet right ventricle ventricular hypertrophy, right to left shunt
Ref: Ghai 8/e p402; Myung K Park 5/e, p75; Nelson 19/e p1551

Ans. 1. b. Umbilical vein 2. b. Caudal regression... 3. c. Holt Oram syndrome 4. a and e


5. a, b and d 6. b, d and e 7. a. Atrial septal defect 8. b. Atrial septal defect
9. b. Left atrial... 10. c. Ostium prirnum... 11. b. Tricuspid atresia 12. b. Vascular changes...
13. b. Left to right shunt...
Disorders of Cardiovascular System 53
b. Left to right shunt, right ventricular hypertrophy, c. Nitric oxide
pulmonary hypertension, right to left shunt d. Indomethacin
c. Pulmonary hypertension, right to left shunt right Ref: Ghai 8/e p402; Nelson 18/e p1856-1857 and 17/e p1580;
ventricular hypertrophy, left to right shunt KDT 5/e p164; Nelson 19/e p1560, 61
d. Left to right shunt, right ventricular hypertrophy, right to 19. Right sided aortic arch is most strongly/commonly
left shunt, pulmonary hypertension associated with? (AI 09)
Ref: Ghai 8/e p421; 7/e p403-404; a. Tetralogy of Fallot (TOF)
Nelson 8/e, p1867 and 17/e p1508; Nelson 19/e p1556 b. Ventricular Septal Defect (VSD) with pulmonary atresia
14. A 29-day-old child presents with features of congestive c. Corrected TGA
cardiac failure and left ventricular hypertrophy. d. Truncus arteriosus
Auscultation shows a short systolic murmur. Most likely Ref: Ghai 8/e p419; 7/e p408; Nelson 18/e p1930; Nelson 19/e p1574
diagnosis is: (AIIMS Nov 00) 20. The following features are true for tetralogy of Fallot,
a. Rheumatic fever except:
b. Tetralogy of fallot a. Ventricular septal defect (AI 06)
c. Transposition of great arteries b. Right ventricular hypertrophy
d. Ventricular septal defect c. Atrial septal defect
Ref: Ghai 8/e p396; 7/e p404; Nelson 8/e, p1888-1891 d. Pulmonary stenosis
and 17/e p1508-1509; Nelson 19/e p1556 Ref: Ghai 8/e p403; 7/e p401-402; Nelson 18/e p1883-1886 and
15. All of the following statements about Patent Ductus 17/e p1524-1526; Nelson 19/e p1573, 1574
Arteriosus (PDA) are true, except? (AI 08) 21. In which of the following a Corn en Sabot shape of the
a. It is more common in males than females heart is seen: (AI 04)
b. It is a common heart lesion in rubella a. Tricuspid atresia
c. Treatment is closure of defect by ligation and division of b. Ventricular septal defect
ductus c. Transposition of great arteries
d. Hypoxia and immaturity are important in maintaining d. Tetralogy of Fallot
the patency
Ref: Ghai 8/e p405; 7/e p410; Nelson 18/e p1906-1912 and 17/e p1525;
Ref: Ghai 8/e p404; 7/e p405-406; Nelson 18/e p1891; Nelson 19/e p1575
Nelson 19/e p1559
22. Potts shunt is: (AI 01)
16. True statement about Ductus Arteriosus is: (AI 00)
a. Right subclavian artery to right pulmonary artery
a. It undergoes anatomic closure within 24 hours of birth
b. Descending aorta to left pulmonary artery
b. Forms the ligamentum venosum in later life
c. Left subclavian to left pulmonary artery
c. It is induced to close by high levels of prostaglandins
d. Ascending aorta to right pulmonary artery waterson
d. May cause a machinery murmur by its patency
Ref: Ghai 8/e p422; 7/e p409
Ref: Ghai 8/e p402; 7/e p405-406; Nelson 18/e p1863-1864 and 17/e p580;
23. Essential criteria for TOF includes all except?
KDT 5/e, p176; Nelson 19/e p1559
a. Valvular stenosis (AIIMS Nov 08)
17. A premature infant is born with a patent ductus arteriosus.
b. Infundibular stenosis
In closure can be stimulated by administration of:
c. Over riding of aorta

PEDIATRICS
a. Prostaglandin analogue (AIIMS May 06)
d. RVH Ref: Ghai 8/e p419;
b. Estrogen
Pediatric Cardiology for practioners, Myung K Park, 4/e p189
c. Anti-estrogen compounds
24. Recurrent respiratory tract infections may occur in all of the
d. Prostaglandin inhibitors
following except: (AIIMS Nov 05)
Ref: Ghai 8/e p402; 7/e p406; Nelson 18/e p1891-1893 and 17/e p580;
a. Ventricular septal defect
KDT 5/e, p176; Nelson 19/e p156
b. Tetralogy of Fallot
18. The most appropriate management for maintaining patency
c. Transposition of great arteries
of ductus arteriosus in a neonate is:
d. Total anomalous venous return
a. Prostaglandin E1 (AIIMS May 04)
Ref: Ghai 8/e p419; 7/e p408; Nelson 19/e p1576
b. Oxygen

Ans. 14. d. Ventricular septal... 15. a. It is more common... 16. d. May cause a... 17. d. Prostaglandin...
18. a. Prostaglandin E1... 19. a. Tetralogy of Fallot (TOF) 20. c. Atrial septal defect... 21. d. Tetralogy of Fallot
22. b. Descending aorta... 23. a. Valvular stenosis 24. b. Tetralogy of Fallot
54 Jaypees Triple A
25. All of the following are true regarding Tetralogy of Fallot later ballooned atrial septostomy was done which showed
except: (AIIMS May 05) improvement in oxygenation. The most likely diagnosis of
a. Ejection systolic murmur in second intercostal space this infant is: (AI 04)
b. Single second heart sound a. Tetralogy of Fallot
c. Predominantly left to right shunt b. Transposition of great vessels
d. Normal jugular venous pressure c. Truncus Arteriosus
Ref: Ghai 8/E P419; 7/e p408-409; Nelson 17/e p1524-1526; d. Tricuspid Atresia
Nelson 19/e p1574 Ref: Ghai 8/e p429; 7/e p413; Nelson 18/e p1918, 1929 and 17/e p524;
26. A 6 months old child with Tetralogy of Fallot develops Nelson 19/e p1582
cyanotic spell initiated by crying. Which one of the 32. A neonate has recurrent attacks of abdominal pain, restless
following drugs you would like to avoid? irritability and diaphoresis on feeding. Cardiac auscultation
a. Sodium bicarbonate (AIIMS Nov 04) reveals a nonspecific murmur. He is believed to be at risk
b. Propranolol for MI Likely diagnosis here is: (AI 01)
c. Phenylephrine a. ASD
d. Isoprenaline b. VSD
Ref: Ghai 8/e p419; 7/e p408-409; Nelson 18/e p1906-1912 and 17/e c. TOF
p1524-1525; Canadian Journal of Anaesthesia 50, 926-929 (03); d. Anomalous coronary artery
Nelson 19/e p1576, 1577 Ref: Ghai 8/e p400; Nelson 118/e p 1922-1924and 7/e p1546;
27. Fallots tetralogy manifestation are: Nelson 19/e p1598
a. Left axis deviation (PGI Dec 05) 33. A neonate has central cyanosis and short systolic murmur
b. Left ventricular hypertrophy on the 2nd day of birth. The diagnosis is:
c. VSD a. Tetralogy of Pallets (AIIMS May 01)
d. Blalock taussig shunt is between pulmonary artery and b. Transposition of great vessels
subclavian artery
c. Atrial septal defect
e. Morphine is contraindicated in cyanotic spells
d. Ventricular Septal defect
Ref: Ghai 8/e p419; 7/e p408-409;
Ref: Ghai 8/e p429; 7/e p408-409;
Nelson 8/e p1888-1891, 1900-1912 and 17/e p1524-1526
Nelson 8/e, p1906-1912 and 17/e p1524-1525;
28. Components of Tetralogy of Fallot is/are: Nelson 19/e p1582
a. VSD (PGI June 04)
34. 7 day old baby presented in the emergency department
b. Lt. Ventricular hypertrophy
with unconscious, blue in appearance with 85 % in Oxygen
c. Lt. Axis deviation
saturation. The diagnosis?
d. Taussig-Blalock shunt is between pulmonary and
a. Tetralogy of Fallot (PGI Dec 08)
subclavian artery
b. TGA
e. Morphine is given for Cyanosis
Ref: Ghai 8/e p419; 7/e p408-409 c. TAPVC
d. PDA
29. Not seen in TOF: (PGI Nov 2009)
Ref: Ghai 8/e p429; Nelson 19/e p1585-1587
a. Squatting relieves spells
b. Cyanosis 35. The most common type of total anomalous pulmonary
c. O2, Morphine useful venous connection is: (AI 05)
d. LVH a. Supracardiac
e. PND b. Infracardiac
f. Clubbed feet c. Mixed
d. Cardiac
PEDIATRICS

Ref: OP Ghai 8/e p419


Ref: Ghai 8/e p413; Nelson 19/e p1589
30. A five years old child presents with left ventricular
hypertrophy and central cyanosis what is the most probable 36. A newborn infant presented with congestive heart failure
diagnosis? (AIIMS Nov 00) which was not improving on treatment. On examination,
a. Tricuspid atresia there is bulging anterior fontanelle with bruit. CT showed
b. Eisenmenger syndrome midline hypoechoic lesion with dilated lateral ventricles.
c. Tetralogy of Fallot Diagnosis is?
d. Total anomalous pulmonary venous drainage a. Vein of Galen malformation (AIIMS May 2010)
Ref: Ghai 8/e p421; 7/e p409-410; b. Arachnoid cyst
Nelson 8/e, p1913, 1914 and 17/e p1531; Nelson 19/e p1580 c. Teratoma
31. A five day old, full term male infant was severely cyanotic d. Encephalocele
at birth. Prostaglandin E was administered initially and Ref: Ghai 8/e p419; Nelson 18/e p1988

Ans. 25. c. Predominantly... 26. d. Isoprenaline 27. c and d 28. a, d and e


29. d and e 30. a. Tricuspid atresia 31. b. Transposition... 32. d. Anomalous...
33. b. Transposition... 34. b and c 35. a. Supracardiac 36. a. Vein of Galen...
Disorders of Cardiovascular System 55
37. A newborn has congenital heart failure, not improving on a. Congenital aortic stenosis
treatment. He has bulging anterior fontanelles with a bruit b. Coarctation of aorta
on auscultation. On trans fontanelle USG a hypoechoic c. Patent ductus arteriosus
midline mass is seen with dilated lateral ventricles. Most d. Congenital aortoiliac disease
probablediagnosis is: (AIIMS Nov 06) Ref: Ghai 8/e p423; 7/e p419; Nelson 18/e p1900-1903 and 17/e p520;
a. Vein of Galen malformation Nelson 19/e p1567, 1568
b. Arachnoid cyst 44. In post ductal coarctation of the aorta, blood flow to the
c. Medulloblastoma lower limb is maintained through which of the following
d. Encephalocele Ref: Ghai 8/e p419; Various Books arteries: (AIIMS Nov 07)
38. Eisenmenger syndrome is characterized by all except: a. Umblical artery and subcostal arteries
a. Return of left ventricle and right ventricle to normal size b. Thoracic and pericardiophrenic arteries
b. Pulmonary veins not distended c. Intercostal arteries and superior epigastric artery
c. Pruning of peripheral pulmonary arteries d. Ant and post circumflex arteries
d. Dilatation of central pulmonary arteries Ref: www.emedicine.com/radio/topic42.htm, http://perfline.com/student/
Ref: Ghai 8/e p428; 7/e p400-401; coa.html, Grays anatomy 38/e Section 10 cardiovascular system;
Nelson 18/e p1727, 1936-1937 and 17/e p1531; Nelson 19/e p1601 Nelson 19/e p1568; Ghai 8/e p432
39. Eisenmenger complex is common in adult in: 45. All of the following causes death in coarctation of Aorta
a. VSD (PGI June 00) except: (PGI June 2000)
b. ASD a. Infective endocarditis
c. PDA b. CCF
d. Cushion defect c. Intra cranial hemorrhage
Ref: Ghai 8/e p427; 7/e p405, 409-414; d. Anterior MI
Nelson 8/e, p1727, 1936, 1937 and 17/e p1531; Nelson 19/e p1601, 1602 Ref: Ghai 8/e p423
40. All are signs of impending Eisenmenger syndrome except: 46. A 4 years old girl always had to wear warm socks even
a. Increased flow murmur across tricuspid and pulmonary in summer season. On physical examination, it was noticed
valve (AIIMS May 2010) that she had high blood pressure and her femoral pulse
b. Single S2 was weak as compared to radial and carotid pulse, a chest
c. Increased intensity of P2 radiography showed remarkable notching of ribs along
d. Graham Steell murmur with their lower borders. This was due to: (AIIMS Nov 02)
Ref: Ghai 8/e p428; 1. Nelson textbook of Pediatrics, 181 e.; 2. Harrisons a. Femoral artery thrombosis
principles of Internal Medicine, 17/e; Nelson 19/e p1601 b. Coarctation of aorta
41. A child with VSD presents with development of cyanosis c. Raynauds disease
because of Eisenmenger physiology. What is the correct d. Takayasa arteritis Ref: Ghai 8/e p423
sequence of events which leads to this change: 47. A child after 4 weeks of birth acyanotic, ejection systolic
a. Left to right shunt, pulmonary hypertension, right, murmur detected causes are/is:
ventricular hypertrophy, right to left shunt a. VSD (PGI June 08)
b. Left to right shunt, right ventricular hypertrophy, b. PDA
pulmonary hypertension, right to left shunt c. TOP
c. Pulmonary hypertension, right to left shunt right d. Coarctation of aorta
ventricular hypertrophy, left to right shunt e. Tricuspid stenosis
d. Left to right shunt, right ventricular hypertrophy, right to Ref: Ghai 8/e p423; 428
left shunt, pulmonary hypertension (AIIMS Nov 2000) 48. About carey Combs murmur which is false:

PEDIATRICS
Ref: Ghai8/e p428; Nelson 18/e p1936; Nelson 19/e p1601 a. Delayed diastole murmur (AIIMS Nov 06)
42. Which condition is most commonly associated with b. Seen in rheumatic fever
coarctation of aorta? (AI 09, AI 08) c. Can be associated with AR
a. PDA d. Low pitched murmur
b. Bicuspid aortic valve Ref: Ghai 8/e p423; Nelson 19/e p1627
c. Aortic stenosis 49. True about Rheumatic fever: (PGI Dec 03)
d. VSD a. Chorea is aggravated during pregnancy
Ref: Ghai 8/e p425; 7/e p419; Nelson 18/e p1900; Nelson 19/e p1567 b. Chorea and arthritis co-existing
43. A 1 month old boy is referred for failure to thrive. On c. Subcutaneous nodules are tender
examination, he shows feature of congestive failure. The d. Erythema multiforme seen
femoral pulses are feeble as compared to branhial pulses. Ref: Ghai 8/e p433-435; Harrison 16/e p1978; Nelson 18/e p1142
The most likely clinical diagnosis is: (AI 06)

Ans. 37. a. Vein of Galen... 38. a. Return of left... 39. a, b and c 40. a. Increased flow...
41. b. Left to right... 42. b. Bicuspid aortic valve 43. b. Coarctation of aorta 44. c. Intercostal arteries...
45. d. Anterior MI 46. b. Coarctation of aorta 47. a, b and d 48. c. Can be associated...
49. a. Chorea is...
56 Jaypees Triple A
50. True about subcutaneous nodule in Rheumatic fever: 57. Commonest cause of enlarged cardiac shadow in X ray of
a. Non tender (PGI Dec 07) a child is: (Karnataka 00)
b. Most common manifestation a. PDA
c. Present on extensor surfaces b. Coarctation of Aorta
d. Associated with arthritis c. Pericarditis
Ref: Ghai 8/e p433; Nelson 18/e p1142, 1143; Nelson 19/e p1627, 1628 d. Rheumatic carditis Ref: Ghai 8/e p433; 7/e p384
51. Drug of choice for Rheumatic fever prophylaxis in penicillin 58. Uncommon finding in congestive cardiac failure in
allergic patient: (AIIMS May 07) newborn:
a. Erythromycin a. Tachycardia (CMC 01)
b. Clindamycin b. Tachypnoea
c. Vancomycin c. Hepatomegaly
d. Gentamycin d. Pedal edema Ref: O.P. Ghai 8/e p396; 7/e p375
Ref: Ghai 8/e p432; Nelson 19/e p1627 59. In a patient of rheumatic carditis dose of steroid is given
52. Which of the following is a minor criteria for diagnosis of for: (Kerala 04)
Rheumatic fever (RF) according to modified Jones criteria: a. 3 weeks
a. ASO titer (AI 07) b. 6 weeks
b. Past history of Rheumatic fever c. 9 weeks
c. Fever d. 12 weeks Ref: Ghai 8/e p433; 7/e p383
d. Subcutaneous nodules 60. The following statements are true of patent ductus
Ref: Nelson 18/e p1141; Ghai 8/e p432-435 arteriosus (PDA) except: (Kerala 00)
53. 8 years old child presented with altered sensorium a. Spontaneous closure occurs in some term infants
and seizure. On examination BP was 180/120. Correct b. Pulmonary hypertension develops
statements: (PGI Dec 04) c. Bacterial endocarditis is more frequent with small PDA
a. Sodium nitroprusside strips d. Recurrent chest infection and congestive failure may
b. Cause of hypertension is essential hypertension develop
c. IV labetolol, hydralazine, and diaoxide are given e. Anatomic existence of PDA is an indication for surgery
d. Nifedipine is used Ref: Ghai 8/e p415; 7/e p405; Nelson 17/e p1511; Harrison 16/e p1386
e. Pheochromocytoma mimics the condition 61. Which one of the following is the most common cause of
Ref: Ghai 7/e p434-435, 6/e p436; Nelson 8/e, p 1993, 1994 and 17/e cyanotic heart disease: (UPSC 02)
p1597, 1919; Nelson 19/e p1645 a. Dextrocardia
54. Which of the following statements regarding Kawasaki b. Fallots tetralogy
disease is true: (AI 08) c. Atrial septal defect
a. Associated with coronary artery aneurysm into 25% of d. Coarctation of aorta Ref: Ghai 8/e p413; 7/e p408
untreated cases 62. Not a feature of Fallots tetrology: (MAHE 05)
b. It is the most common cause of vasculitis in children a. Left ventricular hypertrophy
c. IV immunoglobulins are recommended only if coronary b. Boot shaped heart
artery is involved c. VSD
d. Lymph node biopsy is used for diagnosis d. Overriding of arch of aorta
Ref: Nelson 19/e p1638; Internet Ref: Ghai 8/e p413; 7/e p408
55. Cardiac tumor in childhood includes? 63. All of the following are acyanotic congenital heart disease
a. Rhabdomyoma (PGI Dec 08) except: (SGPGI 05)
b. Lymphoma a. VSD
PEDIATRICS

c. Atrial myxoma b. PDA


d. Sarcoma c. ASD
e. Fibroma d. Tetralogy of fallot
Ref: Ghai 8/e p599-622; Nelson 17/e p1581; 18/e p1975; Nelson 19/e p1637 Ref: Ghai 8/e p413-415; 7/e p408
56. Bacterial endocarditis is most commonly caused by: 64. Children born to mothers with systemic lupus erythematosis
a. Hemolytic Streptococci (PGI Dec 03) are likely to have one of the following anomalies:
b. Hemolytic Streptococci a. Atrial septal defect (Karnataka 03)
c. Staphylococcus aureus b. Tetralogy of fallot
d. Cardiobacterium c. Transposition of great vessels
e. Staph epidermidis d. Complete heart block
Ref: Ghai 8/e p443; Harrison 17/e p790; Nelson 18/e p1953 Ref: Ghai 8/e p457; Nelson 17/e p1563

Ans. 50. a and c 51. a. Erythromycin 52. c. Fever 53. a, d and e


54. a. Associated with... 55. a, c and e 56. c. Staphylococcus... 57. c. Pericarditis
58. d. Pedal edema 59. a. 3 weeks 60. a and e 61. b. Fallots tetralogy
62. a. Left ventricular... 63. d. Tetralogy of fallot... 64. d. Complete heart block
Disorders of Cardiovascular System 57
65. The treatment of choice for a case of congestive failure with c. Vein of Galen malformation
hypertension is: (JIPMER 04) d. Arachnoid cyst
a. ACE inhibitors Ref: Nelson 18/e p1988, www.emedicine.com/neuro/topic538.htm,www.
b. a blockers veinofgalen.co.uk; Nelson 19/e p2009; Internet
c. Cat 2 channel blockers 72. Pentalogy of fallot has which one of following extra entities:
d. Nitrates a. ASD
Ref: Ghai 8/e p396; Harrison 16/e p1478 table (230.11) b. VSD
66. Commonest cause of heart failure in infancy is: c. RVH
a. Myocarditis (Corned 08) d. Pulmonary stenosis
b. Rheumatic fever Ref: Ghai 8/e p403; 7/e p409; Nelson 18/e p1906-1912
c. Cardiomyopathy 73. A neonate has central cyanosis and short systolic murmur
d. Congenital heart disease Ref: OP Ghai 8/e p396; 7/e p375 on the 2nd day of birth. The diagnosis is: (DNB 2007)
67. Most common type of atrial septal defect is: a. Tetralogy of Fallots
a. Ostium primum (Corned 08) b. Transposition of great vessels
b. Ostium secondum c. Atrial septal defect
c. Endocardial cushion defect d. Ventricular Septal defect
d. Endocardial hypertrophy Ref: OP Ghai 8/e p402 Ref: Nelson, 16/e p1385: OP Ghai 8/e p624
68. A two-year-old boy presented with episodes of becoming 74. Commonest cause of heart failure in infancy is: (DNB 2007)
dusky. On examination, there was central cyanosis and a. Myocarditis
dubbin. There was no pallor, edema or respiratory distress. b. Rheumatic fever
The heart was normal sized with a parasternal heave. A c. Cardiomyopathy
systolic thrill was palpable over the left middle sternal d. Congenital heart disease
-border. First heart sound was normal and only the aortic Ref: Op Ghai Pediatrics, 8/e p445
component was audile in the second heart sound. Liver was 75. Essential criteria for TOF includes all, except: (DNB 2008)
not enlarged: (UPSC 07) a. Valvular regurgitation
What would be the likely diagnosisa. b. Infundibular stenosis
a. Congenital methemoglobinemia c. Over riding of aorta
b. Eisenmenger syndrome d. RVH Ref: OP Ghai 6/e p430
c. Aortic stenosis 76. What constitutes Pentalogy of Fallot: (DNB 2008)
d. Tetralogy of Fallot a. TOF + PDA
Ref: OP Ghai 8/e p413; 7/e p408 b. TOF + ASD
69. All of the following are true about Kawasaki disease c. TOF+ COA
except: (AI 07; AIIMS May 03) d. TOF + Polysplenia
a. Purulent conjunctivitis Ref: Ghai 8/e p542; Nelson Pediatrics 18/e p1906, 1911
b. Swelling of limb 77. Most sensitive indicator of intravascular volume in infant
c. Rash is: (DNB 2008)
d. Fever a. Cardiac output
Ref: Ghai 6/e p586; Nelson 18/e p1036-1041 and 17/e p825; b. Heart rate
Nelson 19/e p862; 864 c. Stroke volume
70. Treatment of kawasaki disease in children is: d. Preload
a. Oral steroids (AI 01) Ref: Ghai 8/e p512; CPDT 18/e p375

PEDIATRICS
b. IV steroids 78. ASD with a murmur similar to MR and LAD on ECG is
c. IV/Ig having: (DNB 2009)
d. Mycophenolate mofentil a. Ostium premium defect
Ref: Ghai 7/e p606, 6/e p586; Nelson 18/e, 1036-1041 and b. Ostium secondum defect
17/e p825; Nelson 19/e p866 c. VSD
71. A newborn presents with congestive heart failure, on d. TGA Ref: OP Ghai 8/e p510
examination has bulging anterior fontanelle with a bruit 79. Most common cause of death in PDA is: (DNB 2009)
on auscultation. Transfontanelle USG shows a hypoechoic a. Respiratory compromise
midline mass with dilated lateral ventricles. Most likely b. Cardiac compromise
diagnosis is: c. Infective endocarditis
a. Medulloblastoma (AIIMS Nov 06, AI 07) d. Embolization
b. Encephalocele Ref: OP Ghai 8/e p521; 405, 406, 407

Ans. 65. a. ACE inhibitors 66. d. Congenital heart... 67. b. Ostium secondum 68. d. Tetralogy of Fallot
69. a. Purulent... 70. c. IV/Ig 71. c. Vein of Galen... 72. A. ASD
73. b. Transposition... 74. d. Congenital heart disease 75. a. Valvular regurgitation 76. b. TOF + ASD
77. b. Heart rate 78. a. Ostium premium... 79. b. Cardiac compromise
58 Jaypees Triple A
80. Congenital long QT syndrome is associated with neonatal: 89. A new born child presented with CHD has cyanosis,
a. Sinus bradycardia (DNB 2009) become prominanat on breathing and improved in crying.
b. Sinus tachycardia The diagnosis: (MP PG 2008)
c. Supra ventricular. tachycardia a. Bilateral choanal atresia
d. Ventricular tachycardia b. Diaphragmatic hernia
Ref: 8/e p524; Campbell Text book of Peds, 10/e p589 c. Genitourinary defects
81. Drug of choice for Rheumatic fever prophylaxis in penicillin d. Coloboma Ref: Ghai 8/ep366; 7/e p336
allergic patient: (DNB 2009) 90. Average central aortic pressure in full-term neonate is
a. Erythromycin ______ mm Hg? (MHPGM-CET 2010)
b. Clindamycin a. 75/50
c. Vancomycin b. 60/40
d. Gentamycin c. 40/20
Ref: Ghai 8/e p524; Essentials of paediatrics Nelson 4/e p343-344, 590 d. 20/10
82. Essential criteria for TOF includes all, except: (DNB 2009) Ref: Nelson paediatrics 18/e p1856;
a. Valvular regurgitation OP Ghai 6/e p393
b. Infundibular stenosis 91. Atrilized Right Ventricle is a feature of: (AP 2010)
c. Over riding of aorta a. Tricuspid atresia
d. RVH Ref: OP Ghai 8/e p532 b. Ebsteins anomaly
83. Most sensitive indicator of intravascular volume in infant c. Pulmonary artesia
is: (DNB 2009) d. Fallots tetralogy Ref: Ghai 8/e p423
a. Cardiac output 92. ASD with left axis deviation: (AP 2011)
b. Heart rate a. Ostium Primum defect
c. Stroke volume b. Floppy mitral valve
d. Preload Ref: Ghai 8/e p498; CPDT 18/e p375 c. Ostium secundum
84. The commonest cause of death in diphtheric child is: d. Cleft mitral valve Ref: Ghai 8/e p413
a. IIIrd nerve palsy (DNB 2010) 93. Not true in rheumatic fever: (AP 2011)
b. Tonsilitis a. 5-15 yrs age group
c. Myocarditis b. Migrating arthritis
d. Septicemia Ref: OP Ghai 8/e p400; Nelson 18/e c. Large joints involved
85. Commonest cause of heart failure in infancy is: (DNB 2010) d. Arthritis persists for > 6 months Ref: Ghai 8/e p435
a. Myocarditis 94. A child with tetralogy of fallot uses which of the following
b. Rheumatic fever positions? (DP PGMEE 2010)
c. Cardiomyopathy a. Supine
d. Congenital heart disease b. Prone
Ref: Op Ghai Pediatrics 8/e p512 c. Squatting
86. All of the following are component of Tetralogy of Fallot d. Leaning forwards Ref: Ghai 8/e p420
(TOF) except: 95. The best position for examination of cardiac murmurs in a
a. Ventricular septal defect child is: (Feb DP PGMEE 2009)
b. Pulmonic stenosis a. Sitting
c. Overrriding or dextraposed aorta b. Standing
d. Left ventricular hppertrophy c. Right lateral
Ref: Ghai 8/e p420; Nelsons 18/e p1906 d. Recumbent Ref: Ghai 8/e p457
PEDIATRICS

87. Congenital heart disease with left ventricular hypertrophy 96. All are characteristic of fallots tetralogy, except:
is seen in: (MP PG 2008) a. Infundibular stenosis (Feb DP PGMEE 2009)
a. Tricuspid atresia b. Ventricular septal defect
b. Transpositio of great vessels c. Overriding of aorta
c. Total anomalous pulmonary venous connection d. Left ventricular hypertrophy Ref: Ghai 8/e p420
d. Ebstein anomaly Ref: Ghai 8/e p422; 7/e p411 97. Fallot tetralogy true is: (Raj PGI 2008)
88. Constituents of Tetralogy of fallot (TOF) are all except: a. Central cyanosis
a. Aortic stenosis (MP PG 2008) b. Pulmonary valvular stenosis
b. Pulmonary stenosis c. Sign symptom during newborn
c. Overrriding of aorta d. Clonidine
d. VSD Ref: Ghai 8/e p420; 7/e p408 Ref: Ghai 8/e p420

Ans. 80. a. Sinus bradycardia 81. a. Erythromycin 82. a. Valvular regurgitation 83. b. Heart rate
84. c. Myocarditis 85. d. Congenital heart... 86. d. Left ventricular... 87. a. Tricuspid atresia
88. a. Aortic stenosis 89. a. Bilateral choanal 90. a. 75/50 91. b. Ebsteins anomaly
92. a. Ostium Primum... 93. d. Arthritis persists... 94. c. Squatting 95. c. Right lateral
96. d. Left ventricular 97. a. Central cyanosis
Disorders of Cardiovascular System 59
98. A child with bullous lesion which is best test: 100. Least common cause of pericarditis/pericardial effusion in
a. VDRL of mother & child (Raj PGI 2008) children: (WB PGI 08)
b. ASLO a. Rheumatic fever
c. P24 b. Sarcoidosis
d. HLA B27 c. Rheumatoid Arthritis
Ref: Ghai 8/e p272 d. Systemic Lupus Erythematosus
99. Causes of congestive Heart failure in newborn are all Ref: Nelson 17/e p1579; Ghai 8/e p450
except: (WB PGI 08) 101. The commonest arrhythmia in children is: (J & K PGI 2010)
a. ASD a. Sinus tachycardia
b. TGA b. Supraventicular tachycardia
c. Hypoplastic left ventricle c. Sinus bradycardia
d. Baby of diabetic mother d. Atrial fibrillation
Ref: Ghai 8/e p396; DCD 6/e p287 Ref: OP Ghai 8/e p458

PEDIATRICS

Ans. 98. a. VDRL of mother... 99. a. ASD 100. b. Sarcoidosis 101. a. Sinus tachycardia
9. DISORDERS OF
RESPIRATORY SYSTEM
Disorders of Respiratory System 61

DISORDERS OF RESPIRATORY SYSTEM (QUESTIONS)

1. Wheeze in children caused by: (PGI June 05, 06) 7. Bronchiolitis in children is caused by:
a. Foreign body a. H. influenza (PGI June 05)
b. Gastroesophageal reflux disease b. RSV
c. Bronchial asthma c. Mycoplasma
d. Epiglottis d. EBV
e. Laryngomalacia e. Influenza virus
Ref: Ghai 8/e p382; 7/e p341, 347; Nelson 19/e p1431 Ref: Ghai 8/e p374; 7/e p356-357; Nelson 18/e p1773-1777 and
2. A 3-month-old child presents with intermittent stridor. 17/e p1416; Nelson 19/e p144
Most likely cause is: (AI 01, AIIMS Dec 95) 8. In Bronchiolitis followings is/are seen: (PGI Dec 02)
a. Laryngotracheobronchitis a. Seen in children 5 months to 3 years of age
b. Laryngomalacia b. Caused by streptococcus pneumoniae
c. Respiratory obstruction c. Chest X-ray shows hyperinflation bilaterally
d. Foreign body aspiration d. Symptomatic treatment is given
Ref: Ghai 8/e p371; 7/e p340; Nelson 18/e p1767; Nelson 19/e p1445, 1446 e. Antibiotics should be started
3. A child with three days history of upper respiratory tract Ref: Ghai 8/e p374-375; 7/e p357;
infection presents with stridor, which decreases on lying Nelsons Essentials of Pediatrics 5/e, p502; Nelson 19/e p1446
down postion. What is the most probable diagnosis: 9. Which of the following is/are true about bronchiolitis in
a. Acute Epiglottitis (AI 07) children: (PGI June 01)
b. Laryngotracheobronchitiff a. Caused by respiratory syncytial virus
c. Foreign body aspiration b. Hyperinflation of the chest
d. Retropharyngeal abscess c. Pleural effusion
Ref: Ghai 8/e p374; 7/e, 9 339; Nelson 18/e p1763; d. May lead to bronchial asthma later in life
Nelson 19/e p1445, 1446 e. Lymphopenia is seen
4. A 2-years-old child is brought to emergency at 3 AM with Ref: Ghai 8/e p375; 7/e p357; Nelson 19/e p1445
fever, barking cough and stridor only while crying. The 10. Which of the following is the etiological agent most often
child was able to drink normally. On examination respirator associated with Epiglottitis in children:
rate is 36/min and temperature is 39.6c. What will be your a. Streptococcus influenza
next step: b. Hemophilus influenza type b
a. Racemic epinephrine nebulization (AIIMS Nov 08) c. Neisseria sp (AIIMS May 05, Nov 04, May 94)
b. High dose dexamethasone injection d. Moraxella catarrhalis
c. Nasal wash for influenza or RS V Ref: Ghai 8/e p374; CPDT 18/e p501; Nelson 19/e p1445
d. Antibiotics and blood culture 11. A 6 months old baby coming with H/o increasing difficulty
Ref: Nelson 18/e p1765; Ghai 8/e p375; 7/e p339, 3406/e p339, 340; in breathing of 2 days duration and on examination baby is
Nelson 19/e p1446 afebrile and B/L wheeze and C X R shows B/L hyperinflation
5. A 4 year old child has seal barking like croupy cough. of the lungs with normal WBC count, the diagnosis is:
Management includes all except: (PGI June 00) a. Bronchiolitis (PGI Dec 03)
a. O2 inhalation b. Asthma
b. Antibiotic c. Ch. Bronchitis

PEDIATRICS
c. Hydration d. Pneumonia
d. Morphine e. F.b.
Ref: Ghai 8/e p374-375; 7/e p339; Nelson 19/e p1448 Ref: Ghai 8/e p374; Nelson 18/e p514; Nelson 19/e p1446
6. The most common etiological agent for acute bronchiolitis 12. A 11 months old child presents with complaints of
in infancy is: (AI 06) respiratory distress. On examination there is bilateral
a. Influenza virus crepitation and wheezing. Which of the following is the
b. Para influenza virus most likely cause: (AIIMS Nov 2K)
c. Rhinovirus a. Pneumonia
d. Respiratory syncytial virus b. Adenovirus
Ref: Ghai 8/e p374; 7/e p356; Nelson 18/e p1388-1390 and c. Respiratory synchytial virus
17/e p1416; Nelson 19/e p1446 d. Rhinovirus
Ref: Ghai 8/e p375; Nelson 17/e p1416; Nelson 19/e p1434

Ans. 1. a and c 2. b. Laryngomalacia 3. b. Laryngotracheobronchitis 4. b. High dose...


5. b and d 6. d. Respiratory... 7. b, c and e 8. c and d
9. a, b and d 10. b. Hemophilus influenza... 11. a. Bronchiolitis 12. c. Respiratory...
62 Jaypees Triple A
13. A month old HIV positive child following URTI developed 19. A 6 months old baby coming with h/o increasing difficulty
sudden onset of breathlessness. The chest X-ray shows in breathing of 2 days duration and on examination baby is
hyperinflation. The O2 saturation was greater than 90%. afebrile and B/L wheeze and CXR shows B/L hyperinflation
The treatment of choice is: (AIIMS May 01) of the lungs with normal WBC count, the diagnosis is:
a. Cotrimoxazole a. Bronchiolitis (PGI Dec 03)
b. Ribavarin b. Asthma
c. IV Ganciclovir c. Ch. Bronchitis
d. Nebulized Acyclovir d. Pneumonia
Ref: Ghai 8/e p229; 7/e p358; e. FB
Nelson 18/e p1773-1777 and 17/e p1415-1416; Nelson 19/e p1448 Ref: Ghai 8/e p375; 7/e p357; Nelson 19/e p1463
14. A child with pyoderma becomes toxic and presents with 20. A child presents raised sweat chloride levels and suspicion
respiratory distress. His chest radiograph shows patchy of cystic Fibrosis, which other test would you do to exclude
areas of consolidation and multiple bilateral thin walled air the diagnosis of cystic fibrosis:
containing cysts. The most likely etiological agent in this a. Repeat sweat chloride measurements
case is: b. Nasal electrode potential difference
a. Mycobacterium tuberculosis (AIIMS Nov 03) c. Fat in stool for next 72 hours
b. Staphylococcus aureus d. DNA analysis for delta F-508 mutation (AI 08)
c. Mycobacterium avium intracellular Ref: Ghai 8/e p393; Nelson 18/e p1808;
d. Pneumocystis carinii Nelson 19/e p1482, 1485
Ref: Ghai 8/e p376; 7/e p353; Nelson 19/e p1476 21. Features of cystic fibrosis: (PGI June 06)
15. WHO criteria for hospital admission in pneumonia: a. Lung normal at birth
a. High fever (PGI Dec 08) b. Abnormal sweat chloride tests
b. Nasal flaring c. Autosomal dominant
c. Difficulty in breathing d. Defect in chromosome
d. Difficulty in feeding Ref: Ghai 8/e p393; Nelsons Essentials of
e. Chest indrawing Pediatrics 5/e, p648; Nelson 19/e p1487
Ref: Ghai 8/e p376; Park 18/e p143 22. For a normal husband and wife the first child was diagnosed
16. Which one of the following is the leading cause of mortality to have cystic fibrosis. What is the percentage of chances for
under five years of children in developing countries? the second child be affected: (PGI June 06)
a. Malaria (AIIMS May 05) a. 25
b. Acute lower respiratory tract infections b. 50
c. Hepatitis c. 0
d. Prematurity d. 75
Ref: Ghai 8/e p376 e. 100
17. A 7 months old child with cough, mild stridor is started on Ref: Ghai 8/e p394; Nelsons
oral antibiotics. The child showed initial improvement but Essentials of Pediatrics 5/e p224
later developed wheeze, productive cough, and mild fever. 23. Following are true about bronchial cyst except:
X-ray shows hyperlucency and PFT shows an obstructive a. Mostly mediastinal (AIIMS 96)
curve. The most probable diagnosis is: (AI 08) b. 50-70% occur in lungs
a. Bronchiolitis obliterans c. Usually multiloculated
b. Post viral syndrome d. Are infected quite often
c. Pulmonary alveolar microlithiasis Ref: Schwartz 8/e p597;
PEDIATRICS

d. Follicular bronchitis Nelson 19/e p1466; Internet


Ref: Ghai 8/e p374; Nelson 19/e p1463 24. A 4-year-old child presents with a history of chronic left
18. A 7-year-old girl presents with breathlessness and fever lower lobe pneumonitis. On contrast bronchography, the
for 6-7 days. She has non-productive cough for 6 months. area involved with the pneumonitis does not fill whereas the
X-ray chest shows hyperlucency in the lungs and pulmonary area around it does fill. The most likely diagnosis: (AI 06)
function tests show obstructive pattern. The most probable a. Asthma
diagnosis will be: (AIIMS Nov 08) b. Pulmonary sequestration
a. Lobar emphysema c. Cystic fibrosis
b. Bronchiolitis obliterans d. Bronchopulmonary dysplasia
c. Follicular bronchitis e. Bronchogenic cyst
d. Pulmonary alveolar microlithiasis Ref: Ghai 8/e p372; Nelson 18/e p1784;
Ref: Ghai 8/e p374; Nelson 19/e p1463 Nelson 19/e p14650

Ans. 13. b. Ribavarin 14. b. Staphylococcus aureus 15. All of the above 16. b. Acute lower...
17. a. Bronchiolitis... 18. a and b 19. a. Bronchiolitis 20. b. Nasal electrode...
21. a and b 22. c. 0 23. b. 50-70% occur in lungs 24. b. Pulmonary sequestration
Disorders of Respiratory System 63
25. Which of the statements is not true regarding Macleods 32. Commonest cause of stridor in a new born is: (DNB 2005)
Syndrome: (AI 07) a. Laryngomalacia
a. It is not a true emphysema b. Foreign body
b. Occurs before 8 years of age c. Meconium aspiration
c. It is unilateral emphysema d. Recurrent laryngeal nerve palsy due to birth
d. The pulmonary artery on the affected side is hyperplastic Ref: Ghai 8/e p371; Nelson Pediatrics 18/e p1767
e. It is also called Swyer - James Syndrome Ref: Internet 33. In a prematurely new borne child with respiratory distress
26. Pneumothorax could be a complication of: with grunting after 2 hours. Respiratory management
a. Staphylococcal pneumonia (UPSC 2K) would involve: (DNB 2007)
b. Pneumococcal pneumonia a. Nasal CPAP
c. Klebsiella pneumonia b. Surfactant with mechanical ventilation
d. Viral pneumonia Ref: Ghai 8/e p376; 7/e p352, 6/e p349 c. Intubation
27. Child requiring repeated short acting broncho-dilators and d. Warm oxygen
what could be next line of management: Ref: Nelsons. 18/e p731-5, Manual of Neonatal Care, 6/e p326-7,
a. Methylxanthines (Manipal 06) OP Ghai 8/e p137; 7/e p144
b. Short acting budesonide 34. Most common cause of Acute Epiglottitis is: (DNB 2010)
c. Oral prednisolone a. S. pneumoniae
d. Montelukast b. Influenza
Ref: Ghai 8/e p382; 7/e p361; Nelson 17/e p767, 768 c. H. influenza
28. The correct line of management in child who has swallowed d. Meningococci Ref: Nelson 18/e p1173, 1174
a coin is: (TNPSC 2K) 35. Commonest cause of stridor in a new born is: (DNB 2010)
a. Fibreoptic endoscopy a. Laryngomalacia
b. Rigid endoscopy b. Foreign body
c. Laparotomy c. Meconium aspiration
d. Wait and watch Ref: Ghai 8/e p391; Nelson 17/e p1226 d. Recurrent laryngeal nerve palsy due to birth
29. With references to bronchiolitis, consider the following Ref: Ghai 8/e p371; Nelson Pediatrics 18e p1767
statement: (UPSC 07) 36. A premature baby weighing 1.5 kg, born with emergnancy
1. Is more common among boys Cesarean Section at 32 weeks, now develops respiratory
2. RSV immunoglobulin does not have any role in acute distress with grunting the best management would be:
episode a. C-pap (DNB 2010)
3. It is common around the age of six months b. Mechanical ventilation
4. Mortality is high in it Which ofthe statements given c. Moist oxygen through headbox
above are correct d. Surfactant therapy plus mechanical ventilation
a. 1 and 2 only Ref: Ghai 8/e p137; Campbells Textbook of Pediatrics, 10/e p177,
b. 3 and 4 only OP; Ghai, 6/e p167, Nelson Pediatrics
c. 1, 2 and 3 only 37. Lung maturity is best assessed by: (DNB 2011)
d. 1, 2, 3 and 4 a. L:S ratio
Ref: CPDT 17/e p506; O.P. Ghai 8/e p374; 7/e p357 b. Phosphatidyl glycerol estimation
30. A child presented with mild fever and breathlessness. c. Bilirubin
He was treated and his condition improved over 4 days. d. Lecithin
Later his condition deteriorated with increase in fever and Ref: Dutta obs 5/e p324; Williams OBS, 22/e Chapter 29
breathlessness. X-ray showed hyperlucency. What is the

PEDIATRICS
38. True about Alpha 1 antitrypsin deficiency is: (DNB 2011)
probable diagnosis? a. X linked dominant
a. Bronchiolitis obliterans b. Does not cause pulmonary disease
b. Alveolar proteinosis c. Is synthesized in liver
c. Bronchitis d. Causes decreased sodium in sweat.
d. Bronchiolitis Ref: Harrison Medicine, Nelson; Internet
Ref: Ghai 8/e p374; Nelson text book of pediatrics 18/e p1781-82
39. Most common cause of inspiratory stridor in a newborn is:
31. The most important cause of under 5 mortality is: a. Subglottic stensosis (MP PG 2010)
a. Diarrhoea (DNB 2005) b. Tracheo-oesophageal fistula
b. Malnutrition c. Laryngomalacia
c. Respiratory infections d. Dysphagic Lusoria
d. Trauma Ref: O.P. Ghai Peds 8/e p376 Ref: Ghai 8/e p368-372; Nelsons 18/e p1767

Ans. 25. d. The pulmonary... 26. a. Staphylococcal... 27. b. Short acting... 28. a. Fibreoptic endoscopy
29. c. 1, 2 and 3 only 30. a. Bronchiolitis obliterans 31. c. Respiratory infections 32. a. Laryngomalacia
33. b. Surfactant with... 34. a. S. pneumoniae 35. a. Laryngomalacia 36. d. Surfactant therapy...
37. a > b 38. c. Is synthesized... 39. c. Laryngomalacia
64 Jaypees Triple A
40. The commonest cause bronchiolitis in children is: c. Bronchopneumonia
a. Adenovirus (MP PG 2010) d. Retrophyryngeal abscess
b. Respiratory syncytial virus Ref: Ghai 8/e p711; Nelson 18/e p1769-1770
c. Klebseilla 45. A 3-months-old infant presents with intermittent respiratory
d. Staphyllococci Ref: Ghai 8/e p387; Nelsons 18/e p1774 stridor since 10 days of age. The most likely diagnosis is:
41. Pneumatoceles on chest radiograms in a child with a. Laryngomalacia (Feb DP PGMEE 2009)
pneumonia are seen in infection: (Karnataka PG 2011) b. Tracheoesophageal fistula
a. Staphylococcus & Klebsiella c. Laryngotracheobronchitis
b. Pneumococcus d. Neoplasm Ref: Ghai 8/e p368
c. Streptococcus 46. Most common site of extra pulmonary TB is seen in:
d. Hemophilus influenza a. Lymph node (Raj PG 2008)
Ref: OP Ghai 8/e p377; 7/e p353; Harrisons, 17/e p876,1622 b. Pleura
42. Gastrointestinal disturbance in cystic fibrosis are all except: c. Meningitis
a. Spleen infact (UP PG 09) d. Vertebra Ref: Ghai 8/e p250
b. Biliary cirrhosis 47. 12-year-old boy came with fever, sore throat and cervical
c. Malabsorption lymph node enlargement. Throat swab positive for group A
d. Gallstones beta haemolytic streptococci and started on penicillin. But
Ref: Ghai 8/e p393; 7/e p369-70; Nelson 18/e p1803 he came back with worsening of symptoms even while on
43. Rediological features of transient tachypnea of newborn is: treatment. Next step is: (Kerala PG 09)
a. Irregular pulmonary vascular marking (UP PG 09) a. Start azithromycin
b. Low lung volumes b. Blood monospot test and CBC
c. Air bronchogram sign c. IgE for IMN
d. Horizontal fissure d. Presume it as EBV and start acyclovir and prednisolone
Ref: Ghai 8/e p171; Nelson 18/e p741 Ref: Ghai 8/e p215; Nelson 18/e p1375
44. A 4-year-old child presents with a history of hoarseness, 48. Streptococcal pharyngitis is treated for: (J & K PG 2011)
croupy couth and aphonia, the child has dyspnoea with a. 3 days
wheezing. The most probable diagnosis is: b. 5 days
a. Asthmatic bronchitis (DP PGMEE 2010) c. 7 days
b. Laryngeal foreign body d. 10 days Ref: Ghai 8/e p374
PEDIATRICS

Ans. 40. b. Respiratory... 41. a. Staphylococcus... 42. a. Spleen infact 43. d. Horizontal fissure
44. b. Laryngeal foreign... 45. a. Laryngomalacia... 46. a. Lymph node 47. b. Blood monospot...
48. d. 10 days
10. DISORDERS OF KIDNEY AND
URINARY TRACT
66 Jaypees Triple A

DISORDERS OF KIDNEY AND URINARY TRACT (QUESTIONS)

1. True about infantile polycystic kidney disease include the mEq/L, potassium 4 mEq/L, calcium 7 mg/dl, phosphate 6
following except: (AI 08) mg/dl and alkaline phosphatase 400 U/l. Urinalysis shows
a. Autosomal dominant trace proteinuria with hyaline casts; no red and white cells
b. Hepatic cysts are seen. Ultrasound shows bilateral small kidneys and the
c. Renal cysts present at birth micturating cystourethrogram is normal. The most likely
d. Periportal fibrosis diagnosis is:
Ref: Ghai 8/e p464; 7/e p471; a. Alports syndrome [AIIMS Nov. 03]
Nelson 17/e p1749; Nelson 19/e p1796 b. Medullary sponge kidney
2. Which of the following is the most common renal cystic c. Chronic glomerulonephritis
disease in infant? (AI 05) d. Nephronophthisis
a. Polycystic kidney Ref: Harrisons 18/e p1799; Robbins 6/e p965, 966;
b. Simple renal cyst Nelson 19/e p1816
c. Unilateral renal dysplasia 7. A 13-year-old boy is referred for evaluation of nocturnal
d. Calyceal cyst enuresis and short stature. His blood pressure is normal.
Ref: Ghai 8/e p464-466; 7/e p470;
The hemoglobin level is 8g/dl. urea 112 mg/dl, creatinine
Nelson 17/e p1784; Nelson 19/e p1827
6 mg/dl, sodium 119 mEq/ dl, potassium 4 mEq/L, calcium
3. Which one of the following is the most common cause of 7 mg/dl, phosphate 6 mg/dl and alkaline phosphatase 300
abdominal mass in neonates? (AI 03) U/l. Urinalysis shows trace proteinuria with hyaline casts;
a. Neuroblastoma no red and white cells are seen. Ultrasound shows bilateral
b. Wilms tumor small kidneys and the micturating cystourethrogram is
c. Distended bladder normal. The most likely diagnosis is: (AIIMS May 03)
d. Multicystic dysplastic kidneys
a. Alports syndrome
Ref: Ghai 8/e p507; 7/e p470;
b. Medullary sponge kidney
Nelson 17/e p1784; Nelson 19/e p1827
c. Chronic glomerulonephritis
4. A 6-years-old girl presents with recurrent e. coli infection d. Nephronophthisis
in urine. Ultrasound of abdomen shows Hydroureter and Ref: Harrisons 18/e p1799; Robbins 6/e p965, 966
Hydronephrosis. Micturating cystourethrogram shows
8. An 8-year-old child suffering from recurrent attacks of
filling defect in urinary bladder. The likely diagnosis is:
polyuria since childhood presents to the pediatrics OPd.
a. Sacrococcygeal Teratoma (AI 00)
On examination the child is short statured vitals and BP are
b. Vesicoureteric Refluxgrade II
c. Duplication of Ureter normal. Serum Creatinine - 6 mg%, HCO3 - 16 mEq, Na- 134,
d. Ureterocele IC4.2 On USG bilateral small kidneys are seen. Diagnosis
Ref: Ghai 8/e p505; Nelson 17/e p1799-1800; Nelson 19/e p1843 is:
a. Reflux nephropathy (AIIMS May 01)
5. A 6-months-old boy weighing 3.2 kg presents with recurrent
b. Nephronophthisis
vomiting and polyuria. Investigations show blood area 60
c. Polycystic kidney disease
mg/dl, creatinine 0.7 mg/dL, calcium 12.8 mg/dL, phosphate
d. Medullary cystic kidney disease
PEDIATRICS

3 mg/dL, pH 7.45, bicarbonate 25 mEq/L and PTH 140 pg


Ref: Harrisons 18/e p1799; Robbins 6/e p965, 966
ml (normal <60 pg/ml). Daily urinary calcium excretion is
reduced. Ultrasound abdomen show bilateral: 9. A 5-year-old child suffering from nephrotic syndrome is
a. Bartter syndrome (AIIMS Nov 05) responding well to steroid therapy. What would be the most
b. Mutation of the calcium sensing receptor likely finding on light microscopy: (AI 01)
c. Pseudo-pseudohypoparathyroidism a. No finding
d. Parathyroid adenoma b. Basement membrane thickening
Ref: Nelson 17/e p1895; Harrison 16/e p2256; Nelson 19/e p1816 c. Hypercellular glomeruli
6. A 12-years-old boy is referred for evaluation of nocturnal d. Fusion of foot processes
enuresis and short stature. The blood pressure is normal. Ref: Ghai 8/e p477; 7/e p451; Nelson 17/e p1754-1755;
The blood urea is 112 mg/dl, creatinine 6 mg/dl, sodium 119 Nelson 19/e p1804

Ans. 1. a. Autosomal dominant 2. c. Unilateral renal... 3. d. Multicystic dysplastic... 4. d. Ureterocele


5. b. Mutation of the... 6. d. Nephronophthisis 7. d. Nephronophthisis 8. b. Nephronophthisis
9 a. No finding
Disorders of Kidney and Urinary Tract 67
10. The most common gene defect in ldiopathic steroid resistant a. One year of age
nephrotic syndrome: b. Eighteen months of age
a. ACE (AIIMS Nov 06; May 07) c. Three to six months of age
b. NPHS 2 d. Just before puberty
c. HOX 11 Ref: Ghai 7/e p441-442; Nelson 17/e p1790-1795
d. PAX 17. Which one of the following statements is false with regard
Ref: Ghai 8/e p477; Robbins 6/e p983, 984; to pyuria in children: (AI 03)
Nelson 19/e p1803 a. Presence of more than 5 WBC/hpf (high power field) for
11. A 9-year-old boy has steroid dependent nephrotic syndrome girls and more than 3 WBC/hpf for boys
for the last 5 years. The patient is markedly cushingoid with b. Infection can occur without pyuria
blood pressure of 120/ 86 mmHg and small subcapsular c. Pyuria may be present without Urinary tract infection
cataracts. The most appropriate therapy of choice is: d. Isolated pyuria is neither confirmatory nor diagnostic for
a. Long-term frusemide with enalapril (AIIMS Nov 04) Urinary tract infection
b. Cyclophosphamide Ref: Ghai 8/e p483; 7/e p456; Nelson 17/e p1787
c. Intravenous immunoglobulin 18. A 2 month old girl has failure to thrive, polyuria and
d. Intravenous pulse corticosteroids medullary nephrocalcinosis affecting both kidneys.
Ref: Ghai 8/e p479; Nelson 17/e p1756; Nelson 19/e p1805 Investigations show blood pH 7.48, bicarbonate 25 mEq/L,
12. A 9-year-old child has steroid dependant nephrotic potassium 2 mEq/L, sodium 126 mEq/L and chloride 88
syndrome for the last 5 years. He has received corticosteroids mEq/L. The most likely diagnosis is:
almost continuously during this period and has cushingoid a. Distal renal tubular acidosis (AIIMS Nov 04)
features. The blood pressure is 120/86 mmHg and there are b. Primary hyperaldosteronism
bilateral subcapsular cataracts. The treatment of choice is: c. Bartter syndrome
a. Levamisole (AIIMS May 03) d. Pseudohypoaldosteronism
b. Cyclophosphamide Ref: Ghai 8/e p505; Nelson 17/e p17631764;
c. Cyclosporin A Nelson 19/e p1813
d. Intravenous pulse corticosteroids 19. A 10-month-old boy, weighing 3 kg has polyuria, polydipsia
Ref: Ghai 8/e p479; and delayed motor milestones. Investigations show blood
Nelson 17/e p1756; Nelson 19/e p1805 levels of creatinine 0.5 mg/dl, potassium 3 mEq/L, sodium
13. Nephrotic syndrome in children is caused by: 125 mEq/L, chloride 88 mEq/L, calcium 8.8 mg/ dl, pH
a. Minimal change disease (PGI Dec 08) 7.46 and bicarbonate 26 mEq/L. Ultrasonography shows
b. RPGN medullary nephrocalcinosis. The most likely diagnosis is:
c. MPGN O. FSGS a. Renal tubular acidosis (AIIMS Nov 03)
d. Membranous nephritis b. Diabetes insipidus
Ref: Ghai 8/e p477; 7/e p451-453; c. Bartter syndrome
Nelson 18/e p2191, 2192; Nelson 19/e p1803 d. Pseudohypoaldosteronism
14. True about Nephrotic syndrome in a Child: (PGI June 03) Ref: Ghai 8/e p505; Nelson 17/e p1756;
a. Minimal Change disease is commonest cause Nelson 19/e p1813
b. Proteinuria of 4 gm/m2/hour is characteristic 20. 2-year-old child presented with sudden onset of altered
c. Cyclosporin and Azathioprine is mainstay of therapy sensorium on examination BP was 200/100:
d. Pretreatment biopsy is done in all cases a. Renal artery stenosis (PGI Dec 04)
e. Spontaneous bacterial peritonitis is associated with it b. Coarctation of Aorta
Ref: Ghai 8/e p477-478; 7/e p451; Nelson 17/e p1753 1756; c. Glomerulonephritis

PEDIATRICS
Harrison 16/e p1684-1686; Nelson 19/e p1803 d. Essential hypertension
15. Which of the following is included in definition of e. Pheochromocytoma
Nephrotic syndrome: (PGI June 04) Ref: Ghai 6/e p432-434;
a. Microalbuminuria Nelson 17/e p1592-1594; Nelson 19/e p1785
b. Masisve proteinuria 21. The finnish type of congenital nephritic syndrome occurs
c. Microscopic hematuria due to gene mutation affecting the following protein:
d. Edema a. Podocin (AI 06)
e. Hyperlipidemia b. Alpha-actinin
Ref: Ghai 8/e p478; Robbins 7/e p450; c. Nephrin
Nelson 18/e p2192; Nelson 19/e p1801, 1802 d. CD2 activated protein
16. The neonatal kidney achieves concentrating ability Ref: Ghai 8/e p477; Nelson 18/e p2192, 2195; Nelson 19/e p1803
equivalent to adults kidney by: (AI 04)

Ans. 10. b. NPHS 2 11. b. Cyclophosphamide 12. b. Cyclophosphamide 13. a, b and d


14. a and e 15. b, d and e 16. a. One year of age 17. a. Presence of more...
18. c. Bartter syndrome 19. c. Bartter syndrome 20. c and e 21. c. Nephrin
68 Jaypees Triple A
22. A female child was brought with complaint of generalized c. Start Cyclosporine
swelling of her body. She was passing cast in her urine. No d. Urinary bag sample
hematuria. The true statement is: (AI 09) Ref: Ghai 8/e p476; 7/e p446; Nelson 17/e p1737-1738;
a. No IgG or C3 deposition seen on renal biopsy Harrison 16/e p1690; Nelson 19/e p1782
b. Her C3 levels could be low 29. 12-year-of old Shyam presented with gross hematuria
c. IgA nephropathy is likely with 80% dysmorphic RBCs days after a attack of upper
d. Alport syndrome respiratory tract infection diagnosis is: (AIIMS Nov 01)
Ref: Ghai 8/e p479; 7/e p451; a. Microangiopathic thrombotic anemia
Nelson 17/e p1737-38 b. IgA Nephropathy
23. A 3-year-old male had non blanching rashes over the shin c. PSGN
and swelling of knee joint with hematuria +++ and protein d. HS purpura Ref: Robbins 7/e p986987; Ghai 8/e p476;
+. Microscopic analysis of his renal biopsy specimen is Nelson 17/e p1737-1738; Nelson 19/e p1781
most likely to show? 30. A six year old male baby presents to a hospital with
a. Tubular necrosis (AIIMS Nov 08) recurrent gross hematuria for 2 years. There is no h/o
b. Visceral podocyte fusion burning micturition or pyuria. Urine routine examination
c. Mesangial deposits of IgA demonstrated no pus cells and urine culture was sterile.
d. Basement membrane thickening Serum C3 levels were normal. What is the most probable
Ref: Ghai 8/e p477; Nelson 17/e p1737-1738; diagnosis? (AI 08)
Nelson 19/e p1781 a. Wilms tumor
24. A child was diagnosed as a case of pauci-immune crescentic b. IgA nephropathy
glomerulonephritis. The treatment to be given in this child c. Post-streptococcal glomerulonephritis
is: (AIIMS May 02) d. Urinary tract infection
a. Prednisolone + Cyclophosphamide Ref: Ghai 8/e p476; Other characteristics explained already
b. Methylprednisolone 31. Henoch-Schonlein purpura is characterized by the
c. Cyclophosphamide deposition of the following immunoglobulin around the
d. Immunoglobins vessels: (AIIMS Nov 05)
Ref: Ghai 8/e p474; Harrison 16/e p1683-1684 a. IgM
25. Post-streptococcal glomerulonephritis is associated with: b. IgG
a. Follows skin and throat infection (PGI Dec 01) c. IgA
b. Antibiotic treatment induces remission d. IgE Ref: Ghai 8/e p632; Nelson 18/e p2178, 2179;
c. Is a cause of chronic renal failure in majority of children Robbins 7/e p650, 986, 987; Nelson 19/e p1789
d. Low complement level occurs 32. A 8 year old male had non blanching rashes over the shin
e. Caused by all serotypes and swelling of knee joint with hematuria +++ and protein
Ref: Ghai 8/e p474; 7/e p447, 6/e p447-448 +. Microscopic analysis of his renal biopsy specimen is
26. Child with BP 190/110, pedal edema ++ ; facial edema ascites most likely to show:
-absent. Gross hematuria diagnosis is: (PGI June 00) a. Tubular necrosis (AIIMS Nov 07)
a. Acute GN b. Visceral podocyte fusion
b. Nephrotic syndrome c. Mesangial deposits of IgA
c. Renal thrombosis d. Basement membrane thickening
d. Renal amyloidosis Ref: Ghai 8/e p474-476; 7/e p447 Ref: Ghai 8/e p476, 473; Harrison 17/e, p2128; Nelson 19/e p1781
27. A 6-year-old child presents with recurrent episodes of gross 33. A picture resembling microthrombin in renal vasculature
hematuria for 2 yrs. He is likely to have: occurs in: (PGI Dec 01)
PEDIATRICS

a. IgA nephropathy (AI 08) a. Pre eclampsia


b. Wilms tumor b. Henoch Schonlein
c. Henoch Schonlein Purpura (HSP) c. Benign hypertension
d. Neuroblastoma d. Hemolytic uremic syndrome
Ref: Ghai 8/e p476; 7/e p446; Nelson 18/e p2169, 2171 Ghai, e. Malignant hypertension
6/e p446; Nelson 19/e p1781 Ref: Ghai 8/e p492; Robbins 6/e p1008; Dutta 6/e p223
28. The renal biopsy of a 6 year old boy with recurrent gross 34. Most common cause of urinary obstruction in a male infant is:
hematuria shows IgA nephropathy. The urinary protein a. Anterior urethral valves (AI 01)
excretion is 130 mg/day. Which of the following is the most b. Posterior urethral valves
appropriate next step in the management: (AIIMS May 04) c. Stone
a. Administer corticosteroids d. Stricture
b. Give Azathioprine Ref: Ghai 8/e p465; 7/e p470; Nelson 17/e p1802-1803; Nelson 19/e p1845

Ans. 22. a. No IgG or C3... 23. c. Mesangial deposits... 24. a. Prednisolone... 25. a and d
26. a. Acute GN 27. a. IgA nephropathy 28. a. Administer... 29. b. IgA Nephropathy
30. b. IgA nephropathy 31. c. IgA 32. c. Mesangial deposits... 33. a, d and e
34. b. Posterior urethral...
Disorders of Kidney and Urinary Tract 69
35. A 5-year-old child presents with a calculus of size 2 cm in a. Acute glomerulonephritis
the upper ureter. He also complains of hematuria. USG b. Urinary tract infection
shows no further obstruction in the urinary tract. TT of c. Posterior urethral valves
choice for this patient would be: (AI 01) d. Teratoma
a. Ureterolithotomy Ref: Ghai 8/e p465; 7/e p470;
b. Endoscopic removal Nelson 17/e p1802-1803; Nelson 19/e p1845
c. ESWL 42. The treatment of choice for primary grade V vesicoureteric
d. Observation Ref: Ghai 8/e p468; 7/e p469 reflux involving both kidneys in a 6-month-old boy is:
36. A 3-year-old boy has bilateral renal calculi secondary a. Antibiotic prophylaxis (AIIMS Nov 04)
to idiopathic hypercalciuria. The dietary management b. Ureteric reimplantation
includes all of the following except: c. Cystoscopy followed by subureteric injection of Teflon
a. Increased water intake (AIIMS Nov 04) d. Bilateral ureterostomies
b. Low sodium diet Ref: Ghai 8/e p462; 7/e p458; Nelson 17/e p1793
c. Reduced calcium intake 43. Vesicoureteric reflux is more common in:
d. Avoidance of meat proteins a. Newborn females (AIIMS May 04)
Ref: Ghai 8/e p464; 7/e p446; Nelson 17/e p1748 b. Older girls
37. A child presents with abdominal colic and hematuria. On c. Older boys
ultrasonography a stone 2.5 cm in diameter is seen in the d. Only during pregnancy
renal pelvis. The next step in management of this case is: Ref: Ghai 8/e p462; 7/e p457-458;
a. Pyelolithotomy (AIIMS Nov 00) Nelson 17/e p1790-1791
b. Nephroureterostomy 44. The most common underlying anomaly in a child with
c. Conservative recurrent urinary tract infections is:
d. ESWL Ref: Ghai 8/e p478; 7/e p469 a. Posterior urethral valves (AIIMS Nov 03)
38. A child with recurrent urinary tract infections is most likely b. Vesicoureteric reflux
to show: (AI 05) c. Neurogenic bladder
a. Posterior urethral valves d. Renal calculi
b. Vesicoureteric reflux Ref: Ghai 8/e p462; Nelson 17/e p17901791;
c. Neurogenic bladder Nelson 19/e p1834, 1836
d. Renal and ureteric calculi 45. One year old male child presented with poor urinary stream
Ref: Ghai 8/e p467; 7/e p457-458; Nelson 17/e p1790-1791; since birth. The investigation of choice for evaluation is:
Nelson 19/e p1834, 1835 a. Voiding cystourethrography (VCUG) (AIIMS May 03)
39. The most common cause of renal scarring in a 3 year old b. USG bladder
child is: (AI 05) c. Intravenous urography
a. Trauma d. Uroflowmetry
b. Tuberculosis Ref: Ghai 8/e p467; 7/e p457-458;
c. Vesicoureteral reflux induced pyelonephritis Nelson 17/e p1802-1803; Nelson 19/e p1836
d. Interstitial nephritis 46. A boy suffering from acute pyelonephritis, most specific
Ref: Ghai 8/e p462; Nelson 17/e p1792; urinary finding will be: (AIIMS Nov 06)
Nelson 19/e p1834, 1836 a. Leukocyte esterase test
40. A 6-months-old girl is having recurrent UTI. Ultrasound b. WBC casts
abdomen shows bilateral hydronephrosis. MCU c. Nitrite test
(Micturating cysto urethrogram) shows bilateral Grad IV d. Bacteria in gram stain

PEDIATRICS
vesicoureteral reflux. The treatment of choice is: (AI 02) Ref: Ghai 8/e p473; Nelson 17/e p1787;
a. Endoscopic injection of polyteflon at the ureteric orifices Nelson 19/e p1829, 1830
b. Ureteric reimplantation 47. Which of the following is the most appropriate method for
c. Bilateral ureterostomy obtaining a urine specimen for culture in an 8 month old
d. Prophylactic antibiotics girl: (AIIMS May 04)
Ref: Ghai 8/e p462; 7/e p458; a. Suprapubic aspiration
Nelson 17/e p1793 b. Indwelling catheter sample
41. A 3-year-old boy presents with fever; dysuria and gross c. Clean catch void
hematuria. Physical examination shows a prominent d. Urinary bag sample
suprapubic area which is dull on percussion. Urinalysis Ref: Ghai 8/e p467; 7/e p456; Nelson 17/e p1786-1787;
reveals red blood cells but no proteinuria. Which of the Nelson 19/e p1831-1832
following is the most likely diagnosis? (AIIMS May 06)

Ans. 35. c. ESWL 36. c. Reduced calcium... 37. d. ESWL 38. b. Vesicoureteric reflux
39. c. Vesicoureteral... 40. d. Prophylactic... 41. c. Posterior urethral... 42. a. Antibiotic prophylaxis
43. a. Newborn females 44. b. Vesicoureteric reflux 45. a. Voiding... 46. d. Bacteria in gram...
47. a. Suprapubic aspiration
70 Jaypees Triple A
48. An 8 year old boy during a routine check up is found to d. Malignant hypertension
have E. coli 1,00,000 cc/ml on a urine culture. The urine e. Hemolytic uremic syndrome
specimen was obtained by mid-stream cleancatch void. The Ref: Ghai 8/e p487; Nelsons Essentials of Pediatrics 5/e, p760;
child is asymptomatic. Which is the most appropriate next Nelson 19/e p1819
step in the management: 54. Which one of the following statements is false with regard
a. Treat as an acute episode of urinary tract infection to Xanthogranulomatous pyelonephritis in children:
b. No therapy (AIIMS May 04) a. Often affects those younger than 8 years of age (AI 03)
c. Prophylactic antibiotics for 6 months b. It affects the kidney focally more frequently than diffusely
d. Administer long-term urine alkalinizer c. Boys are affected more frequently
Ref: Ghai 8/e p463; 7/e p456; Nelson 17/e p1787; d. Clinical presentation in children is same as in adults
Harrison 16/e p1718; Nelson 19/e p1831 Ref: Ghai 8/e p474; Internet reference
49. A five year old male child presents with complaints of fever 55. Malformations of the following organ system of the fetus
and abdominal distension. He is having vomiting for the are found to be most commonly associated with single
last five days. On examination there are 6-8 Pus Cell/hpf in umbilical artery:
urine. WBC count shows 78% neutrophils. What is the best a. Central nervous system (SGPGI 05)
line of management: b. Cardiovascular
a. Send urine for culture and sensitivity and wait for c. Genitourinary
results. (AIIMS Nov 00) d. Skeletal Ref: Ghai 8/e p465; Nelson 17/e p608
b. Send urine for culture and sensitivity and start IV
56. Potters syndrome is associated with:
antibiotics immediately
a. Renal anomalies (MAHE 05)
c. Send urine for culture, do an USG and start chloroquine
b. Severe oligohydramnios
d. Radio nucleotide studies
c. Flattened nose
Ref: Ghai 8/e p463; Nelson 19/e p1830, 1831
d. All the above Ref: Nelson 17/e p1783; Internet
50. UTI in infant, true about: (PGI Dec 03)
57. Most common infection ina child nephritic syndrome:
a. Common in female infants
a. Spontaneous bacterial peritonitis (MAHE 07)
b. If two episodes of UTI in females of 7 years occur, then
b. Pneumonia
cystometric evaluation needed
c. UTI
c. If two episodes of UTI in male of 5 year occur then
d. Cellulitis Ref: OP Ghai 8/e p477; 7/e p453
cystometric evaluation needed
Ref: Ghai 8/e p463-465; Nelson 17/e p17851788; 58. Straining and dribbling of urine in a male infant with
Nelson 19/e p1832, 1833 recurrent urinary infection should lead to the suspicion of:
a. Vesico-ureteric reflux (UPSC 07)
51. In Schwartz formula for calculation of creatinine clearance
b. Posterior urethral valve
in a child, the constant depends on the following except:
c. Pelvic ureteric junction obstruction
a. Age (AIIMS Nov 06)
d. Phimosis
b. Method of estimation of creatinine
Ref: OP Ghai 8/e p465; 7/e p467
c. Mass
d. Severity of renal failure 59. A child presents with failure to thrive with frequent
Ref: Ghai 8/e p487, 493; Nelson 19/e p1819, 1823 vomiting, diarrhea, hepatosplenomegaly and abdominal
distension. CT shows adrenal calcification. Which of the
52. An 8 day old breast fed baby presents with vomiting, poor
following is probable the diagnosis? (AIIMS Nov 09)
feeding and loose stools. On examination the heart rate is
a. Adrenal hemorrhage
190/minute, blood pressure 50/30 mm Hg, respiratory rate
b. Woman disease
72 breaths/minute and capillary refill time of 4 seconds.
c. Pheochromocytoma
PEDIATRICS

Investigations show hemoglobin level of 15g/dl, Na 120


d. Addisons disease
mEq/L, K+ 6.8 mEq/L, bicarbonate 15 mEq/L, urea 30 mg/dL
Ref: Ghai 8/e p515; Current Diagnosis and Treatment
and creatinine 0.6 mg/dL. The most likely diagnosis is:
Pediatrics 19/e (2009)); Nelson 19/e p479, 1926
a. Congenital adrenal hyperplasia (AIIMS Nov 04)
b. Acute tubular necrosis 60. Child presented with decreased urination, blood urea-108
c. Congenital hypertrophic pyloric stenosis and creatinine-3.2, Ca-12.5 . Which of the following is least
d. Renal tubular acidosis appropriate line of management?
Ref: Ghai 8/e p487; Nelson 19/e p1819 a. N/2 saline
b. Injection furosemide
53. Renal causes of acute renal failure include:
c. Bisphosphonates
a. Minimal change disease (PGI Dec 01)
d. Hemodialysis
b. Renal amyloidosis
Ref: Nelson, Textbook of Pediatrics, 18/e p2208-9; Ghai 8/e p487
c. Pre-eclampsia

Ans. 48. a. Treat as an acute... 49. b. Send urine... 50. All of the above 51. d. Severity of renal...
52. b. Acute tubular... 53. e. Hemolytic uremic... 54. a. Often affects those... 55. c. Genitourinary
56. d. All the above 57. a. Spontaneous... 58. b. Posterior urethral... 59. b. Woman disease
60. c. Bisphosphonates
Disorders of Kidney and Urinary Tract 71
61. Eight-year-old boy presented with BP 180/110, Hb 9.6 gm, 69. Which of the following bacteria is most commonly
urea 120, creatinine 5.3, urinalysisRBC 1-2/ hpf, WBC ++, responsible for peritionitis in nephrotic syndrome:
protein urea +, no previous history of medical illness: a. Staphylococcus (MP PG 2008)
a. Postinfectious GN b. Pneumooccus
b. RPGN c. Hemophillus influenza type b
c. Accelerated HT with ARF d. Salmonella
d. Chronic interstitial nephritis with reflux Ref: Ghai 8/e p477; Bailey and Loves 25/e p999
Ref: Ghai 8/e p474 70. In management of UTI (Urinary tract infection) a VCUIG
62. PKU first line therapy based on: (voiding cystourethrogram) is always indicated in:
a. Replacing the deficient product a. Asymptomatic bacteriuria (MP PG 2008)
b. Avoiding substrate for the enzyme b. Any child with febrile UTI
c. Replacement of the enzyme c. School age girl with single UTI
d. Administration of the tetrahydrobiopterin d. All children 2-5 years of age
Ref: Ghai 8/e p655; Nelson. Textbook of Pediatrics, 18/e p530.56 Ref: Ghai 8/e p483; Nelsons 18/e p2227
63. A 3-month-old male child with normal genitilia presents 71. Renal papillary necrosis occur with: (MP PG 2008)
to the emergency department with severe dehydration, a. Pernicious anemia
hyperkalemia, and hyponatremia. Measurement of which b. Sickel cell anemia
among the following is most helpful? c. Aplastic anemia
a. 17-hydroxy progesterone d. Sideroplastic anemia
b. Renin Ref: Ghai 8/e p344; Harrisons 16/e p1825; Pediatric up 2011
c. Cortisol 72. A child has nephrotic syndrome presented with fever and
d. Aldosterone sudden painful lump in the abdomen diagnosis is:
Ref: Ghai 7/e p493; Nelson 17/e p1912; 18/e, 2360 a. Spontaneous bacterial peritonitis (MP PG 2008)
64. Commonest nephrotic syndrome in child: (DNB 2005) b. Renal vein thrombosis
a. Minimal change c. Acute glomeruloephritis
b. Chronic glomerulonephritis d. Acute renal failure
c. Hemolytic uremic syndrome Ref: Ghai 8/e p477; 7/e p1815; 453
d. Congenital 73. Commonest cause of nephrotic syndrome in children is:
Ref: Ghai 8/e p720; Nelson paediatrics, 18/e p2175, 2176 a. Minimal change disease (Karnatka PG 2010)
65. Berger Nephropathy is due to mesangial deposition of: b. Membranous GN
a. Fibrin and C3 (DNB 2007) c. IgA nephropathy
b. IgD and C3 d. Mesangioproliferative GN
c. IgE and C3 Ref: OP Ghai 8/e p477; 7/e p451; Harrisons, 17/e p1790
d. IgA and C3 74. All of the following are predisposing Factors to Urinary
Ref: Ghai 8/e p670; Nelson Pediatrics 18/e p2171-2174, Robbins pathology Tract infections in children, except: (AP 2010)
66. Most common cause or Renal Artery Stenosis in children a. Vesicoureteric Reflux
with raised ESR in India is: (DNB 2009) b. Uncircumscised males
a. Takayasu Aortoarteritis c. Spina bifida
b. Fibromedial hypertrophy d. Antobiotic treatment Ref: Ghai 8/e p483
c. Fibrointimal hyperplasia 75. An infant with severe dehydration secondary to diarrhea
d. Polyarteritis Nodosa suddenly presents with proteins and blood in urine. The
Ref: Op Ghai Pediatrics 6/e p586, Nelson Pediatric

PEDIATRICS
most probable diagnosis is: (Feb DP PGMEE 2009)
67. Berger nephropathy is due to mesangiail deposition of: a. Renal vein thrombosis
a. Fibrin and C3 (DNB 2010) b. Pyelonephritis
b. IgD and C3 c. Acute glomerulonephritis
c. IgE and C3 d. Lower nephrosis
d. IgA and C3 Ref: Ghai 8/e p477
Ref: Ghai 8/e p376; Nelson Pediatrics 18/e p2171-2174, Robbins pathology 76. Wormian bones are seen in all except: (Kerala PG 08)
68. Aniridia is associated with: (DNB 2010) a. Osteogrenesis imperfecta
a. Hepatoblastoma b. Pyknodysostosis
b. Medulloblastoma c. Cretinism
c. Nephroblastoma d. Renal osteodystrophy
d. Retinoblastoma Ref: OP Ghai 6/e p482; Ghai 8/e p75, 520
Ref: Nelson Pediatrics 18/e p2140-2143; Ghai 8/e p670

Ans. 61. d. Chronic interstitial... 62. b. Avoiding substrate... 63. d. Aldosterone 64. a. Minimal change
65. d. IgA and C3 66. a. Takayasu Aortoarteritis 67. d. IgA and C3 68. c. Nephroblastoma
69. b. Pneumooccus 70. b. Any child with... 71. b. Sickel cell anemia 72. b. Renal vein...
73. a. Minimal change... 74. d. Antobiotic treatment 75. a. Renal vein... 76. d. Renal osteodystrophy
72 Jaypees Triple A
77. Which of the following is true about congenital nephrotic 78. Daily iron requirement of a 10 year old boy is:
syndrome: (Kerala PG 09) a. 2 mg (Kerala PG 10)
a. Diuretic is primary treatment of choice for ascites b. 4 mg
b. Renal biopsy is mandatory c. 8 mg
c. Prophylactic antibiotic has rule ixn prevention of SBP d. 10 mg
d. Live vaccines are indicated Ref: Ghai 8/e p334; Nelson 18/e p210
Ref: Ghai 8/e p482; Nelson 18/e p2195, Management of Sterioid Sensitive
Nephrotic Syndrome Revised Guidelines IAP, IPNG
PEDIATRICS

Ans. 77. b. Renal biopsy is... 78. c. 8 mg


11. DISORDERS OF
ENDOCRINE SYSTEM

A. Precocious and Delayed Puberty


B. Thyroid Disorders
C. Pituitary Disorders
D. Congenital Adrenal Hyperplasia
E. Diabetes Mellitus
F. Miscellaneous
74 Jaypees Triple A

DISORDERS OF ENDOCRINE SYSTEM (QUESTIONS)

A. PRECOCIOUS AND DELAYED PUBERTY 8. Precocious puberty is seen in: (DNB 2007)
a. Hyperthyroidism
1. First sign of puberty in girls: (PGI Dec 99) b. Addisons disease
a. Puberche c. McCune Albright syndrome
b. Thelarche d. Neuroblastoma
c. Growth spurt Ref: Ghai 8/e p542; Nelson 18/e p2379-2382
d. Menarche Ref: Ghai 8/e p531; Nelson 18/e p61 and 9. Precocious puberty is seen in: (DNB 2010)
17/e p54; Nelson 19/e p651 a. Hyperthyroidism
2. A 13-year-old boy has bilateral gynecomastia. His height is b. Addisons disease
148 cm, weight 58 kg; the sexual maturity rating is stage 2. c. McCune Albright syndrome
The gynecomastia is most likely due to: (AIIMS Nov 04) d. Neuroblastoma
a. Prolactinoma Ref: Ghai 8/e p594; Campbell Text book of Peds, 10/e p1028, 1029
b. Testicular tumor 10. Precocious puberty is seen in: (Raj PGI 2009)
c. Pubertal gynecomastia a. Hypothyroidism
d. Chronic liver disease b. CNS irradiation
Ref: Nelson 18/e p63-65 and 17/e p54-55, 1859, 1930-1931; c. Me cune-albright synd
Nelson 19/e p651 d. All
3. Infantile proportion in adult is seen in: Ref: Ghai 8/e p531
a. Moroquids disease (PGI Nov 07) 11. Precocious puberty in girls is suggestive when pubertal
b. Achondroplasia onset is before: (J & K PG 2011)
c. Hypothyroidism a. 7 years
d. Malnutrition b. 8 years
e. Constitutional dwarfism Ref: Ghai 8/e p512; 7/e p18 c. 9 years
4. Precocious puberty is seen in: (PGI Dec 02) d. 10 years Ref: OP Ghai 8/e p531
a. Hypothyroidism
b. CNS irradiation
c. MC cune-Albright syndrome B. THYROID DISORDERS
d. Turners syndrome
12. Blood specimen for neonatal thyroid screening is obtained
e. Congenital adrenal hypoplasia
on: (AI 05; AIIMS May 03)
Ref: Ghai 8/e p532-534; Nelsons Essentials of Pediatrics 5/e, p801;
a. Cord blood
Nelson 19/e p1886, 1887, 1892
b. 24 hours after birth
5. Delayed puberty in children is associated with: c. 48 hours after birth
a. Poliomyelitis (PGI June 00) d. 72 hours after birth
b. Hypothyroidism Ref: Ghai 8/e p518; Harrison 16/e p2109:
c. Hypopituitarism Nelson 19/e p1899
d. Anorexia nervosa
13. In neonatal screening program for detection of congenital
Ref: Ghai 8/e p535; Nelsons Essentials of Pediatrics 5/e, p796;
hypothyroidism, the ideal place and time to collect the
18/e p63-65; Nelson 19/e p649
PEDIATRICS

blood sample for TSH estimation is:


6. Delayed puberty seen in: (PGI Dec 06) a. Cord blood at time of birth (AIIMS May 03)
a. Chronic disease b. Heal pad blood at the time of birth
b. Hypothyroidism c. Heal pad blood on 4 day of birth
c. Turners syndrome d. Peripheral venous blood on 28 day
d. Malabsorption syndrome Ref: Ghai 8/e p516; Harrison 16/e p2109; Nelson 19/e p1899
Ref: Ghai 8/e p535; Nelsons Essentials of Pediatrics 5/e, p796; 18/e p63-65
14. Infantile proportion in adult is seen in:
7. 6-years-old girl ith vaginal spotting. Diagnosis is: a. Morquois disease (PGI Dec 01)
a. Ovarian cancer (DNB 2001) b. Achondroplasia
b. Foreign body c. Hypothyroidism
c. Sexual abuse d. Malnutrition
d. PID Ref: Nelson 18/e p2279 e. Constitutional dwarfism Ref: Ghai 8/e p512

Ans. 1. b. Thelarche 2. c. Pubertal gynecomastia 3. a, b and c 4. a, b and c


5. b, c and d 6. All of the above 7. b. Foreign body 8. c. McCune Albright...
9. c. McCune Albright... 10. d. All 11. b. 8 years 12. d > a
13. c. Heal pad blood... 14. a, b and c
Disorders of Endocrine System 75
15. 8 years old child presents with lethargy multiple epiphyseal 22. Blood specimen for neonatal thyroid screening is obtained on:
breaks, wormian bones with growth retardation and mental a. Cord blood (DNB 2006, 2007)
retardation Diagnosis is? b. 24 hours after birth
a. Rickets (AIIMS Nov 01) c. 48 hours after birth
b. Hypothyroidism d. 72 hours after birth Ref: OP Ghai 8/e p546
c. Scurvy 23. Epiphyseal dysgenesis is a pathognomonic feature of:
d. Hypoparathyroidism a. Hypoparathyroidism (DNB 2007)
Ref: Ghai 8/e p517; 7/e p480-483; Nelson 18/e p2319-2327 and 17/e p1876; b. Hyperparathyroidism (DNB 2010)
Nelson 19/e p1896, 1897 c. Hypothyroidism
16. Common presentations of Juvenile Hypothyroidism: d. Hyperthyroidism
a. Growth retardation (PGI June 06) Ref: Ghai 8/e p524; Nelson Pediatrics 18/e p2319
b. Mental retardation within 2 years 24. Most common cancer of thyroid in children is: (DNB 2011)
c. Delayed puberty a. Papillary
d. Umbilical Hernia b. Follicular
e. Moist skin c. Medullary
Ref: Ghai 8/e p517; 7/e p480-483; Nelson 18/e p2319-2327 and 17/e p1876; d. Anaplastic Ref: Esssentials of Pediatrics, Nelson 4/e p739
Nelson 19/e p1898
25. Hypothyroidism in infancy is characterized by all except:
17. True about Pendred syndrome: (PGI Dec 04) a. Constipation (UP PG 09)
a. Diffuse colloid goiter b. Coarse facies
b. Nodular goiter c. Wide open cranial sutures
c. Mental retardation d. Hyperthermia Ref: Ghai 8/e p520; 7/e p481
d. BIL sensory neural deafness
26. The features of neonatal hyperthyrodidism include all except:
e. Normal cochle
a. Triangular facies with craniosynostosis
Ref: Harrison 16/e p181, 2104; Ghai 8/e p516; http://www.medicinenet.
b. Congestive cardian failure (DP PGMEE 2009)
com/pendred_syndrome/article.htm; Nelson 19/e p1896, 1906
c. Advanced osseous maturation
18. Pendreds syndrome is: (PGI June 02) d. Goiter is rare
a. Consistently associated with deafness Ref: Ghai 8/e p75, 520; Nelson 18/e p2319-2322
b. Hypothyroidism is seen
27. Clinical features of hypothyroidism in an new born are
c. Mutation in connection coding gene
all, except: (Feb DP PGMEE 2009)
d. Mutation in chromosome 21 causing receptor defect
a. Sluggishness +++
Ref: Ghai 8/e p516; 7/e p480-483; Harrison 16/e p181, 2104; http:// www.
b. Large tongue
medicinenet.com/pendred_syndrome/article.htm
c. Large posterior fontanel
19. Which of the following is true in cretinism: d. Mental retardation Ref: Ghai 8/e p516
a. Goiter present at birth (PGI Dec 01)
28. Human fetus secretes thyroxine from which gestation age?
b. Can be diagnosed by serum T4 levels
a. 8 weeks (J & K PG 2011)
c. Prolonged physiological jaundice present
b. 12 weeks
d. Common in iodine deficiency endemic areas
c. 24 weeks
e. Delayed skeletal development
d. 30 weeks Ref: Ghai 8/e p516
Ref: Ghai 8/ep517; 7/e p480-483; Harrison 16/e p181, 2104; http://www.
medicinenet.com/pendred_syndrome/article.htm
20. Clinical features of hypothyroidism in a newborn are all C. PITUITARY DISORDERS
except: (MAHE 05)

PEDIATRICS
a. Sluggishness +++ 29. Which of the following is true regarding cretinism:
b. Large tongue a. Short limbs compared to trunk (AI 01)
c. Large posterior fontanel b. Proportionate shortening
d. Mental retardation Ref: Ghai 8/e p517; 7/e p481 c. Short limbs and short stature
21. Features of hypothyroidism in infancy include the d. Short limbs and long stature
following except: (UPSC 06) Ref: Ghai 8/e p512; 7/e p481; Nelson 18/e p1885; Nelson 19/e p1896
a. Premature closure of posterior fontanel 30. Manifestations of endemic cretinism include:
b. Coarse facies a. Deafness and facial nerve involvement
c. Umbilical hernia b. Blindness and hypothyroidism (AI 01)
d. Constipation c. Strabismus and spastic diplegia
Ref: Ghai 8/e p517; 7/e p481 d. Multinodular goitre and mental retardation
Ref: Ghai 8/e p519; 7/e p475; Nelson 18/e p1885

Ans. 15. b. Hypothyroidism 16. a, b and d 17. a, c and d 18. a, b and c


19. c, d and e 20. d. Mental retardation 21. a. Premature closure... 22. d. 72 hours after birth
23. c. Hypothyroidism 24. a. Papillary 25. d. Hyperthermia 26. d. Goiter is rare
27. d. Mental retardation 28. b. 12 weeks 29. a and c 30. c. Strabismus...
76 Jaypees Triple A
31. A 9-year-old boy presents with growth retardation and d. Gonadal dysgenesis Ref: Ghai 6/e p468; Nelson 19/e p1932
propensity to hypoglycemia. Physical examination reveals 38. A 8-day-old breast fed baby presents with vomiting, poor
short stature, micropenis, increased fat and high-pitched feeding and loose stooled. On examination the heart rate is
voice. The skeletal survey reveals bone age of 5 years. 190/minute, blood pressure 50/30 mm Hg, respiratory rate
Which of the following is most appropriate diagnosis: 72 breaths/minute and capillary refill time of 4 seconds.
a. Malabsorption (AIIMS Nov 04) Investigations show hemoglobin level of 15g/DL. Na-12
b. Growth hormone deficiency mEq/L, K-6.8 mEq/L, Cl-81 mEq/L, bicarbonate 15 mEq/L, Urea
c. Adrenal tumor 30 mg/dL, creatinine 0.6 mg/dL. The most likely diagnosis is:
d. Thyroxin deficiency Ref: Ghai 8/e p511; 7/e p474-475 a. Congenital adrenal hyperplasia (AIIMS Nov 03)
32. Weak giants are produced by: (AIIMS May 04) b. Acute tubular necrosis
a. Thyroid adenomas c. Congenital hypertrophic pyloric stenosis
b. Thyroid carcinomas d. Galactosemia Ref: Nelson 18/e p2362; Nelson 19/e p1930
c. Parathyroid adenomas 39. A 5-year-old girl presents with hypertension and
d. Pituitary adenomas Ref: Harrison 16/e p2090; virilization. There is also finding of hypokalemia what is
Nelson 18/e p2303 and 17/e p1859; Nelson 19/e p1878 the diagnosis:
a. 21-hydroxylase deficiency (AIIMS May 07)
b. 3-hydroxy seroid deficiency
D. CONGENITAL ADRENAL HYPERPLASIA c. 11-hydroxylase deficiency
d. Cohns disease
33. Which one of the following drugs is used for fetal therapy
Ref: See previous explanations; Nelson 19/e p1935, 1936
of congenital adrenal hyperplasia?
a. Hydrocortisone (AI 05) 40. A five-years-old boy has precocious puberty BP 130/80.
b. Prednisolone Estimation of which of the following will help in diagnosis?
c. Flurocortisone a. 17-Hydroxyprogesterone (AI 09)
d. Dexamethasone Ref: Ghai 7/e p493, 6/e p490; b. 11-Deoxycortisol
Nelson 18/e, 2360-2367 and 17/e p1912; Nelson 19/e p1934 c. Aldosterone
d. DOCA Ref: Ghai 7/e p489; Nelson 18/e,
34. A 1-month-old baby presents with frequent vomiting and
Ch 563, 571, p2360, 2367 Nelson 19/e p1937
failure to thrive. There are features of moderate dehydration.
Blood sodium is 122 mEq/L and potassium is 6.1 mEq/L. The 41. Adrenal hyperplasia due to 21 hydroxylase deficiency is
most likely diagnosis is: (AI 03, 02) treated with low dose: (Kerala 04)
a. Gitelman syndrome a. Androgen
b. Bartter syndrome b. Estrogen
c. 21-hydroxylase deficiency c. Cortisone
d. 11-b hydroxylase deficiency Ref: Ghai 8/e p527; d. Anti androgen
7/e p493; Nelson 17/e p1912; 18/e,2360; Nelson 19/e p1930 Ref: Ghai 8/e p514; 7/e p492; Nelson 17/e p1912
35. A 10-day-old male pseudohermaphrodite child with 46 XY 42. A 6-week-old male infant was brought in a state of
karyotype presents with BP of 110/80 mmHg. Most likely dehydration and shock. Examination revealed hyper
enzyme deficiency is: (AI 01) pigmentation over the body with normal external genitalia.
a. 21 hydroxylase Blood tests revealed hypoglycemia, Na - 124 mEq/L and K -
b. 17 hydroxylase 7 mEq/L. What is the probable diagnosis?
c. 11 hydroxylase a. Congenital adrenal hyperplasia
d. 3-beta hydroxylase b. Adrenal hemorrhage and shock
Ref: Ghai 7/e p489-490, 6/e p487-489; 18/e p2360 and c. Acute gastroenteritis with dehydration
PEDIATRICS

Nelson 17/e p1911, 280; Nelson 19/e p1936 d. Septicemia with ARF
Ref: Ghai 8/e p517; Nelson 18/e p2361-64
36. Congenital adrenal hyperplasis is associated with:
a. Hypoglycemia (PGI Dec 00) 43. A 6 week old male infant was brought in a state of
b. Hyponatremia dehydration and shock. Examination revealed hyper
c. Hypokalemia pigmentation over the body with normal external genitalia.
d. Hyperkalemia Blood tests revealed hypoglycemia, Na 124 mEq/L and K -
Ref: Nelson 18/e p2360, 2367 and 17/e p1913; Nelson 19/e p193 7 mEq/L. Most probable diagnosis is: (DNB 2009)
a. Congenital adrenal hyperplasia
37. Male pseudothermaphroditism is seen in:
b. Adrenal haemorrhage and shock
a. 5 reductase deficiency (PGI Dec 07)
c. Acute gastroenteritis with dehydration
b. 21 hydroxylase deficiency
d. Septicemia with ARF
c. 17 hydroxylase deficiency
(Ref: Ghai 8/e p638; Nelson 18/e p2361-64

Ans. 31. b. Growth hormone... 32. d. Pituitary adenomas 33. d. Dexamethasone 34. c. 21 -hydroxylase...
35. b. 17 hydroxylase 36. a, b and d 37. a, c and d 38. a. Congenital adrenal...
39. c. 11-hydroxylase... 40. b. 11-Deoxycortisol 41. c. Cortisone 42. a. Congenital adrenal...
43. a. Congenital adrenal...
Disorders of Endocrine System 77
44. Most common cause of CAH is deficency of hydroxylase: Serum calcium 12.8 mg/dL, serum phosphorus 2.3 mg/dL,
a. 21 Alpha (DNB 2010) alkaline phosphate 28 Ka units and blood urea32 mg/dL.
b. 11 beta Which of the following is the most probable diagnosis in
c. 3 beta his case: (AI 04)
d. 3 alpha Ref: Ghai 8/e p594; Nelson 18/e p2361-64 a. Nutritional rickets
45. A female child with Hypertension, Hyperpigmentation, b. Renal rickets
Virilization has deficiency of: (DNB 2011) c. Hyperparathyroidism
a. 11 beta hydroxylase d. Skeletal dysplasia Ref: Nelson 17/e p2345; Ghai 8/e p518
b. 17 alpha hydroxylase 52. Which of the following statements about 21 alpha
c. 3 beta hydroxylase hydroxylase deficiency is false: (AI 08)
d. 21 alpha hydroxylase a. Most common cause of Congenital Adrenal Hyperplasia
Ref: Ghai 8/e p498; Nelson Ped, 18/e p2366 (CAH) in children
46. Most common type of Congenital adrenal hyperplasia (CAH)? b. Affected females present with a ambiguous genitalia
a. 21-hydroxylase deficiency (MHPGM-CET 2010) c. Affected males present with precarious puberty
b. 11-hydroxylase deficiency d. Hypokalemic alkalosis is seen
c. 3-hydroxylase deficiency Ref: Ghai 8/e p526; 7/e p493; Nelson 18/e p2360;
d. 17-hydroxylase deficiency Nelson 19/e p1930, 1961
Ref: OP Ghai 8/e p526; 7/e p492; Harrisons 17/e p2267 53. 8 year old boy with undescended testis, your concern to ask
47. In congenital adrenal hyperplasia most common deficiency: for operation is due to:
a. 21 alpha hydroxylase deficiency (Kerala PGI 09) a. Cosmetic reasons (PGI June 01)
b. 11 beta hydroxylase deficiency b. Infertility
c. 17 hydroxylase deficiency c. Risk of malignancy
d. 3 beta hydroxylase deficiency d. Impotence
Ref: Ghai 8/e p526; Nelson 18/e p2360 Ref: Ghai 8/e p540; Nelsons Essentials of Pediatrics 5/e, p769;
18/e p2144; Nelson 19/e p1858-1860
54. Orchidopex is done in cases of undescended testes at the
E. DIABETES MELLITUS age of: (AIIMS Nov 06)
a. Neonate
48. In India the commonest cause of Juvenile Onset of Diabetes b. 1-2 years
mellitus: (AIIMS May 01) c. 5 years
a. IDDM d. Puberty
b. Fibrocalcific pancreaticopathy Ref: Ghai 8/e p540; Nelson 18/e p2261; Nelson 19/e p1859
c. Mody
55. A boy with undescended testis, your concern to ask for
d. Gall stones
operation is due to: (PGI 01)
Ref: Nelson 18/e p2404 and 17/e p1947; Nelson 19/e p1970
a. Cosmetic reasons
49. In children with type I DM when is ophthalmologic b. Infertility
evaluation indicated: (PGI June 06) c. Risk of malignancy
a. At the time of diagnosis d. Impotence
b. After 1 year Ref: Ghai 8/e p540; Nelson 18/e p2260; Nelson 19/e p1859
c. After 2 years
56. Surgical treatment of cryptorchidism of right testis with
d. After 5 years
normally descended left testis can be done:
e. After 10 years Ref: Ghai 8/e p666;
a. Immediately (All India 99)

PEDIATRICS
API Textbook of Medicine, 7/e p1136; Nelson 19/e p1970, 1971
b. At 2 years of age
50. Which one of the following medical disorders leads to c. At 4 years before going to school
delayed fetal lung maturity: (UPSC07) d. At puberty
a. Heart disease e. None
b. Diabetes Ref: Nelson 18/e p2260; Nelson 19/e p1860
c. Thalassemia minor
57. Unilateral undescended testes is ideally operated around:
d. Epilepsy Ref: Ghai 8/e p541; Nelson 17/e, 575
a. 2 months of age (AI 04)
b. 6 months of age
F. MISCELLANEOUS c. 12 months of age
d. 24 months of age
51. A 10-year-old boy has a fracture of femur. Biochemical Ref: Nelson 18/e p2261; Nelson 19/e p1859, 1860
evaluation revealed Hb 11.5 gm/dl and ESR 18 mm 1st hour.

Ans. 44. a. 21 Alpha 45. a. 11 beta hydroxylase 46. a. 21- hydroxylase... 47. a. 21 alpha hydroxylase...
48. b. Fibrocalcific ... 49. d. After 5 years 50. b. Diabetes 51. c. Hyperparathyroidism
52. d. Hypokalemic... 53. b and c 54. b. 1-2 years 55. b and c
56. e. None 57. b. 6 months...
78 Jaypees Triple A
58. A 10-years-old boy has a fracture of femur. Biochemical a. 1, 2 and 3 only
evaluation revealed Hb 11.5 gm/dL and ESR 18 mm first b. 1 and 3 only
hour. Serum calcium 12.8 mg/dL, serum phosphorus 2.3 mg/ c. 1 and 4 only
dL, alkaline phosphate 28 KA units and blood urea 32 mg/ d. 1, 2, 3 and 4 Ref: OP Ghai 8/e p655; Nelson 17/e, 399-400
dL. Which of the following is the most probable diagnosis 64. A child of phenyl ketonuric mother may develop:
in his case? (AI 04) a. Microcephaly, mental retardation, congenital heart
a. Nutritional rickets disease (AIIMS June 99)
b. Renal rickets b. Mental retardation, cataract, congenital heart disease
c. Hyperparathyroidism c. Hydrocephalus, cataract
d. Skeletal dysplasia d. Microcephaly, cataract, renal dysplasia
Ref: Ghai 7/e p486-487, 6/e p129-130; Nelson 18/e p2345-2348; Ref: Ghai 8/e p650-652; Nelsons Essentials of Pediatrics 5/e p256
Nelson 19/e p1922
65. A child has microcephaly, blue eyes, fair skin, and mental
59. A 10-years-old boy is having polyuria, polydypsia, retardation, ferric chloride test is positive. What is the likely
laboratory data showed (in mEq/lit) Na-154, K-4.5, HCO3- diagnosis: (AI 07)
22, Serum osmolality - 295, Blood urea-50, Urine specific a. Phenylketonuria (PKU)
gravity -1.005. The likely diagnosis is: (AIIMS May 02) b. Homocystinuria
a. Diabetes insipidus c. Tyrosinosis
b. Renal tubular acidosis d. Alkaptonuria
c. Bartters syndrome Ref: Ghai 8/e p650; 7/e p628, Nelson 18/e p529; Ghai 6/e p609
d. Recurrent UTI
66. Which one of the following is not a feature of
Ref: Ghai 8/e p514; 7/e p477; API 6/e p994; Neslon 18/e p2299-2301
Phenylketonuria? (UPSC 04)
60. ACTH secretion is highest during: (PGI Dec 99) a. Severe mental retradation
a. Noon b. Reduced tendon reflexes
b. Evening c. Enamel hypoplasia
c. Morning d. Vomiting in early infancy
d. Night Ref: Ghai 8/e p654; 7/e p628; Nelson 17/e p399, 400
Ref: Ghai 7/e p490, 6/e p497; Nelson 18/e p2291, 2292 and
67. A 15-day-old baby came with history of seizures. Blood
17/e p1917; Nelson 19/e p1940
tests revealed Ca 5 mg/dl, PO4 9 mg/dl, PTH 30pg/ml (n
61. Which one of the following is not a feature of = 10-60). What is the most probable diagnosis?
Phenylketonuria? (UPSC 04) a. Pseudohypoparathyroidism
a. Severe mental retardation b. Vitamin D deficiency
b. Reduced tendon reflexes c. Hyperparathyroidism
c. Enamel hypoplasia d. HIE
d. Vomiting in early infancy Ref: Ghai 8/e p517; Manual of neonatal care, Cloherty p550,
Ref: Ghai 8/e p653; Nelson 17/e p399, 400 Nelson 18/e p2343
62. Consider the following diseases: (UPSC 07) 68. Earliest indication of sexual maturation in a girl is:
1. Addisons disease a. Menarche (DNB 2007)
2. Hypervitaminosis A b. Pubarche
3. Obesity c. Menopause
4. Hyperparathyroidism d. Maturation of breasts
Which of the above are associated with pseudotumor cerebri Ref: Ghai 8/e p324; Nelson 18/e p2374, 2379, 2389
a. 1, 2 and 3 only
69. Not a cause of hirusitism is: (DNB 2010)
PEDIATRICS

b. 1 and 3 only
a. Testicular feminization
c. 2 and 4 only
b. Cushings syndrome
d. 1, 2, 3 and 4
c. CAH
Ref: O.P. Ghai 8/e p549; Harrison 16/e, 1698, 1701; Nelson 17/e, 2048
d. PCOD
63. With reference to classical phenylketonuria, consider the Ref: OP Ghai 8/e p610; Nelson 18/e p2375, 2379
following statements: (UPSC 07)
70. Earliest indication of sexual maturation in a girl is:
1. Mental retardation is usually mild
a. Menarche (DNB 2010)
2. Vomiting is usually severe
b. Pubarche
3. Hypotonia and reduced tendon reflexes are present
c. Thelarche
4. Convulsions may occur in 25% of cases
d. Maturation of breasts
Which of the above are the features of classical
Ref: Ghai 8/e p519; Nelson Pediatrics 18/e p2374, 2390, 16/e p53
phenylketonuria: (UPSC 07)

Ans. 58. c. Hyperparathyroidism 59. a. Diabetes insipidus 60. c. Morning 61. b. Reduced...
62. d. 1, 2, 3 and 4 63. c. 1 and 4 only 64. a. Microcephaly... 65. a. Phenylketonuria (PKU)
66. b. Reduced tendon... 67. a. Pseudohypoparathyroidism 68. d. Maturation of... 69. a. Testicular feminization
70. c. Thelarche
12. DISORDERS OF
NEUROMUSCULAR SYSTEM
80 Jaypees Triple A

DISORDERS OF NEUROMUSCULAR SYSTEM (QUESTIONS)

1 A baby presents with tetany. First thing to be done is 7. All are seen in metahromatic leukodystrophy except:
administration of: (AIIMS June 00) a. Mental retardation (TN 03)
a. Diazepam b. Optic atrophy
b. Vitamin D c. Decrebrate posture
c. Calcium gluconate d. Exaggerated tendon reflexes
d. Calcitonin Ref: OP Ghai 8/e p724; 7/e p7/e, p639; Nelson 17/e p2032
Ref: Nelson 18/e p2864 and 17/e p1892; Ghai 8/e p118 8. Ataxia telangiectasia is characterized by all of the following
2. With reference to Polyarticular juvenile rheumatoid except: (AI 04)
arthritis, consider the following statements: a. Chronic sinopulmonary disease
1. It is more common in girls (UPSC 07) b. Decreased level of a-fetoprotein
2. Five or more joints are affected within the first six months c. Chromosomal breakage
of onset d. IgA deficiency
3. Uveitis occurs in 95% patients Ref: Ghai 8/e p580, 546;
4. Rheumatoid factor may be negative Ref: Ghai 8/e p589 Nelson 18/e p891, 2489, 2674 and 17/e p699, 2020
Which of the statements given above are correct ? 9. True regarding febrile convulsion: (PGI Dec 00)
a. 1 and 4 only a. Carbamazepine is good drug to treat it
b. 1, 2 and 4 b. Patients with family h/o FC have increased incidence of
c. 2, 3 and 4 recurrence
d. 2 and 3 only c. Long-term neurological deficits are common
Ref: O.P. Ghai 7/e p599, 6/e p52; IAP textbook of Pediatrics 3/e p868 d. Usually last for short while
3. A 2-years-old child is brought to emergency at 3 AM with Ref: Ghai 8/e p556; Nelsons Essentials of Pediatrics 5/e,
fever, barking cough and stridor only while crying. The p838; 18/e p2457; Nelson 19/e p2013, 2017
child was able to drink normally. On examination respirator 10. 4-years-old male child had febrile seizures, best prophylaxis:
rate is 36/min and temperature is 39.6c. What will be your a. Paracetamol 6 hourly (PGI June 00)
next step: b. Paracetamol and diazepam
a. Racemic epinephrine nebulization (AIIMS Nov 08) c. Diazepam
b. High dose dexamethasone injection d. Phenobarbitone
c. Nasal wash for influenza or RS V Ref: Ghai 8/e p556, 557; 7/e p528; Nelson 18/e p2457 and
d. Antibiotics and blood culture 17/e p1994-1995 Nelson 19/e p2018
Ref: Nelson 18/e p1765; Ghai 8/e p375; 7/e p339, 3406/e p339, 340;
11. Which of the following is NOT associated with increase in
Nelson 19/e p1446
the risk of seizures in future in a child with febrile seizures?
4. All are seen in metachromatic leukodystrophy except: a. Developmental delay (AI 2010)
a. Mental retardation (TN 03) b. Early age of onset
b. Optic atrophy c. Complex partial seizures
c. Decerebrate posture d. Family history positive
d. Exaggerated tendon reflexes Ref: OP Ghai 8/e p556; 7/e p528;
Ref: Ghai 8/e p647; 7/e p561; Nelson 17/e p2032 Nelson Paediatrics IT11/1994; Nelson 19/e p2017
5. Macrocephaly is seen in which of the following syndromes? 12. Which one of the following is the characteristic feature of
PEDIATRICS

a. Metachromatic leukodjystrophy (AIIMS May 08) juvenile myoclortic epilepsy?


b. Adrenoleukodystrophy a. Myoclonic seizures frequently occur in morning
c. Canavans disease b. Complete remission is common (AIIMS May 06)
d. Krabbes disease c. Response to anticonvulsants is poor
Ref: Internet reference; Nelson 19/e p455, 456 d. Associated absence seizures are present in majority of
6. Deep white matter lesion with bilateral deep bright patients
thalamic appearance is suggestive of: Ref: Ghai 8/e p557; Nelson 18/e p2457-2475 and 17/e p1998;
a. Alexander disease (AIIMS Nov 07) Harrison 16/e p2359; Nelson 19/e p2015, 2016
b. Canavans diseases 13. True about juvenile myoclonic epilepsy:
c. Krabbes disease a. DOC is Sodium Valproate (PGI June 08)
d. Metachromatic leukodystrophyz b. Mental retardation
Ref: Ghai 7/e p637; Nelson 18/e p2502; Nelson 19/e p48

Ans. 1. c. Calcium gluconate 2. b. 1, 2 and 4 3. b. High dose... 4. d. Exaggerated tendon...


5. c. Canavans disease 6. c. Krabbes disease 7. d. Exaggerated tendon... 8. b. Decreased level...
9. b and d 10. b. Paracetamol and... 11. b. Early age of onset 12. a. Myoclonic seizures...
13. a, c and e
Disorders of Neuromuscular System 81
c. Seizure can develop 21. The commonest organism responsible for meningitis in
d. Neurological examination abnormal neouate child is:
e. Life long treatment needed a. Listeria (AI 07, PGI June 2007)
Ref: Ghai 8/e p557; Nelson 18/e p2457-2475 and 17/e p1998; b. E.coli
Nelson 19/e p2015 c. H. Influenza
14. Most common cause of seizure in newborn is: d. Type B streptococcus
a. Hypoxia induced ischemic encephalopathy Ref: Ghai 8/e p563; Nelson 18/e p2514,
b. Hypocalcemia (AIIMS Nov 08) 2517 and 17/e p2038-2039; Nelson 19/e p2087
c. Metabolic abnormality 22. Bacterial meningitis in children (2 months-12 years of age)
d. Sepsis is usually due to the following organisms except: (AI 04)
Ref: Ghai 8/ep166; Nelson 18/e p718-720; Nelson 19/e p2033, 2034 a. Streptococcus pneumoniae
15. Seizure and Jitteriness can be differentiated by all except: b. Neisseria meningitidis
a. Gaze (AI 07) c. Hemophilus influenzae type B
b. Autonomic disturbances d. Listeria monocytogenes
c. Sensitivity to stimulus Ref: Ghai 8/e p563; Nelson 18/e p1157-1159 and 17/e p2038;
d. Frequency of movements Nelson 19/e p2088
Ref: Ghai 8/e p561; Nelson 18/e p718-720; Nelson 19/e p2035 23. Hemophilus influenzae has been isolated from the CSF of
16. Commonest type of seizure in newborn: a two year old boy suffering from meningitis. The strain is
a. Clonic (AIIMS June 08) beta-lactamase producing and resistant to chloramphenicol.
b. Tonic The most appropriate antimicrobial in such a situation is:
c. Subtle a. Trimethoprim-sulfamethoxazole combination
d. Myoclonic b. Ciprofloxacin (AI 04)
Ref: Ghai 8/e p558; 7/e p142; Care of the New born, c. Third-generation cephalosporin
Meharban Singh, 6/e p329; Nelson 19/e p2034 d. Vancomycin Ref: Ghai 8/e p565;
Nelson 18/e p1119-1120 and 17/e p2042; Nelson 19/e p209
17. Commonest type of seizure in newborn: (AI 09)
a. Atonic 24. Most common cause of pyogenic meningitis in 6 month to 2
b. Tonic years of age is: (AI 98)
c. Subtle a. Staphylococcus aureus
d. Myoclonic b. Pneumococcus
Ref: Ghai, 8/e p558; Nelson 18/e p1340; Nelson 19/e p2034 c. Streptococcus pneumonia
d. H. influeuenzae
18. All are features of absence seizures except:
Ref: Nelson 18/e p2514-2517 and 17/e p2038; Ghai 8/e p563
a. Usually seen in childhood [AI 04]
b. 3-Hz spike wave in EEG 25. Neurological complications of meningitis include all of the
c. Postictal confusion following except: (PGI June 08)
d. Precipitation by hyperventilation a. Seizures
Ref: Ghai 8/e p557; 7/e p530; Nelson 18/e p2032 and 17/e p1997; b. Increased intracranial pressure
Nelson 19/e p2023 c. Cerebral hamartoma
d. Subdural effusions
19. A school going boy was noted with vacant stare several
e. Brain abscess Ref: Ghai 8/e p563; Nelson 19/e p2089
times a day. There was no history of fever, seizures and
neurological deterioration. What is the diagnosis? 26. CSF examination of a patient shows high protein, markedly
a. Atonic seizures (AI 2010) low sugar, low chloride and increased neutrophils. The
diagnosis is: (AIIMS Nov 99)

PEDIATRICS
b. Absence seizures
c. Myoclonic seizures a. Viral meningitis
d. School phobia b. Meningococcal meningitis
Ref: Ghai 8/e p557; 7/e p530; Nelson 19/e p2023 c. Tuberculous meningitis
d. Fungal meningitis Ref: Ghai 8/e p563, 564;
20. A 6 years old child with acute onset of fever of 104 F
Nelson 18/e p2513-2531 and 17/e p2039; Nelson 19/e p2088
developed febrile seizures and was treated. To avoid future
recurrence of seizure attacks what should be given: 27. Aseptic meningitis is caused by all except:
a. Paracetamol 400 mg + Phenobarbitone daily a. Mumps (AIIMS June 98)
b. Oral Diazepam 6 hourly (AIIMS May 01) b. Polio
c. Paracetamol 400 mg 6 hourly c. Measles
d. IV diazepam infusion over 12 hours d. Coxsackie virus
Ref: Ghai 8/e p560; 7/e p528; Nelson 18/e p2457-2475 and Ref: Ghai 8/e p562; Nelsons Essentials of Pediatrics 5/e,
17/e p1994-1995; Nelson 19/e p2018 p481-482; 18/e p2513-2531

Ans. 14. a. Hypoxia induced... 15. d. Frequency of... 16. c. Subtle 17. c. Subtle
18. c. Postictal confusion 19. b. Absence seizures 20. b. Oral Diazepam... 21. d. Type B streptococcus
22. d. Listeria... 23. c. Third-generation... 24. b and c 25. c. Cerebral hamartoma
26. b. Meningococcal... 27. c. Measles
82 Jaypees Triple A
28. 5-year-old comes to pediatric ICU with RR 42, BP 110/80 agent in the maternal genital tract can be the causative agent
and E1V1M3 on Glasgow coma scale, next step (tachycardia, of this disease:
unconscious with 2 days h/o fever): (PGI Nov 2009) a. Neisseria gonorrhea (AIIMS May 04)
a. Intubatc and ventilate b. Chlamydia trachomatis
b. IV Manitol c. Streptococcus agalactiae
c. Give IV dopamine 10g/kg d. Haemophilous ducreyi
d. Give both furoseimde and dopamine Ref: Ghai 8/e p163, 162; Nelson 18/e p2513-2531
e. Start antibiotics and watch CT Scan and 17/e p2038; Harrison 16/e p829
Ref: Ghai 8/e p708 35. Most common cause of neonatal meningitis:
29. A 25 years old woman had premature rupture of membranes a. Staphylococcus
and delivered a male child who became lethargic and apneic b. E. coli (PGI Dec 99)
on the 1st day of birth and went into shock. The mother had c. H. influenza
a previous history of abortion 1 year back. On vaginal swab d. Pneumococcus
culture growth of b-hemolytic colonies on blood agar was Ref: Ghai 8/e p162; Nelson 17/e p2038;
found. On staining these were found to be gram-positive Harrison 16/e p829
cocci. Which of the following is the most likely etiological 36. CSF picture in tubercular meningitis is: (AI 07)
agent? (AI 04) a. Increased protein, increased sugar, increased lymphocyte
a. Streptococcus pyogenes b. Increased protein, decreased sugar, increased lymphocyte
b. Streptococcus agalactiae c. Decreased protein, increased sugar, increased
c. Peptostreptococci lymphocyte
d. Enterococcus faecium d. Decreased protein, decreased sugar, increased
Ref: Ghai 8/e p162; Nelson 18/e p1145-1149 and 17/e p627 lymphocyte
30. Most common cause of meningitis in children between 6 Ref: Ghai 8/e p566; Nelson 18/e p2513-2531 and 17/e p2039;
months to 2 years of age is: Nelson 19/e p2088
a. Pneumococcus (AI 01, AI 98) 37. A 10 month old child presents with two weeks history of
b. Staphylococcus fever, vomiting and alteration of sensorium Cranial CT scan
c. H influenza reveals basal exudates and hydrocephalus, the most likely
d. E. coli etiological agent is:
Ref: Ghai 8/e p162; Nelson 18/e p2541-2517 and 17/e p2038; a. Mycobacterium tuberculosis (AI 04, 95)
Ghai 6/e p517; Nelson 19/e p2087 b. Cryptococcus neoformans
31. A newborn of 7 days old presented with meningitis. Most c. Listera monocytogenes
common cause: (PGI Nov 08) d. Streptococcus pneumonia
a. E. Coli Ref: Ghai 8/e p566; Nelson 18/e p1248, 1249;
b. Streptococcal pneumonia Nelson 19/e p1066
c. N. meningitides 38. The CSF findings in TB meningitis include:
d. Streptococcal agalactiae a. High sugar + low protein (AI 94, 07)
e. H.influenzae Ref: Ghai 8/e p162 b. Low sugar + high protein and Lymphocytosis
32. Neonatal meningitis is caused by: (PGI Nov 07) c High sugar + high chloride
a. Group A streptococcus d. Low sugar + high protein and Lymphopenia
b. Group B streptococcus Ref: Ghai 8/e p566; Nelson 19/e p2088
c. E. coli 39. All of the following are neural tube defects except: (AI 04)
d. H. influenza a. Myelomeningocoele
PEDIATRICS

e. Klebsiella b. Anencephaly
Ref: Ghai 8/e p162, 163 c. Encephalocele
33. The following bacteria are most often associated with acute d. Holoprosencephaly
neonatal meningitis except: (AI 05) Ref: Ghai 8/e p575; 7/e p550-551; Nelson 18/e p2449, 2450 and 17/e
a. Escherichia coli p1983-1987; Nelson 19/e p1998, 1999
b. Streptococcus agalactiae 40. Neural tube defects are prevented by: (AI 99)
c. Neisseria meningitidis a. Pyridoxin
d. Listeria monocytogenes b. Folic acid
Ref: Ghai 8/e p162; Nelson 18/e p2513-2531 and c. Thiamine
17/e p627, 2038; Nelson 19/e p2095 d. Iron
34. A neonate develops signs of meningitis at seven days of Ref: Ghai 8/e p575; 576; 7/e p550-551; Nelson 19/e p2001-2002
birth. The presence of which of the following infectious

Ans. 28. a, b and e 29. b. Streptococcus... 30. a. Pneumococcus 31. d. Streptococcal agalactiae
32. b, c and e 33. c. Neisseria meningitidis 34. c. Streptococcus... 35. b. E. coli
36. b. Increased protein... 37. a. Mycobacterium... 38. b. Low sugar + high... 39. d. Holoprosencephaly
40. b. Folic acid
Disorders of Neuromuscular System 83
41. Increase acetylcholinestrase in amniotic fluid indicates: 49. Reyes syndrome is characterized by:
a. Open neural tube defects (AI 99) a. Viral infection is seen (PGI Dec 03)
b. Esophageal atresia b. Present as deep jaundice
c. Downs syndrome c. Cerebral edema
d. Edwards syndrome d. Microvesicular fatty infiltration
Ref: Ghai 8/e p575; Nelson 18/e p2449, 2450 and 17/e p1983; 50. True about Reyes syndrome: (PGI June 05)
Nelson 19/e p2001 a. Microvesicular fatty infiltration
42. Which one of the following is the common cause of b. Hepatic encephalopathy
congenital hydrocephalus? (AI 05) c. Brain edema
a. Craniosynostosis d. Hypoglycemia
b. Intra uterine meningitis e. All
c. Aqueductal stenosis Ref: Ghai 8/e p569; See previous explanations
d. Malformations of great vein of Galen 51. 10 years old Ramu has increasing muscle weakness and
Ref: Ghai 8/e p574; 7/e p549; Nelson 18/e p2452-2455 and raised CPK levels. The most likely defect is in plasma
17/e p1990; Nelson 19/e p2008, 2009 membrane of: (AIIMS Nov 01)
43. Porencephaly refers to: (AI 99) a. Nerves
a. Fetal alcohol syndrome b. Muscle fibers
b. Dandy-walker-syndrome c. Basement membrane
c. Vascular lesion due to degenerative vessel disease and d. All body cells
head injury Ref: Ghai 8/e p594; Nelson 18/e p2095 and 17/e p2053-2054;
d. Neural tube defects Harrison 16/e p1450; Nelson 19/e p2119, 2120
Ref: Ghai 8/e p575; Nelson 18/e p2449 and 17/e p1987; 52. All of the following are associated with proximal muscle
Nelson 19/e p2010, 2011 weakness except: (AIIMS May 02)
44. Most common cause of hydrocephalus in children is: a. Spinomuscular atrophy
a. Post inflammatory obstruction (AI 98) b. Duchenne muscular dystrophy
b. Buddchiary syndrome c. Polymyositis
c. Brain tumor d. Myotonic dystrophy
d. Perinatal injury Ref: Ghai 8/e p589; Nelson 18/e p2544-2547 and 17/e p2065;
Ref: Ghai 8/e p574; Nelson 18/e p2452-2455 and 17/e p1990, 2040 Nelson 19/e p2123, 2125
45. Macrocephaly is seen in which of the following syndromes? 53. All of the following are features of systemic Juvenile
a. Metachromatic leukodystrophy (AIIMS Nov 08) Rheumatoid Arthritis except: (AIIMS May 01)
b. Adrenoleukodystrophy a. Uveitis
c. Canavans disease b. Rash
Ref: Ghai 8/e p35; CDTP 18/e (2007), Robbins 7/e p1398 c. Fever
46. Mild MR Feature are: (PGI June 06) d. Hepatosplenomegaly
a. Present in 5-10% population Ref: Ghai 8/e p624;
b. Incidence in low socioeconomic group Nelson 18/e p1001-1010 and 17/e p799-801;
c. Present in 2 years Nelson 19/e p2173
d. Genetic background present 54. Childhood osteopetrosis is characterized by:
Ref: Nelson 18/e p191, 192, Ghai 8/e p39; Nelson 19/e p124 a. B/L frontal bossing (PGI June 04)
47. A male child of 15 years with a mental age of 9 years has an b. Multiple fracture
c. Hepatosplenomegaly

PEDIATRICS
1Q of: (AI 03)
a. 50 d. Cataract
b. 60 e. Mental retardation
c. 70 Ref: Ghai 8/e p587; Nelson 18/e p2882 and 17/e p2333
d. 80 Ref: Ghai 8/e p38 55. Duchenne Muscular Dystrophy is a disease of:
48. Condition associated with MR: (PGI 87) a. Neuromuscular junction (AI 04)
a. Trisomy 21 b. Sarcolemmal proteins
b. Fragile - X c. Muscle contractile proteins
c. Homocystinemia d. Disuse atrophy due to muscle weakness
d. Phenylketonuria Ref: Ghai 8/e p594; 7/e p566-569;
e. Tuberous sclerosis Nelson 18/e p2540 and 17/e p2060-2063;
f. All Ref: Ghai 8/e p39; Has been explained Harrison 16/e p2528-2529; Nelson 19/e p851

Ans. 41. a. Open neural... 42. c. Aqueductal stenosis 43. c. Vascular lesion... 44. a. Post inflammatory...
45. c. Canavans disease 46. b and d 47. b. 60 48. f. All
49. a, c and d 50. e. All 51. b. Muscle fibers 52. d. Myotonic dystrophy
53. a. Uveitis 54. b and c 55. b. Sarcolemmal proteins
84 Jaypees Triple A
56. All of the following are essential features of attention 63. Not found in cerebral palsy: (PGI June 98)
deficit hyperactivity disease (ADHD) except: a. Hypotonicity
a. Lack of concentration (AI 04) b. Microcephaly
b. Impulsivity c. Ataxia
c. Mental retardation d. Flaccid paralysis
d. Hyperactivity Ref: Ghai 8/e p581; 7/e p575; Nelsons Essentials of Pediatrics 5/e, p58;
Ref: Ghai 8/e p584; 7/e p37-38; Nelson 18/e p68-69 and 17/e p107-108 18/e p2494, 2495; Nelson 19/e p2007, 2008
57. Duchennes dystrophy; investigation of choice: 64. A 2 year old child is brought by parents with history
a. Serum creatine (AIIMS Nov 08) of seizures and developmental delay. He has multiple
b. Neure conduct hypopigmented macules over the back. What tis the most
c. ESR probab le diagnosis?
d. CPK a. Neurofibromatosis type 1 (AI 2010)
Ref: Ghai 8/e p594; Nelson 18/e p2540; Nelson 19/e p2123 b. Tuberous sclerosis
58. An infant presents with hypotonia and hyporeflexia. c. Sturge Weber syndrome
During his intrauterine period there was polyhydramnios d. Linear Sebacous Nevus Syndrome
and decreased fetal movements. Most probable diagnosis Ref: Ghai 8/e p586; 7/e p564
is: 65. True about infantile tremor syndrome:
a. Spinal muscular atrophy (AIIMS May 02) a. Hyperpigmentation of extremities
b. Congenital myasthenia b. Fine tremor
c. Congenital myotonia c. Cortical atrophy
d. Muscular dystrophy d. Self-limiting disorder
Ref: Ghai 8/e p593; 7/e p574-575; Nelson 18/e p2557-2559 and e. More common in girls Ref: Ghai 8/e p580; 7/e p558
17/e p2066, 2075, 534; Harrison 16/e p2520; Nelson 19/e p2136-2138 66. A child in status epilepticus should not be given:
59. Give the most probable diagnosis of a 1-year-old child a. Clonazepam (JIPMER 04)
of normal intelligence with features of hypotonia. On b. Phosphenytoin
examination there are tongue fasc iculations and he keeps c. Lamotrigine
his body in a frog like position: (AIIMS Nov 00) d. Diazepam Ref: Ghai 8/e p558; 7/e p524; Nelson 17/e p2008
a. Guillian Barre Syndrome 67. A previously well 2 year old boy presents with a brief
b. Limb girdle atrophy generalized seizure. There is no past or family history of
c. Downs syndrome seizures. On examination, child is alert, active, febrile
d. Spinal muscular atrophy with axillary temperature of 39 40c. Except for bilateral
Ref: Ghai 8/e p595; 7/e p574-575; tonsillar enlargement and congestion, systemic examination
Nelson 18/e p2557-2559 and 17/e p2075; Nelson 19/e p2138 is normal. Regarding the management at this time, consider
60. A 3-year-old child comes with complaint of limp diagnosis the following: (UPSC 06)
is: (PGI Nov 08) 1. Immediate reduction of body temperature
a. Septic arthritis 2. Intrevenous diazepam and phenytoin, followed by a
b. Slipped capital femoral epiphysis lumbar puncture
c. Perthes disease 3. Institution of phenytoin for maintenance therapy for at
d. DOH Ref: Ghai 8/e p584; 7/e p578 least one year
61. Features of cerebral palsy: (PGI June 05) 4. Rectal diazepam every 8 hourly for 2 3 days to reduce
a. Athetosis risk of recurrence.
b. Spasticity Which of the above should be included in the management
PEDIATRICS

c. Saturday night palsy at this time?


d. Mixed palsy a. 1 and 2 only
e. Rigidity Ref: Ghai 8/e p581; 7/e p575 b. 1 and 4 only
c. 3 and 4
62. A Child is suffering from fever, sinusitis, proptosis and
d. 1, 2 and 4 Ref: Ghai 7/e p528, 6/e p508-509
extraocular muscle palsy. Possible diagnosis: (PGI June 03)
a. Cavernous sinus thrombosis 68. 4-year-old male child has febrile seizures. Best prophylaxis
b. Langerhans cell histiocytosis is:
c. Meningitis a. Paracetamol 6 hourly (UPSC 00)
d. Secondary metastasis b. Paracetamol and diazepam
e. Frontal abscess c. Diazepam
Ref: Ghai 8/e p570; Harrison 16/e p2490 d. Phenobarbitone
Ref: Ghai 8/e p556; 7/e p528; CPDT. 15/e p645

Ans. 56. c. Mental retardation 57. d. CPK 58. a. Spinal muscular atrophy 59. d. Spinal muscular atrophy
60. a, b and c 61. a, b, d and e 62. a. Cavernous sinus... 63. b. Microcephaly
64. b. Tuberous sclerosis 65. a, c and d 66. a > c 67. b. 1 and 4 only
68. b and c
Disorders of Neuromuscular System 85
69. Management of typical seizures include except: b. Enterovirus
a. Tepid sponging (Karnataka 05) c. Mumps
b. Paracetamol ibuprofen d. Listeria Ref: Ghai 8/e p721; Nelson 18/e p2521
c. Intermittent diazepam 79. Generalised 3-4 Hz spike and slow wave complexes in EEG
d. Prophylactic phenobarbitone Ref: Ghai 8/e p557; 7/e p528 are seen in: (DNB 2009)
70. Rapid antigen test for meningitis organisms in a child can a. GTC
be done on the following specimens except: (COMED 06) b. Absensce seizures
a. Blood c. Myoclonic epilepsy
b. CSF d. Temporal lobe epilepsy Ref: OP Ghai 8/e p720
c. Urine 80. Proximal muscle weakness is caused by all except:
d. Throat swab Ref: Ghai 8/e p563; 7/e p537 a. Erb duchenee paralysis (DNB 2010)
71. Most common cause of Hydrocephlus in children is: b. Beckers paralysis
a. Post inflammatory obstruction (SGPGI 04) c. Myotonic dystrophy
b. Budd chiari syndrome d. Polymyositis
c. Brain tumor Ref: Ghai 8/e p580; Nelson 18/e p2544
d. Perinatal injury 81. Which of the following is a non-progressive neuro-motor
e. None Ref: Ghai 8/e p574; Nelson 17/e p1990 deficit of central nervous system?
72. Acquired extracarnial infection that causes Aqueductal a. Metachromatic leucodystrophy (MP PG 2008)
Stenosis is: (UPSC 04) b. Cerebral palsy
a. Bacterial endocarditis c. Friederichs ataxis
b. Mumps d. Wilson disease Ref: Ghai 8/e p581; Nelsons 18/e p2494
c. Measles 82. In intraosseous canulation of tibia, the needle is inserted
d. Staphylococcal septicemia into the bone at an angle of: (MP PG 2008)
Ref: Ghai 8/e p209; Harrison 16/e p734 a. 45
73. Which of the following has the worst prognosis: b. 60
a. Rolandic epilepsy (JIPMER 01) c. 75
b. Versive epilepsy d. 90
c. Absence epilepsy Ref: Ghai 8/e p731-732; Pediatric emergency procedures by
d. Infantile spasm Ref: Ghai 8/e p558; CPDT 16/e p730 Christoher King and Fred. M. Henreting, p69
74. Commonest cause of obstructive hydrocephalus in children: 83. Residual auditory defect is a common complication in
a. Acqueductal stenosis (UP 07, 05) meningitis caused by: (Karnataka PG 2011)
b. Aqueductal gliosis a. Staphylococcus
c. Subarachnoid hemorrhage b. Meningococcus
d. Tubercular meningitis c. Escherichia coli
Ref: Ghai 8/e p574; Nelson 17/e, 1990 d. Hemophilus influenza
75. Child with generalized petechiae. CSF shows gram negative Ref: OP Ghai 8/e p564; 7/e p536;
diplococci. Treatment: (PGI June 07) Harrisons 17/e p924
a. IV Ceftriaxone 84. Sign of increased intracranial tension are all except:
b. IV Penicillin G a. Papilledema (UP PG 09)
c. IV Cefotaxime Ref: Ghai 8/e p566; Harrison 16/e, 2476 b. Eighth nerve palsy
76. The behavior therapeutic falls in management of enuresis, c. Dilated scalp vein

PEDIATRICS
The pharmacological drug of choice for this case is: (UP 07) d. Diplopia Ref: Ghai 8/e p570; 7/e p544
a. Phenytoin 85. Drug of choice in status epilepticus: (UP PG 09)
b. Diazepam a. IV phenytoin
c. Imipramine b. Phenolbarbitone
d. Alprax Ref: OP Ghai 8/e p504 c. Ethosuximide
77. Most common cause of meningoencephalitis in children? d. Diazipam Ref: Ghai 8/e p555; 7/e p526
a. HSV 86. Increase intracranial tension causes all except: (AP 2011)
b. Enterovirus a. Tachycardia
c. Mumps b. Papilloedma
d. Listeria Ref: Ghai 8/e p563; Nelson 18/e p2521 c. Headach
78. Most common cause of meningoencephalitis in children is: d. Convulsions
a. HSVI > Enterovirus > HSV2 (DNB 2007) Ref: Ghai 8/e p570

Ans. 69. d. Prophylactic... 70. d. Throat swab 71. a. Post inflammatory... 72. b. Mumps
73. d. Infantile spasm 74. a. Acqueductal stenosis 75. b. IV Penicillin G 76. c. Imipramine
77. b. Enterovirus 78. b. Enterovirus 79. b. Absensce seizures 80. c. Myotonic dystrophy
81. b. Cerebral palsy 82. d. 90 83. d. Hemophilus influenza 84. b. Eighth nerve palsy
85. d. Diazipam 86. a. Tachycardia
86 Jaypees Triple A
87. Management of typical febrile seizures include all the 92. In Arnold Chiari malformation type II there is: (WB PG 08)
following except: (DP PGMEE 2010) a. Herniation of medulla oblongata
a. Tepid sponging b. Hermiation of medulla with cerebellum
b. Paracetamol ibuprofen c. Meningomyelocele
c. Intermittent diazepam d. Aqueductal Stenosis
d. Prophylactic phenobarbitone Ref: Ghai 8/e p574; Nelson 17/e p1990
Ref: Ghai 8/e p556; Nelson 18/e p2457 93. All are congenital myopathies except: (WB PG 08)
88. A 10-month-old child presents with 2 weeks history of a. Nemaline
fever, vomiting and alteration of sensorium. Cranial CT b. Central core
scan reveals basal exudates and hydrocephalus. The most c. Centro nuclear/myotubular
likely etiological agent is: (Feb DP PGMEE 2009) d. Mitochondrial
a. Mycobacterium tuberculosis Ref: Ghai 8/e p594; Harrison 17/e p2688-89
b. Cryptococcus neoformans 94. Hypsarrythmia in EEG is seen in which of the following
c. Listeria monocytogenes conditions: (J & K PGI 2011)
d. Streptococcus pneumoniae Ref: Ghai 8/e p566 a. Infantile spasms
89. In a pregnant woman with raised phenylalanine in blood, b. Absence attacks
offspring is most likely to have: (Kerala PGI 08) c. Partial seizure
a. Mental retardation d. Generalised seizure Ref: OP Ghai 8/e p559
b. Lighter complexion 95. Macewens sign indicates: (J & K PGI 2011)
c. Microcephaly a. Hepatic failure
d. Reduction of limb growth b. Raised intracranial tension Increased Intracranial tension
Ref: Ghai 8/e p652; Nelson 17/e p400; 18/e p531 c. Meningitis
90. Meningitis in a 4 year old child; microsopy of the Gram d. Rickets Ref: OP Ghai 8/e p570
stained CSF sample will reveal: (WB PGI 08) 96. Sub acute Sclerosing Panencephalitis follows an attack of:
a. Gram positive cocci a. Measles (J & K PGI 2011)
b. Gram positive bacilli b. Varicella
c. Gram negative diplococci c. Mumps
d. Gram negative bacilli d. Rubella Ref: Ghai 8/e p584
Ref: Ghai 8/e p563; Harrison 17/e p2622
97. Midline intra- cranial cyst is: (J & K PGI 2010)
91. Hydrocephalus is associated with all except: (WB PG 08) a. Choroid plexus cyst
a. Increased head size b. Dandy-Walker cyst
b. Bulging tense fofntanelle c. Unilateral hydrocephalus
c. Diplopia d. Porencephalic cyst Ref: Ghai 8/e p575
d. Papilledema Ref: Nelson 17/e p1990; Ghai 8/e p574
PEDIATRICS

Ans. 87. d. Prophylactic... 88. a. Mycobacterium... 89. a. Mental retardation 90. a. Gram positive cocci
91. c. Diplopia 92. c. Meningomyelocele 93. d. Mitochondrial 94. a. Infantile spasms
95. b. Raised intracranial 96. a. Measles 97. b. Dandy-Walker cyst
13. METABOLIC DISORDERS
88 Jaypees Triple A

METABOLIC DISORDERS (QUESTIONS)

1. Injection of Glucagon is effective for management of e. Some marrow shows gaucher cells
persistent hypoglycemia in all except: Ref: Ghai 8/e p658-659; 7/e p638; Robbins 7/e p163, 164
a. Large for date baby (AIIMS May 04) 8. Injection Glucagon is effective for management of
b. Galactosemia persistent hypoglycemia in all, except:
c. Infant of diabetic mother a. Large for date baby (AIIMS May 04)
d. Nesidioblastosis Ref: Ghai 8/e p647; Nelson 17/e p505-507 b. Galactosemia
2. Leigh disease is due to the accumulation of: c. Infant of diabetic mother
a. Glycogen (NIMHANS 01) d. Nesidioblastosis Ref: Ghai 8/e p655; 7/e p634;
b. Pyrodoxine Dutta 6/e p287; 18/e p603, 609, 610; Nelson 19/e p503
c. Sphingomyelin 9. Child of Vasanthi was weaned from breast milk on the 5th
d. Lipofuscin day and was given sugarcane juice the child developed
e. None Ref: Ghai 8/e p656; Nelson 16/e p1846-1847 hypoglycemia and hepatomegaly Biochemical examination
3. A child presents with massive hepatomegaly and showed hypophosphatemia and reducing substances in
hypoglycemia. There is no improvement in blood glucose urine. The child is probably suffering from which of the
on administration of Glucagon. The probable diagnosis is: following enzyme deficiencies: (AIIMS Nov 00)
a. Von Gierke disease (AI 09) a. Fructokinase
b. McArdle disease b. Aldolase B
c. Coris disease c. Glucose 6 Phosphatase
d. Forbes disease d. Beta galactosidase Ref: Ghai 8/e p657; 7/e p634, 635;
Ref: Ghai 8/e p647; 7/e p635; Nelson 18/e p602; Nelson 19/e p492 Nelson 18/e p601, 602 and 17/e p476- 477; Nelson 19/e p503
4. For which of the following diseases is enzyme replacement 10. A boy comes with complains of vomiting, bloated abdomen
therapy available? (AI 04) and abdominal pain. He has history of attending ice-cream
a. Albinism eating competition last night. He also past history of similar
b. Neimann-Pick disease episodes following ingestion of milk and milk products.
c. Metachromatic leukodystrophy The likely cause: (AIIMS Nov 99)
d. Gauchers disease cerezyme a. Pancreatic amylase deficiency
Ref: Ghai 8/e p659; 7/e p635; Nelson 17/e p463-464; b. Lactase deficiency
Harrison 16/e p2316-2317; Nelson 19/e p494 c. Salivary amylase deficiency
5. Enzyme replacement therapy is available for which of the d. Food poisoning
following disorders? Ref: Ghai 8/e p657; Nelson 18/e p1554 and 17/e p1268;
a. Gauchers disease (AIIMS May 06) Nelson 19/e p502, 503
b. Niemann-Pick disease 11. An 8 days old child presents with yellow sclera, whitish
c. Mucolipidosis stool and turmeric colour urine on 3rd day of septicemia on
d. Metachromatic leukodystrophy broad spectrum antiboitics, the likely diagnosis is:
Ref: Ghai 8/e p659; 7/e p635; Nelson 18/e p602 and 17/e p463-464; a. Galactosidase deficiency (AIIMS June 98)
Harrison 16/e p2316-2317 b. Ammonia toxicity
6. Coarse facies, hepatosplenomegaly and tall QRS on ECG c. Galactose 1-4 phosphatase uridyl transferase deficiency
are characteristic feature of: d. Glucose 6 phosphatase deficiency
PEDIATRICS

a. Glycogen storage disease type II Ref: Ghai 8/e p656-657; 7/e p634; Nelsons Essentials of
b. Hurlers disease (AIIMS Nov 01) Pediatrics 5/e, 254-255, 18/e p609, 610; Nelson 19/e p502
c. Hunters disease 12. An infant presents with history of seizures and skin rashes.
d. Hemochromatosis Investigations show metabolic acidosis increased blood
Ref: Ghai 8/e p652; 7/e p636; Nelson 18/e p602 and 17/e p474; ketone levels. This child is likely to be suffering from:
Harrison 16/e p2321-2322 a. Propionic aciduria (AI 02)
7. True about Gauchcrs disease: (PGI 2010) b. Urea cyclic disorder
a. PAS positive cells c. Phenylketonuria
b. Oil Red O-He cells d. Multiple carboxylase deficiency
c. Deficiency of acid Sphingomyelinase Ref: Ghai 8/e p652; 7/e p626; Nelson 18/e p451 and 17/e p411;
d. Def. Glucocerebrosidase Nelson 19/e p417

Ans. 1. b. Galactosemia 2. e. None 3. a. Von Gierke disease 4. d. Gauchers disease...


5. a. Gauchers disease 6. a. Glycogen storage... 7. a, d and e 8. b. Galactosemia
9. b. Aldolase B 10. b. Lactase deficiency 11. c. Galactose 1-4... 12. d. Multiple carboxylase...
Metabolic Disorders 89
13. True statement regarding a 3 week old child with Phenyl- c. Fabrys disease
ketonuria is all, except: (AI 00) d. Tyrosinemia
a. Provocative protein meal tests helps in the diagnosis Ref: Ghai 8/e p652; 7/e p630; Nelson 18/e p534; Lippincott 3/e p272
b. Tyrosine becomes an Essential amino-acid in diet 18. In a patient, muscle cramps on exercise, positive
c. Serum Phenylalanine is increased and urinary Phenyl myoglobulinemia, the disorder is:
Pyruvate level is elevated a. Pompes disease (PGI June 98)
d. Phenylalanine should be completely stopped in diet b. Myotonia congenital
Ref: Ghai 8/e p652; 7/e p629; Harrison 17/e p23332334; c. Myotonic dystrophy
Nelson 19/e p418-419 d. Mc Ardles disease
14. Deficiency of enzyme a-keto acid decarboxylase leading to Ref: Ghai 8/e p659; 7/e p636, 637;
a block in the metabolism of branch chain amino acids is Nelsons Essentials of Pediatrics 5/e, p253; 18/e p602
observed in: 19. LEIGH disease is due to the accumulation of:
a. Maple syrup urine disease (AIIMS May 05) a. Glycogen (NIMHANS 01)
b. Hartnups disease b. Pyrodoxine
c. Alkaptonuria c. Sphingomyelin
d. Phenylketonuria d. Lipofuscin
Ref: Ghai 8/e p650; 7/e p632; Nelson 18/e p540-549 and 17/e p409 e. None
15. Treatment of multiple carboxylase deficiency: Ref: Nelson 16/e p1846 and 1847 also see p1884 and 1885.
a. Biotin (AI 07) level of Pyruvate and lactose; Internet
b. Pyridoxine 20. Which glycogen storage disease does not affect muscles?
c. Thiamine a. Type 1 (APPG 08)
d. Folic acid b. Type 2
Ref: Ghai 8/e p653; Nelson 18/e p614 c. Type 3
16. A 5 day old child presents with intractable seizures. He d. Type 4
had rashes all over the body. Blood examination showed Ref: Ghai 8/e p655; 7/e p637; Harrison 17/e p2458
hyperammonemia and lactic acidosis. The probable 21. Massive aminoaciduria without a corresponding increase in
diagnosis is: (AIIMS 2K) plasma amino acid level is characteristic of which one of the
a. Organic aciduria following diseases? (UPSC 108)
b. Mitochondrial encephalopathy with lactic aciduria a. Homocystinuria
c. Phenylketonuria b. Hartnup disease
d. Urea cycle enzyme deficiency c. Tyrosinemia
Ref: Ghai 8/e p655; 7/e p632 d. Maple syrup urine disease
17. Darkening of lurine on standing is associated with: Ref: Nelson 18/e p539, 540; Ghai 8/e p652; 7/e p632
a. Alkaptonuria (AI 07)
b. Cystinuria

PEDIATRICS

Ans. 13. d. Phenylalanine... 14. a. Maple syrup... 15. a. Biotin 16. a. Organic aciduria
17. a. Alkaptonuria 18. d. Mc Ardles disease 19. e. None 20. a. Type 1
21. b. Hartnup disease
14. GENETIC DISORDERS
Genetic Disorders 91

GENETIC DISORDERS (QUESTIONS)

1. Which of the following when present is associated more 8. An affected male infant born to normal parents could be an
than 20% of the time with a chromosomal disorder? example of all of the following, except: (AI 06)
a. Omphalocele (AIIMS Nov 94) a. An Autosomal dominant disorder
b. Gastroschisis b. An Autosomal recessive disorder
c. Spinabifida c. A polygenic disorder
d. Cleft Palate d. A vertically transmitted disorder
Ref: Atlas of genetic diagnosis and counseling, Ref: Ghai 8/e p636; 7/e p617; Nelsons Essentials of
CPDT 19/e p1011; Nelson 19/e p1252 Pediatrics 5/e, p221; Nelson 19/e p383
2. Which one of the following is NOT true regarding Noonans 9. Absent thumb, radial deviation of wrist, bowing of forearm,
syndrome: (COMED 06) which inv. need not to be done?
a. Affects males and females a. Echocardiography (AIIMS Nov 07)
b. Short stature b. Bone marrow examination
c. Chromosomal abnormality c. Platelet count
d. Congenital heart disease ASD d. Karyotyping Ref: Ghai 8/e p645; 7/e p611
Ref: Ghai 7/e p475, 6/e p493; Nelson 17/e p1925 10. All of the following conditions have autosomal dominant
3. The chromosomal disorder in patau syndrome is: (UP 07) inheritance except: (AI 07)
a. Chromosome 21 a. Fabry disease
b. Chromosome 18 b. Marfans syndrome
c. Chromosome 13 c. Osteogenesis imperfecta
d. Chromosome 45x / 46xx Ref: OP Ghai 8/e p643 d. Ehlers Danlos syndrome
Ref: Ghai 8/e p638; Robbins 7/e p151
4. Which one of the following is a distinguishing feature of
11. The chances of having an unaffected baby, when both
Edwards syndrome: (UPSC 07)
parents have achondroplasia, are:
a. Hypotonia
a. 0%
b. Hypotelorism
b. 25% (AI 05, AIIMS May 04)
c. Holoprosencephaly
c. 50%
d. Rocker bottom feet Ref: Ghai 8/e p638; Nelson 17/e, 384
d. 100% Ref: Ghai 8/e p642; Robbins 7/e p160, 161
5. Only male are affected in: (AIIMS May 08)
12. In family, the father has widely spaced eyes, increased facial
a. Scheies syndrome
hair and deafness. One of the three children has deafness with
b. Hunters syndrome
similar facial features. The mother is normal. Which one of the
c. Hurlers syndrome following is least likely pattern of inheritance in this case:
d. Gauchers disease a. Autosomal dominant (AI 06)
Ref: Ghai 8/e p662; 7/e p635; CPDT 18/e p1006, 1007; b. Autosomal recessive
Nelson 19/e p511-512 c. X-linked dominant
6. For which of the following diseases is enzyme replacement d. X-linked recessive Ref: Ghai 8/e p641
therapy available: 13. A parent is homozygous and a parent heterozygous for an
a. Albinism (AI 04, AIIMS May 06, Nov 03) autosomal recessive gene. What will be the outcome:
b. Neimann-Pick disease a. 75% children affected (AIIMS May 94)

PEDIATRICS
c. Metachromatic leukodystrophy b. No child affected, but all are carriers
d. Gauchers disease c. 50% children affected, rest are carriers
Ref: Ghai 8/e p659; Nelson 18/e p595, 596 d. 25% children affected, rest are carriers
7. A child with a small head, minor anomalies of the face Ref: Ghai 8/e p636; Robbins 7/e p161
including a thin upper lip, growth delay, and developmental 14. For a normal husband and wife the first child was diagnosed
disability can have all of the following, except: (AI 06) to have cystic fibrosis. What is the percentage of chances for
a. A chromosomal syndrome the second child be affected: (PGI June 06)
b. A teratogenic syndrome a. 25
c. A mendelian syndrome b. 50
d. A polygenic syndrome c. 0
Ref: 8/e p635; Nelsons Essentials of Pediatrics 5/e, p217 d. 75
Nelson 19/e p393, 394 e. 100 Ref: Robbins 7/e p151; Nelson 19/e p385

Ans. 1. a. Omphalocele 2. c. Chromosomal... 3. c. Chromosome 13 4. d. Rocker bottom feet


5. b. Hunters syndrome 6. d. Gauchers disease 7. d. A polygenic syndrome 8. a. An Autosomal...
9. d. Karyotyping 10. a. Fabry disease 11. b. 25% 12. a. Autosomal dominant
13. c. 50% children... 14. c. 0
92 Jaypees Triple A
15. An albino girl gets married to a normal boy, What are the 22. Single gene defect causing multiple unrelated problems:
chances of their having an affected child and what are the a. Pleotropism (AIIMS Nov 06)
chances of their children being carriers? (AI 03) b. Pseudo dominance
a. None affected, all carriers c. Penetrance
b. All normal d. Anticipation Ref: Nelsons Essentials of Pediatrics 5/e, p221
c. 50% carriers 23. True in Klinefelter syndrome is: (AI 98)
d. 50% affected, 50% carriers a. Short stature
Ref: Ghai 8/e p636; Robbins 7/e p151; Nelson 19/e p385 b. Pituitary adenoma
16. In an Autosomal Recessive (AR) disorder, one parent is c. Subnormal intelligence
normal and the other is carrier and the child is also affected. d. Breast adenoma
What is the reason: (AI 07) Ref: Ghai 7/e p615, 6/e p594; Harrison 16/e p2214-2215
a. Germ line mosaicism 24. A baby presenting with multiple deformities, cleft lip,
b. Genomic imprinting cleft palate, microcephaly, small eyes, scalp defect and
c. Penetrattion polydactyly, seen in which patau syndrome:
d. Uniparental disomy Ref: Ghai 8/e p641; Nelson 19/e p392 a. Trisomy 13 (AIIMS Nov 06, May 08)
17. Fragile X-syndrome is characterized by all of the following b. Trisomy 18
features except: (AI 99) c. Trisomy 21
a. Long face d. Monosomy 2
b. Large ear Ref: Robbins 7/e p176, 177, OP Ghai 7/e p615; 6/e p593; Nelson 19/e p404
c. Large-nose 25. Cat eye syndrome is: (AI 07)
d. Large-testis a. Partial trisomy 18
Ref: Nelson 18/e, 135, 499 and 17/e p388-389; Internet b. Partial trisomy 13
18. Kinky hair desease is disorder where an affected child c. Partial trisomy 21
has peculiar white stubby hair, does not grow, brain d. Partial trisomy 22 Ref: Dorlands 28/e, p162
degeneration is seen and dies by age of two years. Mrs A is 26. A Down syndrome patient is posted for surgery, the
hesitant about having children because her two sisters had necessary preoperative investigation to be done is:
sons who had died form kinky hair disease. Her mothers a. Echocardiography (AI 08)
brother also died of the same condition. Which of the b. CT brain
following is the possible mode of inheritance in her family: c. X-ray cervical spine
a. X-linked recessive (AI 04) d. USG abdomen
b. X-linked dominant Ref: Ghai 8/e p638; 7/e p613; Nelson 18/e p507-509; Nelson 19/e p402
c. Autosomal recessive
27. All of the following may occur in Downs syndrome except:
d. Autosomal dominant
a. Hypothyroidism (AI 06)
Ref: Ghai 8/e p643; Robbins 7/e p152; Nelson 19/e p388
b. Undescended testis
19. The process underlying differences in expression of a gene, c. Ventricular septal defect
according to which parent has transmitted, is called: d. Brushfields spots
a. Anticipation (AI 06) Ref: Ghai 8/e 8/e p637-638; 7/e, 613, 614; Nelson 18/e, 507-509 and17/e
b. Mosaicism p384; Robbins illustrated 7/e p176-177; Nelson 19/e p402
c. Nonpenetrance
28. Transient myeloproliferative disorder of the newborn is
d. Genomic imprinting
seen in association with: (AI 03)
Ref: Ghai 8/e p641; Harrison 16/e p375; Robbins 7/e p186;
a. Turner syndrome
Nelson 19/e p391
b. Downs syndrome
PEDIATRICS

20. Differential expression of same gene depending on parent c. Neurofibromatosis


of origin is referred to as: (AI 05, 06) d. Ataxia telangiectasia
a. Genomic imprinting Ref: Ghai 8/e p599; Nelson 18/e p507-509 and 17/e p1697;
b. Mosaicism Nelson 19/e p401, 402
c. Anticipation
29. Triple test for diagnosis of downs syndrome includes all of
d. Nonpenetrance
the following except: (AI 99)
Ref: Ghai 8/e p641; Robbins 7/e p186; Nelson 19/e p391
a. P-HCG
21. Atavism means child resembles with his: b. a-Fetoprotein
a. Father (AIIMS Nov 99) c. Serum HPL level
b. Siblings d. Serum oestriol level
c. Grand parents Ref: Ghai 8/e p638; Dutta 5/e, p112; Nelson 19/e p401, 402
d. Neighbor Ref: Tabers Medical Dictionary

Ans. 15. a. None affected... 16. d. Uniparental disomy 17. c. Large-nose 18. a. X-linked recessive
19. d. Genomic imprinting 20. a. Genomic imprinting 21. c. Grand parents 22. a. Pleotropism
23. c. Subnormal 24. a. Trisomy 13 25. d. Partial trisomy 22 26. a. Echocardiography
27. b. Undescended testis 28. b. Downs syndrome 29. c. Serum HPL level
Genetic Disorders 93
30. All of the following are features of downs syndrome 37. Posterior iliac horns are seen in:
except: (AI 05) a. Fishers syndrome (AIIMS May 02)
a. Increased PAPPA b. Crouzan syndrome
b. Increased free beta HCG levels c. Nail patella syndrome
c. Absent nasal bone d. Pierre Robbins syndrome
d. Abnormal ductus venous flow velocity Ref: Nelson 18/e p2731-2882 and 17/e p2220, 2332;
Ref: Ghai 8/e p638; Nelson 18/e p567-509 and 17/e p535; Nelson 19/e p2293, 2294; Internet
Dutta, 6/e p107; Nelson 19/e p402 38. A 4-year-baby is having large face, large jaw, large ear and
31. A 35 years old lady has chromosomal translocation 21/ 21. macro orchidism is: (AMU 05)
The risk of Downs syndrome in the child is: a. Mc Cune-Albright syndrome
a. 100% (AIIMS June 99) b. Downs syndrome
b. 0% c. Cri-du chat syndrome
c. 10% d. Fragile X syndrome
d. 50% Ref: Ghai 8/e p634; 7/e p6
Ref: Ghai 8/e p637; Nelson 18/e p490, 509 and 17/e p387; 39. Downs syndrome predisposes to....cancer:
Nelson 19/e p402 a. AML (MAHE 05)
32. Downs syndrome most commonly occurs due to? b. CML
a. Reciprocal translocation (AI 2010) c. ALL
b. Nondysjunction in maternal meiosis d. CLL
c. Translocation defect Ref: Ghai 8/e p599; 7/e p613, 614
d. Nondysjunction in paternal meiosis 40. In Downs symdrome the shape of the head is:
Ref: Ghal 8/e p637; 7/e p630; H-17/Table 63-3; a. Oxycephalic (MAHE 05)
Nelson 19/e p402 b. Scaphocephalic
33. Webbing of neck, increased carrying angle, low posterior c. Brachicephalic
hair line and short forth metacarpal are characteristics of: d. Plagiocephalic
a. Klinefelter syndrome (AI 04) Ref: Ghai 8/e p637; 7/e p613
b. Turners syndrome 41. The chromosomal disorder in patau syndrome is:
c. Cri-du-chat syndrome a. Chromosome 21 (UP 07)
d. Noonans syndrome. b. Chromosome 18
Ref: Ghai 8/e p536, 640; Nelson 18/e p2386-2389; c. Chromosome 13
Robbins illustrated 7/e p179-180; Nelson 19/e p409 d. Chromosome 45x /46xx Ref: Ghai 8/e p640; 7/e p615
34. All of the following may occur in Noonans syndrome 42. Which one of the following is a distinguishing feature of
except: (AI 03) Edwards syndrome: (UPSC 07)
a. Hypertrophic cardiomyopathy a. Hypotonia
b. Cryptorchidism b. Hypotelorism
c. Infertility in females c. Holoprosencephaly
d. Autosomal dominant transmission d. Rocker bottom feet Ref: Ghai 8/e p635, 636; 7/e p614
Ref: Ghai 8/e p537; Nelson 18/e p514 and 17/e p1935, 1925
43. Which one of the following is NOT true regarding Noonans
35. Turner syndrome is maximally associated with: syndrome: (COMED 06)
a. Horseshoe kidney (AIIMS May 08) a. Affects males and females
b. Coarctation of aorta b. Short stature
c. VSD

PEDIATRICS
c. Chromosomal abnormality
d. ASD d. Congenital heart disease - ASD
Ref: Ghai 8/e p536; Nelson 17/e p2387 Ref: Ghai 8/e p636; 7/e p616
36. A patient with short stature, sexual infantilism and 44. A particular genetic disorder appears in three consecutive
congenital anomalies with chromosomal abnormalities generations of a family without any sex predilection.
XO. Diagnosis is: (PGI June 04) It was also noticed that phenotypically normal family
a. Turners syndrome members were have helthy offspring. What is the pattern of
b. Klinefelter syndrome inheritance of this disorders?
c. Testicular feminization slyndrome a. Autosomal recessive (UPSC 06)
d. Gonadal agenesis b. Autosomal dominant
e. Gonadal dysgenesis c. Mitochondrial inheritance
Ref: Ghai 8/e p640; See previous question d. Uniparental disomy Ref: Ghai 8/e p636

Ans. 30. a. Increased PAPPA 31. a. 100% 32. b. Nondysjunction... 33. b. Turners syndrome
34. c. Infertility in females 35. b. Coarctation of aorta 36. a. Turners syndrome 37. c. Nail patella syndrome
38. d. Fragile X syndrome 39. a. AML 40. c. Brachicephalic 41. c. Chromosome 13
42. d. Rocker bottom feet 43. c. Chromosomal... 44. b. Autosomal dominant
94 Jaypees Triple A
45. Maternal disomy of chromosome 15 is seen in: b. Normal gonads
a. Prader-Willi syndrome c. Normal intelligence
b. Klinefelters syndrome d. Long stature
c. Angelman syndrome Ref: OP Ghai 8/e p547; Nelson essentials of Pediatrics 4/e p143, 210, 727
d. Turners syndrome 53. Single gene defect causing multiple unrelated problems:
Ref: Ghai 8/e p641; Nelson, Textbook of a. Pleiotropism (DNB 2009)
Pediatrics, 18/e p498, 513, 516 b. Pseudodominance
46. A child presents with faliure to thrive with frequent c. Penetrance
vomiting, diarrhea, hepatic splenomegaly and abdominal d. Anticipation
distension. CT shows adrenal calcification. Which of the Ref: NMS Medicine, Genetics
following is the diagonosis? 54. All of the following are features of downs syndrome,
a. Adrenal hemorrahage except: (DNB 2009)
b. Wolmans disease a. Increased PAPPA
c. Pheochromocytoma b. Increased free beta HCG levels
d. Addisons disease c. Absent nasal bone
Ref: Current Diagnosis and Treatment Pediatrics 19/e p2009 d. Abnormal ductus venous flow velocity
47. All are true regarding Dents disease except? Ref: Ghai 8/e p638; Nelson essentials of Pediatrics 4/e p13, 18, 141, 142
a. Proteinura 55. Turner syndrome is maximally associated with:
b. Hypercalciuria a. Pancake kidney (DNB 2009)
c. Father will have the same disease b. Coarctation of aorta
d. Rickets is present c. VSD
Ref: Nelson Textbook of Pediatrics, 18/e p519 d. ASD
48. Duchenne muscular dystrophy is transmitted as: Ref: OP Ghai 8/e p524;
a. X-linked dominant (DNB 2005) Nelson essentials of Pediatrics 4/e p143, 210, 727
b. X-linked recessive 56. A neonate with recurrent infection and abscess was
c. Autosomal dominant diagnosed with Kostmann syndrome. Treament is:
d. Autosomal recessive a. G-CSF (DNB 2010)
Ref: Ghai 8/e p522; Kumar Kotran, Oxford Medicine, Philip Kalra, b. GM-CSF
CMDT, NMS Medicine, Harrison c. Antithymocyte globulin+cyclosporin
49. Unconjugated hyperbilirubinemia in newborn is caused by d. Antithymocyte globulin+cyclosporin +GM-CSF
following, except: (DNB 2006) Ref: Campbell Text book of Peds, 10/e p1259; Internet
a. Breast milk jaundice 57. Which of the following is associated with >20% risk of
b. Dubin Johnnson Syndrome chromosomal anomalies: (DNB 2010)
c. Sphereocytosis a. Cleft lip
d. Crigler najjar syndrome b. Gastroschisis
Ref: Ghai 8/e p433 c. Omphalocele
50. All of the following are features of downs syndrome, d. Spina bifida
except: (DNB 2008) Ref: Nelson Essentials of Pediatrics, 4/e p199, 211, 493, 201,
a. Increased PAPPA Updates on associations of Omphalocele
b. Increased free beta HCG levels 58. Death in Klienfilters syndrome is usually because of:
c. Absent nasal bone a. Respiratory causes (DNB 2011)
d. Abnormal ductus venous flow velocity b. Cardiovascular causes
PEDIATRICS

Ref: Ghai 8/e p410; Nelson essentials of c. Cerebrovascular causes


Pediatrics 4/e p13, 18, 141, 142 d. Infections
51. Turner syndrome is maximally associated with: Ref: Ghai 8/e p732; Esssentials of Pediatrics,
a. Pancake kidney (DNB 2008) Nelson 4/e p143, 144, 647, Harrison 17/e p2340, 2341
b. Coarctation of aorta 59. The karyotype associated with the classic pattern of
c. VSD Klienfelters syndrome is: (MP PG 2010)
d. ASD a. 47, XXY
Ref: OP Ghai 8/e p524, Nelson essentials of b. 45, X
Pediatrics 4/e p143, 210, 727 c. 47, XX
52. True about Turners Syndrome is: (DNB 2009) d. 47, XX, + 21
a. Normal breast Ref: Ghai 8/e p640; Nelsons 18/e p514

Ans. 45. a. Prader-Willi 46. b. Wolmans disease 47. c. Father will have... 48. b. X-linked recessive
49. b. Dubin Johnnson... 50. a. Increased PAPPA 51. b. Coarctation of aorta 52. c. Normal intelligence
53. a. Pleiotropism 54. a. Increased PAPPA 55. b. Coarctation of aorta 56. a. G-CSF
57. c. Omphalocele 58. b. Cardiovascular... 59. a. 47, XXY
Genetic Disorders 95
60. A two years old girl child is brought to the out patient d. Ataxia telangiectasia
with features of hand wringing, stereotype movements, Ref: Ghai 8/e p637; Nelson 18/e p2122
impaired stereotype movements, impaired language and 67. Acute leukemia in children is associated with:
communication development, breath holding spells, poor a. Downs syndrome (Feb DP PGMEE 2009)
social skills and deceleration of head growth after six b. Klinefelters syndrome
months of age. The most likely diagnosis is: (MP PG 2010) c. Marfans syndrome
a. Asperger syndrome d. Turners syndrome Ref: Ghai 8/e p599
b. REtts syndrome
68. All are features of Di George syndrome except:
c. Fragile-X syndrome
a. Cardiac anomaly (Kerala PG 08)
d. Cottard syndrome Ref: Nelsons 18/e p2504
b. Facial dystrophy
61. Increased nuchal fold thickness and reduced length of c. Cell mediated immune deficiency
femur in a fetus by prenatal ultrasound predicts: d. Hypercalcaemia
a. Trisomy 21 (MP PG 2008) Ref: Ghai 8/e p185; Harrison 16/e p1942; 17/e p2057;
b. Turners syndrome Nelson 17/e p211, 694, 735; 18/e p884, 885
c. Trisomy 18
69. Feature of Down Syndrome: (Kerala PG 09)
d. Trisomy 13
a. 18 trisomy
Ref: Ghai 8/e p637; Nelsons 18/e p509-509
b. 14/21 translocation
62. The gene for Wilsons disease is on: (MHPGM-CET 2010) c. 21 trisomy
a. Long arm of Chromosome 13 d. Trisomy 13 Ref: Nelson 18/e p637; 508
b. Long arm of Chromosome 6
70. Myeloproliferative disorders (transient?) occur with
c. Short arm of Chromosome 13
increased incident in children with: (WB PG 08)
d. Short arm of Chromosome 6
a. Downs syndrome
Ref: OP Ghai 8/e p320; 7/e p291; Robbins pathology 7/e p910
b. Turners syndrome
63. Which of the following is common in the children of c. Ataxia telanglectasia
pregnant woman age is above 40 years of age? (AP 2010) d. Neuro fibromatosis Ref: Ghai 8/e p637; Nelson 17/e p1697
a. Klinefelters syndrome
71. Fragile X syndrome is associated with all except:
b. Turners syndrome
a. Large nose (WB PG 08)
c. Marfans syndrome
b. Large face
d. Downs syndrome Ref: Ghai 8/e p637
c. Large ears
64. A 50-year-old boy with Downs syndrome is found to have d. Large testis Ref: Nelson 17/e p388
21/21 Translocation. The chromosomal analysis of the
72. Down syndrome diagnosed by triple test with all except:
parents revealed the father to have a balanced translocation.
a. Alpha Feto Protein (UP PG 2010)
The mother has missed a period and the parents want to
b. HCG
know the risk for the fetus: (AP 2012)
c. Estriol
a. 15%
d. Esteriol
b. 25%
Ref: Ghai 8/e p637; 7/e p614
c. 40%
d. 100% Ref: Ghai 8/e p637 73. The gene for ataxia telangiectasia is localized to
chromosome: (J & K PG 2011)
65. Wiskott-Aldrich syndrome is characterized by all of the
a. 11 q
following, except: (AP 2012)
b. 12 p
a. X-linked recessive mode of inheritance
c. 21 p
b. Atopic dermatitis and eczema during infancy

PEDIATRICS
d. 18 q Ref: Ghai 8/e p580
c. Thrombocytopenia
d. IgM is decreased Ref: AP2012; WB2006; 74. Karotyping of a healthy, normal male baby with monglian
spots is: (J & K PG 2010)
66. Transient myeloproliferative disorder of the newborn is
a. 46 XY
seen in association with: (DP PGMEE 2010)
b. 46XX
a. Turners syndrome
c. Trisomy XYn
b. Downs syndrome
d. 45 XO Ref: Ghai 8/e p679
c. Neurofibromatosis

Ans. 60. b. REtts syndrome 61. a. Trisomy 21 62. a. Long arm of... 63. d. Downs syndrome
64. d. 100% 65. d. IgM is decreased 66. b. Downs syndrome 67. a. Downs syndrome
68. d. Hypercalcaemia 69. b and c 70. a. Downs syndrome 71. a. Large nose
72. d. Esteriol 73. a. 11 q 74. a. 46 XY
15. CHILDHOOD MALIGNANCIES
Childhood Malignancies 97

CHILDHOOD MALIGNANCIES (QUESTIONS)

1. Which of the following statements about cerebellar b. Most tumors are below the tentorium
astrocytomas in pediatric age group is False: c. Hemiparesis is a frequent form of presentation
a. These are usually Low grade tumors (AI 08) d. Papilledema is infrequent
b. These tumors have a good prognosis Ref: Ghai 8/e p610; 7/e p545-546
c. These are more commonly seen of the 1st and 2nd 8. Commonest location for craniopharyngioma is:
decades a. Intrasellar (SGPGI 05)
d. These tumors are more common in females b. Suprasellar
Ref: Ghai 8/e p612; NTBP 18/e p2130; Nelson 19/e p1747 c. Intraventricular
2. True statement regarding Brain Tumor in children is: d. Intracerebral Ref: Ghai 8/e p612; 7/e p546
a. Mostly is infra-tentorial (AI 00) 9. Commonest childhood malignancy is: (AI 07)
b. Papilledema is rare a. Leukemia
c. Is the most common tumor in children b. Lymphoma
d. Hydrocephalus is rare c. Neuroblastoma
Ref: Ghai 8/e p612, 613; Nelson 18/e p2487-2492; d. Wilms tumor
Nelson 19/e p1746, 1747 Ref: Ghai 8/e p599; 7/e p580; Nelson 18/e p2116-2122;
3. Common posterior-cranial fossa tumors include all of the Nelson 19/e p1732
following except: (AI 99) 10. All of the following is associated with good prognosis in
a. Medulloblastoma childhood leukemia except: [AI 07]
b. Oligodendroglioma a. Common ALL subtype.
c. Ependydoma b. Precursor B cell ALL
d. Cystic astrocytoma c. Hyperdiploidy.
Ref: Ghai 8/e p610; Nelsons Essentials of Pediatrics 5/e p742; d. Female gender
18/e p2132; Nelson 19/e p1747 Ref: Ghai 8/e p599; 7/e p580-581; Nelson 18/e p2116-2122 and 17/e
4. A 10 year old boy presents with midline swelling arising p1696;Robbins illustrated 7/e, 673; Nelson 19/e p1732, 1733
from cerebellum. The diagnosis is: 11. A 5 year old child presents with history of fever off and
a. Astrocytoma (AIIMS Dec 98) on for past 2 weeks and petechial spots all over the body
b. Glioblastoma multiforme and increasing pallor for past 1 month. Examination reveals
c. Ependymoma splenomegaly of 2 cm below costal margin. The most likely
d. Medulloblastoma diagnosis is: (AI 04)
Ref: Ghai 8/e p613; Harrison 16/e p24522455; Nelson 19/e p1748 a. Acute leukemia
5. Infantile polycythemia is seen in: (PGI June 98) b. Idiopathic thrombocytopenic purpura
a. Cerebellar hemangioblastoma c. Hypersplenism
b. Retinoblastoma d. Aplastic anemia
c. Hepatoblastoma Ref: Ghai 8/e p596; 7/e p584-585; Nelsons Essentials of Pediatrics 5/e,
d. AllRef: Ghai 8/e p623; Robbins 7/e p649; Nelson 18/e p2487 p737; 18/e p2116-2120; Nelson 19/e p1732
6. A 6 year old boy has been complaining of headache, 12. Most common malignancy in children is: (AI 99)
ignoring to see the objects on the sides for four months. a. Retinoblastoma

PEDIATRICS
On examination he is not mentally retarded, his grades at b. Lymphoma and leukemia
school are good, and visual acuity is diminished in both the c. Wilms tumor
eyes. Visual charting showed significant field defect. CT d. Neuroblastoma
scan of the head showed suprasellar mass with calcification. Ref: Ghai 8/e p599; 7/e p580; Nelson 18/e p2116-2120;
Which of the following is the most probable diagnosis? Nelson 19/e p1732, 1733
a. Astrocytoma (AIIMS Sep 96) 13. Poor prognostic indicator of ALL is:
b. Craniopharyngioma a. Female sex [AIIMS May 02]
c. Pituitary adenoma b. Leukocyte count < 50,000
d. Meningioma Ref: Ghai 8/e p613; Has been explained c. Age greater than 1 year
7. Which one of the following statements is true of brain d. Hypodiploid
tumors in childhood? (SGPGI 04) Ref: Ghai 8/e p599, 600; 7/e p581; Nelson 18/e p2116-2122 and 17/e
a. Is a rare from of malignancy p1696; Robbins illustrated 7/e p673

Ans. 1. d. These tumors ... 2. a. Mostly is ... 3. b. Oligodendroglioma 4. d. Medulloblastoma


5. a. Cerebellar... 6. b. Craniopharyngioma 7. b. Most tumors... 8. b. Suprasellar
9. a. Leukemia 10. b. Precursor B cell ALL 11. a. Acute leukemia 12. b. Lymphoma
13. d. Hypodiploid
98 Jaypees Triple A
14. X-ray features of leukemia in a 2 year old child is/ are: 21. The most common malignant neoplasm of infancy is:
a. Osteolytic lesions in flat bones (PGI Dec 00) a. Malignant teratoma (AI 05)
b. Subperiosteal erosions b. Neuroblastoma
c. Osteoporosis c. Wilms tumor
d. Thick line just below growth plate d. Hepatoblastoma
e. Metaphyseal infarcts Ref: Ghai 8/e p599; Sutton 6/e p2001 Ref: Ghai 8/e p616; 7/e p590; Nelson 18/e p2137-2139 and
15. All of the following are good progonostic factor for All 17/e p1709; Nelson 19/e p1754
except: (AIIMS Nov 08) 22. Most common presentation of neuroblastoma is:
a. Age of onset between 2-8 years a. Lytic lesion in skull with suture diasthesis
b. Initial WBC count less than 50000 b. Lung metastasis [AI 98]
c. Hyperdiploid c. Renal invasion
d. t (9: 22), t (8:14), t (4:11) d. Secondaries in brain
Ref: Ghai 8/e p599; Sutton 7/e p1327, 6/e p2001; Nelson 19/e p1733 Ref: Ghai 8/e p616; 7/e p590; Nelson 18/e p2137-2139 and
16. A 2 year old boy suffering from leukemia, following are the 17/e p1709-1710; Nelson 19/e p1754
x-ray findings: (PGI June 03) 23. Neuroblastoma originates from: (PGI June 08)
a. Osteolytic lesion in flat bones a. Adrenals
b. Metaphyseal osteoporosis b. Mediastinum
c. Periosteal new bone formation c. Chest wall
d. Osteosclerosis of long bones d. Peripheral nerves
e. Transverse line of dark band below the growth plate e. Neck
Ref: Ghai 8/e p608; See above explanation Ref: Ghai 8/e p616; 7/e p590-591; Robbin S 7/e p500; Nelson 19/e p1754
17. All of the following syndromes are associated with AML 24. Childhood malignancy producing proptosis is/are:
except: (AI 07) a. Neuroblastoma (PGI June 05)
a. Downs syndrome b. Hepatoma
b. Klinefelters syndrome c. Retinoblastoma
c. Patau syndrome d. Germ cell tumor
d. Turners syndrome e. Nephroblastoma Ref: Ghai 8/e p614-620; 7/e p590-591;
Ref: Ghai 8/e p606; Harrson 16/e p631; Nelson 19/e p1738 Khurana 4/e, p380; Nelson 19/e p1754
18. Transient myeloproliferative disorder of the newborn is 25. To which of the following events is good outcome in
seen in association with: neuroblastoma associated: (AI 04)
a. Turner syndrome a. Diploidy
b. Down syndrome b. N-myc amplification
c. Neurofibromatosis c. Chromosome Ip depletion
d. Ataxia telangiectasia d. Trk A expression Ref: Ghai 8/e p616; Robbins 7/e p763
Ref: Ghai 8/e p599; Nelson 18/e p2122; Nelson 19/e p1738 26. Staging for Wilms syndrome?  (AI 09)
19. Which of the following childhood tumors most frequently a. Chadwick
metastasizes to the bone: (AI 02) b. International staging international society of pediatrics
a. Neuroblastoma (ISOP)
b. Ganglioneuroma c. AJCC
d. TNM
PEDIATRICS

c. Wilms tumor
d. Ewings Sarcoma Ref: International Society of Pediatric. Oncology (ISOP); Nelson 19/e
Ref: Ghai 8/e p616; Nelson 17/e p17091713; Nelson 19/e p1753, 1754 p1757, 1758; Ghai 8/e p617
20. A malignant tumor of childhood, that metastasizes to bones 27. The most common presentation of a child with Wilms
most often, is: (AI 06) tumor is:  (AI 05)
a. Wilms tumor a. As asymptomatic abdominal mass
b. Neuroblastoma b. Hemaruria
c. Adrenal gland tumors c. Hypertension
d. Hemoptysis due to pulmonary secondary
d. Granulose cell tumor of ovary
Ref: Ghai 8/e p617; 7/e p591-592; Nelson 18/e p2140-2143 and 17/e p1712;
Ref: Ghai 8/e p616; 7/e p590; Nelson 18/e p2137-2139 and
Nelson 19/e p1758
17/e p1709-1713; Nelson 19/e p1753-1755

Ans. 14. b. c. and d. 15. d. t (9: 22), t (8:14), t (4:11) 16. b. c. and e 17. d. Turners syndrome
18. b. Down syndrome 19. a. Neuroblastoma 20. b. Neuroblastoma 21. b. Neuroblastoma
22. a. Lytic lesion in... 23. a. b. d. and e 24. a and c 25. d. Trk A expression
26. b. International... 27. a. As asymptomatic
Childhood Malignancies 99
28. Presence of Nephroblastomatosis in a biopsy specimen 34. Pilocytic astrocytoma false is:  (AIIMS May 09)
from Wilms Tumor indicates? (AIIMS Nov 09) a. Spreads to posterior fossa
a. Chance of LN involvement increased b. Seen at eighty years of age
b. Chance of Wilms tumor in Left kidney increased c. Best prognosis among all intracranial neoplasms.
c. Poor prognosis d. Surgery and radiation therapy used for treatment
d. The child could have Denys-Drash syndrome Ref: Ghai 8/e p612; Harrisons 17/e p2602-2603
(Ref: Nelson Textbook of Paediatrics, 18th edition; American journal, 35. Chang staging is used for:  (AIIMS May 2010)
Nephroblastomatosis: Clinicopathologic Significance and Imaging a. Retinoblastoma
Characteristics; National Center for Biotechnology Information; Nelson b. Medulloblastoma
19/e p1758, 1759; Ghai 8/e p617 c. Ewings sarcoma
29. A 7-year old boy with left renal mass had bone pain and d. Rhabdomyosarcoma
was detected to have bone metastatic deposits. The most Ref: Ghai 8/e p612; Nelson 19/e p1751, 1752
likely renal tumor is: (AIIMS Nov 04) 36. Which of the following childhood tumors show extracranial
a. Favorable histology Wilms tumor metastasis?  (AIIMS May 2010)
b. Renal cell carcinoma a. Glioblastomamultiforme
c. Clear cell sarcoma b. Medulloblastoma
d. Rhabdoid tumor c. Choroid plexus carcinoma
Ref: Ghai 8/e p617; Nelson 18/e p2140, 2141; Nelson 19/e p1758 d. Ependymoblastoma Ref: Ghai 8/e p612
30. The most important determinant of prognosis in Wilms 37. What are the cancer in children group: (PGI Nov 09)
tumor:  (AI 06) a. Wlims
a. Stage of disease b. Embryonal rhabdomyosarcoma
b. Loss of heterozygosity of chromosome Ip c. Yolk cell tumor
c. Histology Ref: Ghai 6/e p561; Robbins 7/e p111; Nelson 19/e p1751, 1752
d. Age less than one year at presentation
38. The most characteristic radiographic sign in a child with
Ref: Ghai 8/e p617; Nelson 18/e p2143; Nelson 19/e p1759
leukemia is:  (COMED 06)
31. Which of the following is the most common inherited a. Osteosclerosis of the metaphysic
malignancy ?  (AIIMS May 05) b. Metaphyseal translucencies
a. Infant leukemia c. Periosteal reaction
b. Retinoblastoma d. Osteolytic lesion Ref: Ghai 8/e p599; Sutton 7/e p1327
c. Wilms tumor
39. Which is the commonest childhood tumor: (JIPMER 03)
d. Neuroblastoma
a. ALL
Ref: Ghai 8/e p614; 7/e p594; Robbins illustrated 7/e p284-287; Nelson
b. CLL
18/e p2151, 2152; Nelson 19/e p1768, 1769
c. AML
32. Most common cause of mass in posterior mediastinum in d. CML Ref: Ghai 8/e p599
children :  (PGI June 01)
40. Genetic risk factors for leukemia are all except: (Kerala 04)
a. Rhabdosarcoma
a. Downs syndrome
b. Duplication cyst of esophagus
b. Bloom syndrome
c. Lymphoma
c. Ataxia telangiectasia

PEDIATRICS
d. Neuroblastoma
d. Turners syndrome Ref: Ghai 8/e p598
e. Thymoma
Ref: Ghai 8/e p616; Bailey and Love 24/e p790; Nelson 18/e p2137; Nelson 41. CML in children is associated with:  (NIMH. 01)
19/e p1753 a. Downs syndrome
b. Klinefelter syndrome
33. Commonest tumor of face in children is: (PGI Dec. 99)
c. Marrans syndrome
a. Rhabdomyosarcoma
d. Turners syndrome Ref: Ghai 8/e p608
b. Sq. cell carcinoma
c. Basal cell carcinoma 42. Retinoblastoma gene is located on: (TNPSC 00)
d. Mixed parotid tumor a. Chromosome 5
Ref: Ghai 8/e p618; Nelson 18/e p2144 and 17/e p1714; Nelson 19/e p1760- b. Chromosome 8
1762 c. Chromosome 13
d. Chromosome 16 Ref: Ghai 8/e p614; Nelson 17/e p1722

Ans. 28. b. Chance of Wilms... 29. c. Clear cell sarcoma 30. a. and c. 31. b. Retinoblastoma
32. d. Neuroblastoma 33. a. Rhabdomyosarcoma 34. b. Seen at eighty... 35. b. Medulloblastoma
36. b. Medulloblastoma 37. a and b 38. b. Metaphyseal 39. a. ALL
40. d. Turners syndrome 41. a. Downs syndrome 42. c. Chromosome 13
100 Jaypees Triple A
43. Deletion of chromosome 11 leads to: (DNB 01) 52. Chang staging is for:  (DNB 2009)
a. Wilms a. Retino blastoma
b. Neuroblastoma b. Rhabdo myosarcoma
c. Retinoblastoma c. Ewings sarcoma
d. Osteosarcoma d. Medulloblastoma
Ref: Ghai 8/e p617; Nelson 17/e p1683 table (484.2) Ref: Ghai 8/e p724; Nelson Pediatrics 18/e p2132-2134
44. All are associated with malignancy except: (MP 00) 53. Wilms tumour is associated with all except:  (DNB 2009)
a. Downs syndrome a. Aniridia
b. Fragile X syndrome b. Hemihypertrophy
c. Bloom syndrome c. Hypertension
d. Fanconis anaemia d. Bilateral polycystic Kidney
Ref: Ghai 8/e p608-620; Nelson 17/e p1683 table (484.2) Ref: Ghai 8/e p670; Nelson Pediatrics 18/e p2140-2143
45. Most common maligant tumour in childhood: (UP 07) 54. About Wilms tumor incorrect statement is:  (DNB 2011)
a. Rhabdomyosarcoma a. At age 10 years is the most common presentation
b. Leukemia b. Presents as a mass in abdomen
c. Lymphangioma c. Lungs are the commonest site for metastasis
d. Neuroblastoma Ref: OP Ghai 8/e p599 d. Haematuria is the presenting symptom
Ref: OP Ghai 8/e p638; 7/e p592
46. Most common benign tumours during infancy is:  (UP 07)
a. Lymphangioma 55. Second most common neoplasm seen in children is
b. Hemangioma a. Lymphoma (Karnatka PG 2011)
c. Cystic hygroma b. Brain tumour
d. Lipoma Ref: OP Ghai 8/e p623 c. Wilms tumour
d. Neuroblastoma
47. Poor prognostic factor for ALL is?
Ref: Op Ghai 8/e p571, 612; 7/e p545, 580
a. Hyperdiploidy
b. t(9;22) t(4;11) 56. Increased fetal Hb is seen in: (DP PGMEE 2010)
c. 2-8 yrs of age a. Juvenile CML
d. TLC < 50000 b. Congenital red cell aplasia
Ref: Ghai 8/e p599; Nelson 18/e p2123 c. Hereditary spherocytosis
d. AML
48. Most common inheritend childhood tumor is: (DNB 2008)
Ref: Ghai 8/e p608; Nelson 18/e p2122
a. Leukemia
b. Neuroblastoma 57. Which is not a tumour of the first decade of life?
c. Retinoblastoma a. Rhabdomyosarcoma (DP PGMEE 2010)
d. Wilms tumor b. Neuroblastoma
Ref: Ghai 8/e p638; Nelson Pediatrics 18/e p2151-2153 c. Medulloblastoma
d. Retinoblastoma
49. Chang staging is for:  (DNB 2008)
Ref: Ghai 8/e p612, 613
a. Retino blastoma
b. Rhabdo myosarcoma 58. Best prognostic cerebral tumour in children:  (Raj PG 2009)
c. Ewings sarcoma a. Medulloblastoma
d. Medulloblastoma b. Astrocytoma
Ref: Ghai 8/e p830; Nelson Pediatrics 18/e p2132-2134 c. Glioma
d. Meningioma Ref: Ghai 8/e p612
50. Wilms tumour is associated with all except:  (DNB 2008)
59. All of the following brain tumors spread via CSF except:
PEDIATRICS

a. Aniridia
b. Hemihypertrophy a. Craniopharyngoma (Kerala PG 09)
c. Hypertension b. CNS lymphoma
d. Bilateral polycystic Kidney c. Germ Cell tumours
Ref: Ghai 8/e p729; Nelson Pediatrics 18/e p2140-2143 d. Medulloblastoma Ref: Ghai 8/e p572
51. Most common inheritend childhood tumor is:  (DNB 2009) 60. Neuroblastoma arises form:  (Kerala PG 10)
a. Leukemia a. Adrenal
b. Neuroblastoma b. Chestwall
c. Retinoblastoma c. Rib
d. Wilms tumor d. Cerebrum
Ref: Ghai 8/e p719; Nelson Pediatrics 18/e p2151-2153 Ref: Ghai 8/e p616; Nelson 18/e p2137

Ans. 43. a. Wilms 44. b. Fragile X syndrome 45. b. Leukemia 46. b. Hemangioma
47. b. t(9;22) t(4;11) 48. c. Retinoblastoma 49. d. Medulloblastoma 50. d. Bilateral polycystic Kidney
51. c. Retinoblastoma 52. d. Medulloblastoma 53. d. Bilateral polycystic... 54. a. At age 10 years
55. b. Brain tumour 56. a. Juvenile CML 57. a. Rhabdomyosarcoma 58. b. Astrocytoma
59. a. Craniopharyngoma 60. a. Adrenal
16. MISCELLANEOUS
102 Jaypees Triple A

MISCELLANEOUS (QUESTIONS)

1. At the end of 1 year of age, the number of carpal bones seen 7. True about Langerhans histocytosis X: (PGI June 01)
in the skiagram ofthe hand is: (Corned 08) a. Can be associated with diabetes insipidus
a. Nil b. X-ray shows pathognomonic osteosclerotic lesions
b. 1 c. Birbecks granules in langhans cell
c. 2
d. Proliferation of antigen presenting cells
d. 3 Ref: Ghai 8/e p32; Nelson 17/e p. 37 table (10.7)
e. Associated with specific HLA DR
2. A girl on exposure to cold pallor, then cyanosis, predisposed Ref: Ghai 8/e p620; Nelson 19/e p1774
to:  (AIIMS Nov 08)
a. Scleroderma 8. Which of the following drugs is used for the treatment of
b. JRA refractory histiocytosis: (AIIMS Nov 08)
c. SLE a. High dose methotiocytosis
d. None b. High dose cytarabine
Ref: Ghai 8/e p630; Nelson 18/e p1024; Nelson 19/e p851 c. Cladribine
3. A child presents with seborrheic dermatitis, lytic skull d. Fludarabine
lesions, ear discharge and hepatosplenomegaly; likely Ref: Ghai 8/e p620; Nelson 18/e p2161; Nelson 19/e p1774
diagnosis:  (AI 01)
9. A 2-year-old girl child is brought to the out patient with
a. Leukemia
features of hand wringing stereotype movements, impaired
b. Lymphoma
c. Histiocytosis X language and communication development, breath-holding
d. Multiple myeloma spells, poor social skills and deceleration of head growth
Ref: Ghai 8/e p620; 7/e p595-596; Nelson 18/e p902; after 6 months of age. The most likely diagnosis is:
Nelson 19/e p1773-1775 a. Aspergers syndrome (AIIMS Nov 08)
4. A 2-year-old child comes with discharge, seborrheic b. Retts syndrome
dermatitis, polyuria and hepatosplenomegaly. Which of the c. Fragile X syndrome
following is the most likely diagnosis :  (AIIMS May 04) d. Colard syndrome
a. Leukemia Ref: Nelson 18/e p2504, 2505 and 17/e p94, 2034; Nelson 19/e p2075;
b. Lymphoma Internet
c. Langerhans cell histiocytosis
10. A mother is exposed to Diethyl stilbesterol during
d. Germ cell tumor
Ref: Ghai 8/e p620; 7/e p595; Nelson 18/e p902; Nelson 19/e p1775 pregnancy. All the following features may be seen in the
child after birth, except:  (AIIMS May 02)
5. A one-year-old boy presented with hepatosplenomegaly
a. Clear cell carcinoma
and delayed milestones. The liver biopsy and bone marrow
biopsy revealed presence of histocytes with PAS positive. b. Microglandular hyperplasia
Electron microscopic examination of these histiocytes is c. Malformation of the vagina and uterus
most likely to reveal the presence of:  (AIIMS 03) d. Vaginal adenosis
a. Birbeck granules in the cytoplasm Ref: Gynae shows 15/e p248; Nelson 17/e p1839; Robbins illustrated 7/e
b. Myelin figures in the cytoplasm p1071; Nelson 19/e p1873
c. Parallel rays of tubular structures in lysosomes
11. Kasabach-Merritt syndrome, true about A/E:
PEDIATRICS

d. Electron dense deposit in the mitochondria


a. Platelet sequestration (AIIMS Nov 09)
Ref: Ghai 8/e p620; Nelson 18/e p2159, 2160, Robbins 7/e p701; Nelson
19/e p1773, 1774 b. Infantile hemangioma
c. Consumption coagulopathy
6. A 2-year-old child presents with scattered lesions in the
d. Portwine hemangioma
skull. Biopsy revealed Langerhans giant cells. The most
commonly associated is marker with this condition will be: e. Thrombocytosis Ref: Medscape
a. CD Ia (AIIMS 05) 12. All of the followings are seen in Systemic Juvenile Arthritis,
b. CD 57 except:  (AI 00)
c. CD 3 a. Rheumatoid factor positive
d. CD 68 b. Hepatosplenomegaly
Ref: Ghai 8/e p620; Nelson 18/e p2159, 2160, Robbins 7/e p701; Nelson
c. High fever with rash
19/e p1773
d. Elevated SR Ref: Ghai 8/e p624; Nelson 19/e p829

Ans. 1. c. 2 2. a. Scleroderma 3. c. Histiocytosis X 4. c. Langerhans cell histiocytosis


5. a. Birbeck granules... 6. a. CD Ia 7. a, b, c and e 8. c. Cladribine
9. b. Retts syndrome 10. b. Microglandular hyperplasia 11. b, d and e 12. a. Rheumatoid factor...
Miscellaneous 103
13. What is the probable diagnosis for a cyst in a child which is with a variety of ecchymoses. What is the most likely cause
located at and associated with vertebral defects:  (AI 08) of the injuries: (Karnataka 03)
a. Myelocele a. Automobile hit - and - run accident
b. Bronchogenic cyst b. Fall from tree
c. Neuroenteric cyst c. Child abuse
d. Neuroblastoma d. Fall from a bicycle Ref: Ghai 8/e p708; Nelson 18/e p174
Ref: Ghai 8/e p619; Sabiston 18/e p1696, Internet 20. Cherry red spots and cloudy cornea are seen in: (AI 08)
14. A child is brought with drowsiness, decreased deep tendon a. Glycogen storage disease
reflexes and seizures. On examination the child has a line b. Lysosomal storage disease
on gums. There is history of constipation. Which will be c. Amino acidurias
most appropriate drug that should be used in this child: d. Fall from a bicycle Ref: Nelson 18/e p594, Ghai 8/e p652
a. EDTA (AI 07) 21. WHO 3 step ladder is for management of: (Orissa 04)
b. DMSA a. Pain
c. BAL b. Dysphagia
d. Penicillamine c. Dyspnea
Ref: Ghai 8/e p696; KDT 6/e p645, Nelson 18/e p2916; Nelson 19/e p2452 d. Dysuria Ref: Nelson 18/e p476
15. A two year old boy presents with fever for 3 days which 22. In infants, the cause of blindness arising out of oxygen
responded to administration of paracetamol. Three days toxicity is:  (AI 08)
later he developed acute renal failure, marked acidosis a. Degeneration of crystalline lens
and encephalopathy. His urine showed plenty of oxalate b. Growth of blood vessels into vitreous followed by fibrosis
crystals. The blood anion gap and osmolal gap were c. Damage to cornea
increased. Which of the following is the most likely d. Enzymic defect in lens Ref: OP Ghai 8/e p665
diagnosis:  (AIIMS Nov 05)
23. Ten percent dextrose is given with NS in neonates, the
a. Paracetamol poisoning
sister has to:
b. Diethyl glycol poisoning
a. 20%D + 80%NS
c. Severe malaria
b. 40% + 60%NS
d. Hanta virus infection
c. 60%D + 40%NS
Ref: Ghai 8/e p696; Nelson 17/e p2372
d. 80%D + 20%NS Ref: Ghai 8/e p320
16. Combinnation of retinitis pigmentosa and ichthyosis is
24. Which of the following is the least likely cause of neonatal
seen in:  (AI 08)
mortality in India?
a. Netherton syndrome
a. Severe infections
b. Refsums symdrome
b. Congenital malformation
c. Downs syndrome
c. Permaturity
d. Mobs syndrome
d. Birth asphyxia Ref: SPM Part 20/e p485
Ref: Ghai 8/e p666; CPDT 18/e p759
25. Regarding an imbecile, all are true except?
17. Sphenoid wing dysplasia is seen in:  (JIPMER 2K)
a. IQ is 50-60
a. Von Hippel Lindaus disease
b. Intellectual capacity equivalent to a child of 3-7 years of age
b. Neurofibromatosis
c. Not able to take care of themselves
c. Sturge Weber syndrome
d. Condition is congenital or acquired at an early age

PEDIATRICS
d. Bournvilles diisease
Ref: O P Ghai 7/e p562
Ref: Ghai 8/e p586; www.springerlink
26. A term male baby, with birth weight 3.5 kg, developed
18. A child presents with a history of scorpion sting. He is
respiratory distress at birth, not responded to administration
having increased sweating. What is the next best step:
of surfactant. Echocardiogram was normal. X-ray chest
a. Lytic coktail (JIPMER 2K)
shows ground glass appearance. There is history of death of
b. Atropine
a female sibling at 1 month of age. What is the diagnosis?
c. Antevenom
a. TAPVC
d. Local xylocaine infiltration
b. Meconium aspiration
Ref: Ghai 8/e p700; www.springerlink
c. Neonatal alveolar proteinosis
19. A small boy who is brought to the emergency department d. None of above
by his parents is found to have a spiral fracture of the femur, Ref: Nelson 18/e p1820-1821

Ans. 13. c. Neuroenteric cyst 14. a. EDTA 15. b. Diethyl glycol... 16. b. Refsums symdrome
17. b. Neurofibromatosis 18. d. Local xylocaine... 19. c. Child abuse 20. b. Lysosomal...
21. a. Pain 22. b. Growth of blood ... 23. d. 80%D + 20%NS 24. b. Congenital malformation
25. a. IQ is 50-60 26. c. Neonatal alveolar proteinosis
104 Jaypees Triple A
27. Which of the following is not true about JRA? b. Magnesium deficiency
a. Fever c. Infections
b. Uveitis d. Vitamin b 12 deficency
c. Rheumatoid nodules e. None  Ref: OP Ghai 8/e p639
d. Raynauds phenomenon 36. Pawn ball megakaryocytes are characteristic of: (DNB 2010)
Ref: Ghai 6/e p581 - 582; Nelson 18/e p1001-1010 and 17/e p799 801 a. Myelodysplastic syndrome
28. False about acrodermatitis enteropathica b. Idiopathic thrombocytopenic purpura
a. Triad of diarrrhea, dementia, and dermatitis c. Thrombotic thrombocytopenic purpura
b. Low zinc level d. Chloramphenicol toxicity
c. Symptoms improve with zinc supplement. Ref: Ghai 8/e p230; Campbell Text book of Peds, 10/e p868, 869
d. Autosomal recessive Ref: Internet 37. Early Neonatal sepsis in india is most commonly due to:
29. Histopathological featues of fat necrosis in a newborn a. Escherichia coli  (DNB 2010)
closely resembles? b Group-B Streptococci
a. Steroid induced lipid necorsis c. Staphylococci
b. Lupoid necrosis d. Pseudomonas
c. Erythema induratum Ref: Ghai 8/e p274; Nelson Essentials of Pediatrics 4/e p375, 376, 377
d. Lipodermatosclerosis Ref: Internet 38. Hypoglycemia in a new borne is glucose level of less than:
30. A 9-year-old girl has difficulty in combing hair and a. 50  (DNB 2011)
climbing upstairs. Gowers sign is positive. What is the next b. 55
investigation to be performed? c. 4045 mg/day
a. ESR d. 35
b. RA factor Ref: Ghai 8/e p218; Essentials of Peds 4/e p672, Nelson Paeds 17/e p816
c. Creatine kinase 39. Not a response seen in new borne as a measure of
d. EMG Thermogenesis is:  (DNB 2011)
Ref: Nelson 18/e p2095 and 17/e p2053-2054; Harrison 16/e p1450 A. Flexion
31. Failure to pass meconium within 48 hrs of birth in a B. Vasoconstriction
newborn with no obvious external abnormality should lead C. Brown fat breakdown
to the suspicion of:  (DNB 2005) D. Shivering
a. Anal atresia Ref: Ghai 8/e p99; Op Ghai 6/e p150, 151, 152, Campbell Peds
b. Congenital pouch colon 40. Neonatal hypoglycaemia is defined as blood glucose of less
c. Congenital aganglionosis than:  (MP PG 2010)
d. Meconium ileus a. 30 mg/dl
Ref: Ghai 8/e p512; Nelson 16/e p1888 15/e p1784) b. 40 mg/dl
32. Best method of diagnosis of childhood HIV:  (DNB 2007) c. 50 mg/dl
a. CD4 cell counts d. 60 mg/dl
b. P24 antigen Ref: Ghai 8/e p179; Nelsons 18/e p702; NNF Manual of neonatal care 92
c. ELISA 41. The most common mode of transmission of HIV from the
d. Anti HIV antibody mother to the baby is:  (MP PG 2009)
Ref: OP Ghai peds 8/e p510, Nelson Pediatrics 18/e p1427-1435 a. Transplacental
33. Most common sequelae to periventricular leukomalacia is: b. During delivery
a. Spastic diplegia (DNB 2008) c. Through breast milk
b. Spastic quadriplegia d. Kiss and skin contract
PEDIATRICS

c. Hypotonia Ref: Ghai 8/e p183; Nelsons 18/e p1429


d. Mental retardation 42. Which channel is defective in Malignant Hyperpyrexia? 
Ref: Ghai 8/e p633; Nelsons text book of paediatrics, Campbell Text book of a. Sodium (MP PG 2008)
Peds, 10/e p1259 b. Potassium
34. In Hutchinsons triad not seen is:  (DNB 2009) c. Calcium
a. Keratitis d. Magnessium Ref: Ghai 8/e p521; Harrisons 17/e p118
b. Notched incisors 43. Major criteria for jones classification is:  (MP PG 2008)
c. Deafness a. Fever
d. Cataracts Ref: 8/e p710 b. Arthralgia
35. Not a cause of infantile tremor syndrome is:  (DNB 2009) c. Chorea
a. Malnutrition d. PR Interval Ref: Ghai 8/e p433; 7/e p38

Ans. 27. d. Raynauds... 28. a. Triad of diarrrhea... 29. d. Lipodermatosclerosis 30. c. Creatine kinase
31. c. Congenital... 32. b. P24 antigen 33. a. Spastic diplegia 34. d. cataracts
35. e. None 36. a. Myelodysplastic 37. b. Group-B Streptococci 38. c. 4045 mg/day
39. d. Shivering 40. b. 40 mg/dl 41. b. During delivery 42. c. Calcium
43. c. Chorea
Miscellaneous 105
44. Antidote of paracetamol poisoning: (MP PG 2008) a. All symptomatic HIV infected children>5 year of age
a. Flumazenil irrespective of CD4
b. N-acetyle cysteine b. All HIV exposed infants till HIV infection can be ruled
c. Nalexone out
d. Sodium bicarbonate Ref: Ghai 8/e p700; 7/e p680 c. All HIV infected infants less than 1 year age irrespective
45. Hypoglycemia in neonates occurs in blood glucose less than of symptoms or CD4 counts
a. 20 mg%  (UP PG 09) d. As secondary prophylaxis after initial treatment for
b. 40 mg% pneumocystis carini pneumonial
c. 60 mg% Ref: Ghai 8/e p183; 7/e p205-207
d. 10 mg% Ref: Ghai 8/e p179; 7/e p155 54. Indications to start ART in children under NACO all except:
46. Increased ammonia level, normal pH, acidosis (high anion  (DP PGMEE 2009)
gap) is seen in  (UP PG 09) a. <11 months infant-if CD4 Count <1500 cells/mm3
a. Urea cycle b. 12-35 months-if CD4 count <750 cells/mm3
b. Galectosemia c. 36-59 months-if CD4 Cont <350 cells/mm3
c. Organic-acidemia d. >5 year-if CD4 Cout <200 cells/mm3
d. Alkaptonuria Ref: Ghai 8/e p655 Ref: Ghai 8/e p183; 7/e p204, Nelson 18/e p1430-1432
47. What should be the internal diameter of the endotracheal 55. Factor associated with increased incidence for sudden
tube in a child weighing 1500g ? (MHPGM-CET 2007, 2010) infant death syndrome is: (DP PGMEE 2009)
a. 2mm a. High socioeconomic status
b. 2.5mm b. Female gender
c. 3mm c. Supine sleep positioning
d. 3.5mm Ref: OP Ghai 8/e p137 d. Maternal smoking
Ref: Ghai 8/e p708; Reddy 22/e p374
48. A 10-year-old gets drowned and stays under water for 20
minutes, what should you do?  (AP 2010) 56. An infant is crying excessively even after being given
a. Barbiturate Reflux good feed. He is passing a large quantity of urine and
b. Steroid administration repeatedly getting dehydrated. Urine examination shows
c. Cardiac massage no proteinuria and a specific gravity of 1004. The most
d. Wait & watch  Ref: Ghai 8/e p705, 706 likely diagnosis is: (DP PGMEE 2010)
a. Diabetes mellitus
49. All of the following drugs are used in the remission
b. Diabetes insipidus
induction in a case of all except:  (AP 2010)
c. Congenital nephritic syndrome
a. Vincristine
d. Protein losing enteropathy Ref: Ghai 8/e p519
b. Prednisone
c. L-asparaginase 57. A small boy who is brought to the emergency department
d. Cytosine arabinoside Ref: Ghai 8/e p599 by his parents is found to have a spiral fracture of the femur,
with a variety of ecchymoses. What is the most likely cause
50. Which is not an induction for bone marrow transplantation of the injures? (DP PGMEE 2010)
a. Severe combined immunodeficiency disease  (AP 2010) a. Automobile hit-and-run accident
b. Wiskott-Aldrichs syndrome b. Fall from a tree
c. Ataxia telangiectasia c. Child abuse
d. Chronic granulomatous disease Ref: Ghai 8/e p187 d. Fall from a bicycle Ref: Ghai 8/e p770; Nelson 18/e p174
51. A positive Apt test would mean that the blood is due to 58. Which of the following is the principal mode of heat
a. Urogenital bleeding from the neonate exchange in an infant incubator? (DP PGMEE 2010)

PEDIATRICS
b. Gastrointestinal bleeding from the neonate a. Radiation
c. Neonate swallowed maternal blood b. Evaporation
d. Neonate aspirated maternal blood c. Convection
52. Sudden ophthalmoplegia in Thai child eating raw fish and d. Conduction Ref: Ghai 8/e p145
cooked rice  (AP 2011) 59. Enzyme replacement therapy is available for which of the
a. Thaimine following disorders? (Feb DP PGMEE 2009)
b. Lead Poisoning a. Gauchers disease
c. Botulism b. Niemann pick disease
d. Influenza  Ref: Ghai 8/e p696 c. Mucolipidosis
53. Prophylaxis with cotrimoxazole is recommentded in the d. Metachromtic leukodystrophy
following situation except:  (DP PGMEE 2009) Ref: Ghai 8/e p661

Ans. 44. b. N-acetyle cysteine 45. b. 40 mg% 46. c. Organic-acidemia 47. c. 3mm
48. c. Cardiac massage 49. d. Cytosine arabinoside 50. c. Ataxia telangiectasia 51. c. Neonate swallowed
52. a. Thaimine 53. a. All symptomatic... 54. b. 12-35 months-if CD4... 55. d. Maternal smoking
56. b. Diabetes insipidus 57. c. Child abuse 58. c. Convection 59. a. Gauchers disease
106 Jaypees Triple A
60. Costochondral junction swelling are seen in:  (Raj PG 2009) 66. Which among the following is not true regarding artificial
a. Scurvy feeding in mothers with HIV infection:  (Kerala PG 10)
b. Rickets a. Acceptable if safe alternative
c. Chondrodystrophy b. Continue if affordable
d. All of the above Ref: Ghai 8/e p113 c. Rapid weaning from initial 6 months of breast feeding
61. Percentage of burn in an infant if head and neck are d. Not at all recommended
involved:  (Kerala PG 08) Ref: Ghai 8/e p183; Nelson 18/e p215, 1429
a. 9 % 67. In Ehlers Danlos Syndrome there is defect In:  (WB PG 08)
b. 18 % a. Glycoprotein
c. 27 % b. Collagen
d. 36 % c. Cartilage
Ref: Ghai 8/e p705; Nelson, 17/e p333 Fig. 62-1, 18/e p453, Fig. 74-3 d. Mucopolysaccharide
62. Contraindication for bag and mask ventilation among the Ref: Ghai 8/e p624, 630; Harrison 6/e p2328
following is:  (Kerala PG 08) 68. I systemic form of JRA, all are true except:  (WB PG 08)
a. Meconium aspiration a. RA factor positive
b. Tracheo esophageal fistula b. High Fever with rash
c. Congenital lung cyst c. Hepato splenomegaly
d. Birth asphyxia Ref: OP Ghai 8/e p125 d. Elevated ESR Ref: Ghai 8/e p624, 630
63. Calcification of intervertebral disc occurs in: (Kerala PG 08) 69. All of the following are indirect markers of Neonatal Sepsis
a. Alkaptonuria except:  (J & K PG 2011)
b. Phenylketouria a. Blood Culture
c. Gout b. Total Leucocyte count
d. Rickets Ref: Ghai 8/e p653; Nelson, 17/e p403; 18/e p534 c. I/T ratio
64. All are true about cephalhematoma except:  (Kerala PG 08) d. C Reactive Protein  Ref: OP Ghai 8/e p163
a. Subperiosteal location 70. In which of the following poisonings is gastriclavage
b. Donot cross suture line contraindicated:  (J & K PG 2010)
c. Subsides within 24 hours a. Organophosporous compound
d. Increase in jaundice Ref: Ghai 8/e p141; Dutta 6/e p483 b. Kerosene
65. Lithium administration in pregnancy will cause which of c. Opium
the following abnormality in fetus:  (Kerala PG 08) d. INH Ref: Ghai 8/e p696
a. Ebstein anomaly 71. The bones of which joint are e valuated for estimation of
b. Transposition of great vessels skeletal age in children between 3 and 9 months:
c. Truncus arteriosus a. Wrist (J & K PG 2010)
d. Tetralogy of Fallot b. Shoulder
Ref: Ahuja textbook of psychiatry, 5/e p196; Ghai 8/e p423 c. Elbow
d Hip Ref: OP Ghai 8/e p11
PEDIATRICS

Ans. 60. d. All of the above 61. b. 18% 62. a. Meconium aspiration 63. a. Alkaptonuria
64. c. Subsides within... 65. a. Ebstein anomaly 66. d. Not at all recommended 67. b. Collagen...
68. a. RA factor positive... 69. a. Blood Culture 70. b. Kerosene 71. b. Shoulder
Section B

PRACTICE QUESTIONS
(Comprising of Questions from Recent Exams and
NEET Pattern Questions)
PEDIATRICS
Practice Questions

1. Indication of kidney biopsy in Post-streptococcal 8. Constipation in child is/are caused by: (PGI May 2013)
Glomerulonephritis in a child is/are: (PGI May 2013) a. Stool consistency matters, not frequency
a. Presence of nephrotic syndrome b. Milk of magnesia
b. Presence of microscopic hematuria c. Defect in auerbach plexus as occur in Hirschsprung
c. Low complement level in first week disease
d. Acute renal failure d. In majority of infant organic cause is present
e. Absence of evidence of streptococcal infection e. Hypothyoidism
Ref: Ghai 8/e p474 Ref: Ghai 8/e p283, 284
2. Feature of scurvy in children include(s): (PGI May 2013) 9. True regarding classical Tetralogy of Fallot:
a. Bleeding from gums a. Right to left shunt (PGI May 2013)
b. Tibial edema b. Left to right shunt
c. Costochondral junction become angular
d. Windswept hip deformity in children with cerebral palsy c. Severe pulmonary valvular stenosis
e. Splaying and cupping of metaphysis d. Subvalvular pulmonary stenosis may be severe
Ref: Ghai 8/e p120 e. VSD is small Ref: Ghai 8/e p420
3. Use of IV magnesium in children includes: (PGI May 2013) 10. Most common cause of acute tracheitis in children is:
a. Carpopedal spasm in calcium deficiency a. Staphylococcus aureus (PGI May 2013)
b. Vit. D resistant rickets b. H.influenzae
c. Torsades de pointes c. Streptocccus pyogenes
d. 2 AV heart block d. Moraxella catarrhails
e. Severe hypomagnesimea e. Mycoplasma Ref: Ghai 8/e p368
Ref: Internet 11. All of the following are the causes of neonatal cholestasis
4. True statement regarding children: (PGI May 2013) except: (PGI May 2013)
a. BMI more than 95 percentile for age as obesity a. Extrahepatic Biliary atresia
b. BMI more than 85 percentile for age as overweight b. Alagille syndrome
c. Weight more than 95 percentile for age as obesity c. Gilbert syndrome
d. Weight more than 85 percentile for age as overweight d. Rotor syndrome
e. BMI is calculated by dividing the weight in kilograms by e. Gauchers disease Ref: Ghai 8/e p327
the height in meters square (kg/m2). Ref: Internet 12. Cause of unconjugated bilirubinemia: (PGI May 2013)
5. True about autism: (PGI May 2013) a. Dubin Johnson syndrome
a. More common is girls b. Gilbert syndrome
b. Concordance in monozygotic twins is 60-90% c. Neonatal hepatitis
c. MMR vaccine increases the risk d. Rotor syndrome
d. Child may have mental retardation Ref: Ghai 8/e p61 e. Crigler-Najjars syndrome
6. Poor prognostic factor in bacterial meningitis in children Ref: Ghai 8/e p324
includes(s): (PGI May 2013) 13. Which of the following match is correct regarding
a. Age less than 6 months milestones (child can do): (PGI May 2013)
b. Seizures occurring more than 4 days into therapy a. 4 month: recognize mother
c. Neck rigidity b. 2 yr; Can make tower of blocker 15-20 cubes
d. Focal neurological deficits c. 6 Month: Coos
e. High concentrations of bacteria in CSF d. 1yr: Standing without support
Ref: Ghai 8/e p563 e. 2yr-sentence of 2-3 words Ref: Ghai 8/e p46-53
7. Hypercalcemia in children is/are seen in all of the following 14. A baby can breathe and suck at the same time. This is due
combinations except: 137/9, 134: (PGI May 2013) to: (AIIMS May 2013)
a. Wolfram syndrome and MEN II syndrome a. Highly placed larynx
b. William syndrome and thiazide diuretics b. Wide short tongue
c. MEN I and hypervitaminosis D c. Short soft palate
d. Thyrotoicosis and Hypervitaminosis A d. Short pharynx
e. Hypervitaminosis C Ref: Ghai 8/e p81, 523 Ref: b.d. Chaurasia 6/e p252

Ans. 1. a, d and e 2. a, b and c 3. a, c, d and e 4. a, b and e


5. b and d 6. a, b, d and e 7. a and e 8. c and e
9. a, c and d 10. a. Staphylococcus 11. c. Gilbert syndrome 12. b and e
13. a, c, d and e 14. a. Highly placed
Practice Questions 109
15. An 8-year-old body complains of increasing muscle 22. All of the following are considered development delay
weakness. On examination, his valves are bulky and show except: (AIIMS May 2013)
muscle tightening. His serum creatine kinase levels are a. Pincer grasp not at 9 months
increasing with age. Which of the following is the most b. Not able to sit at 9 months
likely diagnosis? (AIIMS May 2013) c. Not able to go up and down stairs at 2.5 years of age
a. Hereditary sensorimotor neuropathy d. 2 words phrase at 15 months of age
b. Myelin deficiency Ref: Ghai 8/e p419
c. Dystrophin deficiency 23. Which of the following is least likely is PDA?

PRACTICE Qs Pediatrics
d. Congenital myopathy Ref: Internet a. CO2 wash out (AIIMS May 2013)
16. A 12-year-old boy develops sore throat of 4 days duration. b. Necrotizing enterocolitis
On examination, yellow grayish patch seen over both the c. Bounding pulse
tonsils and foul smell coming from his mouth. Which of the d. Pulmonary haemorrhage Ref: Ghai 8/e p419
following non-suppurative complication is of concern? 24. In a patient of nephrotic syndrome with steroid toxicity,
a. Acute rheumatic fever (AIIMS May 2013) alternative treatment: (AIIMS May 2013)
b. Acute glomerulonephritis a. Levamisole
c. Both acute rheumatic fever and acute glomerulonephritis b. Cyclophosphamide
d. Scarlet fever Ref: Ghai 8/e p367 c. Mycophenolate
17. A 4-years-old child presented in emergency with respiratory d. Cyclosporine Ref: Internet
difficulty and noisy breathing with drooling of saliva. 25. Drug of choice for juvenile myoclonic epilepsy is?
Emergency X-ray showed thumb sing. Diagnosis is: a. Phenytoin (DNB 2013)
a. Croup (AIIMS May 2013) b. Lamotrigine
b. Epiglottitis c. Valproate
c. Foreign body aspiration d. Zonisamide
d. Retropharyngeal abscess Ref: Ghai 8/e p368 Ref: KD Tripathi 7/e p212, 421
18. A 4-year-child with barky cough and stridor only while 26. First sign of puberty in females is? (DNB 2013)
crying, RR-36/min, temperature 39.6 C was brought to a. Pubarche
emergency at 3.00 am. The child is well hydrated, feeding b. Thelarche
well and consolable. Next line of management in this c. Menarche
patient: (AIIMS May 2013) d. Increase in height Ref: Ghai 8/e p63
a. Racemic epinephrine nebulization
27. Adolescent starts at the age of? (DNB 2013)
b. High dose of dexamethasone injection
a. 7 years
c. Antibiotics and send blood for culture
b. 10 years
d. Nasal washings for RSV or influenza
c. 14 years
Ref: Ghai 8/e p368
d. 17 years
19. A child of 2 years of age present with an abdominal Ref: Ghai 8/e p63
mass. Abdominal mass appears solid in consistency. On
28. Ballard score is useful for assessment of?
examination, his right upper and lower extremities appear
a. Bone age (NEET Pattern Question)
slightly longer. Diagnosis is: (AIIMS May 2013)
b. Mental age
a. Wilms tumor
c. Gestational age
b. Neuroblastoma
d. Lung maturity Ref: Ghai 8/e p138
c. Nephroblastoma
d. Angiomyolipoma Ref: Ghai 8/e p617
29. Number of pustules of diagnose a possible severe bacterial
20. A 2-days-old premature neonate develops GTCS. What is
infection in children is? (DNB 2013)
the investigation done to diagnose the pathology?
a. > 5
a. Transcranial ultrasound (AIIMS May 2013)
b. > 10
b. CT Head
c. > 15
c. MRI brain
d. > 20
d. X-ray Ref: Internet
30. Child draws triangle at what age? (DNB 2013)
21. In India, neonatal sepsis not caused by:
a. 3 years
a. Staphylococcus aureus (AIIMS May 2013)
b. 5 years
b. E. coli
c. 6 years
c. Group b streptococci
d. 7 years
d. Klebsiella Ref: Internet
Ref: Ghai 8/e p51

Ans. 15. c. Dystrophin... 16. c. Both acute... 17. b. Epiglottitis 18. a. Racemic...
19. a and c 20. a. Transranial... 21. c. Group b streptococci 22. d. 2 words phrase at 15...
23. a. CO2 wash out 24. b. Cyclophosphamide 25. c. Valproate 26. b. Thelarche
27. b. 10 years 28. c. Gestational age 29. a. > 5 30. b. 5 years
110 Jaypees Triple A
31. Which of the following has no role in diagnosis of childhood 40. Preterm neonate target oxygen saturation is?
TB? (DNB 2013) a. 85-95% (DNB 2013)
a. ELISA b. > 95%
b. Mantoux c. 70-95%
c. CXR d. < 80%
d. FNAC Ref: Ghai 8/e p253 Ref: Internet
32. Lactose intolerance in a suspected patient is diagnosed by 41. Most common cause of fetal ventriculomegaly is?
giving an oral and measuring concentrations of? a. Arnold Chiari Malformation-I
PRACTICE Qs Pediatrics

a. CO2 (DNB 2013) b. Arnold Chiari malformation-II (DNB 2013)


b. H2 c. Aqueductal stenosis
c. One/two carbon compounds d. Dandy Walker Malformation
d. N2 Ref: Internet Ref: Ghai 8/e p574
33. A child 4 month old has 10 episodes of vomiting and 2-3 42. A 6-year-old child with acute onset of fever of 104 degree
episodes of loose stools and crying since the last 24 hours, fever developed febrile seizure was treated. To avoid
best line of management will be? (DNB 2013) recurrence of seizure attack what should be given?
a. Intravenous fluids a. Paracetamol 400 mg and phenobarbitone (DNB 2013)
b. ORS b. Oral diazepam 6 hourly
c. Intravenous fluids then ORS c. Paracetamol 400 mg 6 hourly
d. Hospitalise and treat Ref: Ghai 8/e p293 d. IV Diazepam infusion over 12 hourly
34. Most common cause of Bronchiolitis is? Ref: Ghai 8/e p554
a. RSV (DNB 2013) 43. A child climb stairs at: (FMGE March 2013)
b. Adenovirus
a. 6 month
c. Parainfluenza
b. 12 month
d. Mycoplasma Ref: Ghai 8/e p381
c. 24 month
35. True about Wilsons disease is? (DNB 2013) d. 36 month
a. Increased serum ceruloplasmin Ref: Ghai 8/e p47
b. Decreased liver copper
44. Ponderals index for small for gestational age child:
c. Increased urinary copper excretion
a. Less than 1 (FMGE March 2013)
d. Decreased urine copper excretion
b. Less than 2
Ref: Ghai 8/e p321
c. Less than 3
36. Best prognosis of neonatal seizures is in? d. Less than 4
a. Myoclonic
Ref: Ghai 6/e p14; Internet
b. Tonic clonic (DNB 2013)
c. Focal 45. A woman gives birth to a child who has rashes on the
d. Opsoclonus Ref: Internet palmar plantar surface & this woman has a h/o previous
still birth. The most likely condition this newborn having
37. Diagnosis of toxoplasmosis in newborn is done by?
is? (FMGE March 2013)
a. IgG antibody against toxoplasma (DNB 2013)
a. Pemphigus
b. IgM antibodies to toxoplasma
b. Congenital syphilis
c. IgA antibodies to toxoplasma
c. HIV
d. IgE antibodies to toxoplasma Ref: Ghai 8/e p272
d. Chancroid
38. Most common enzyme deficiency leading to childhood Ref: Davidson 21/e p418, 419; Harrison 17/e, p1038-1045;
hypertension: (DNB 2013) Fitzpatricks Dermatology in General Medicine 6/e, p 2164-2212
a. 17-Alpha hydroxylase
b. 21-Beta Hydroxylase 46. A 15-year-old female child following URTI developed
c. 11-Beta Hydroxylase maculopapular Rash on the face spreading onto the trunk
d. 3-Beta Hydroxy steroid dehydrogenase which cleared on the 3rd day without desquamation and
Ref: Ghai 8/e p526) tender post auricularand suboccipital lymphadenopathy.
The diagnosis is: (FMGE March 2013)
39. Blood volume is preterm neonate is? (DNB 2013)
a. Kawasaki disease
a. 90 ml/kg
b. Erythema infectiosum
b. 80 ml/kg
c. Rubella
c. 70 ml/kg
d. Measles
d. 60 ml/kg
Ref: Davidson 21/e p321; Ghai 8/e p272;
Ref: Internet
Nelson 18/e p1339

Ans. 31. a. ELISA 32. b. H2 33. b. ORS 34. a. RSV


35. c. Increased urinary... 36. c. Focal 37. b. IgM antibodies... 38. c. 11-Beta Hydroxylase
39. a. 90 ml/kg 40. a. 85-95% 41. c. Aqueductal... 42. b. Oral diazepam...
43. c. 24 month 44. b. Less than 2 45. b. Congenital syphilis 46. c. Rubella
Practice Questions 111
47. Anterior fontanelle ossifies by: (FMGE March 2013) c. Leftventricular hypertrophy
a. 6 month d. ASD Ref: Ghai 8/e p420; Nelsons 17/e p1527-1529
b. 12 month 57. Commonest type of seizure in newborn:
c. 18 month a. Clonic (FMGE March 2013)
d. 24 month Ref: Ghai 6/e p48; Internet b. Tonic
48. A 10-year-old child is having a mental age of 4. He is: c. Subtle
a. Idiot (FMGE March 2013) d. Myoclonic Ref: Nelson 17/e p675; Internet
b. Normal 58. A child with alopecia, hyperpigmentation psoriatic

PRACTICE Qs Pediatrics
c. Imbecile dermatitis in genitals & mouth and hypogonadism is likely
d. Moron Ref: Park 21/e, p629 to be suffering from: (FMGE March 2013)
49. Which of the following does not indicate an infant good a. Cu deficiency
attachment to mothers breast during breast feeding: b. Iron deficiency
a. Only upper areola visible (FMGE March 2013) c. Mg deficiency
b. Babys chin touches mothers chest d. Zn deficiency Ref: Nelson 16/e p145; Ghai 8/e p121
c. Lower lip inverted 59. Calculate the ponderal Index of a baby of weight 2000 gm
d. Babys mouth is wide open Ref: Ghai 8/e p154 and length 50 cm at birth?
50. Most common deficient enzyme in congenital adrenal a. 1.6 (FMGE March 2013)
hyperplasia: (FMGE March 2013) b. 2.0
a. 21-hydroxylase deficiency c. 2.2
b. 18-hydroxylase deficiency d. 2.6 Ref: Ghai 7/e p107
c. 3-beta-hydroxysteroid dehydrogenase deficiency 60. A neonate with PDA is NOT likely to have?
d. 17-alpha hydroxylase deficiency a. CO2 washout (FMGE March 2013)
Ref: Ghai 8/e p526, 527 b. Bounding pulse
51. RSV is treated by: (FMGE March 2013) c. Nectrotizing enterocolitis
a. Acyclovir d. Pulmonary hemorrhage
b. Ribavirin Ref: Nelson 17/e p1215; Ghai 8/e p417, 418
c. Interferon 61. The height of a child is double the birth height at the age
d. Abacavir Ref: Ghai 8/e p382 of: (NEET/DNB Pattern Question)
52. Cause of reyes syndrome in children: (FMGE March 2013) a. 1 years
a. Polio virus b. 2 years
b. Measles virus c. 4 years
c. Influenza virus d. 6 years
d. HIV Ref: Ghai 8/e p570 e. 8 years Ref: Ghai 8/e p13; 7/e p6
53. A full term 80 hours old new born baby develops jaundice. 62. Pincer grasp is attained months:
What should be the minimum level of serum bilirubin to a. 4 (NEET/DNB Pattern Question)
start phototherapy: (FMGE March 2013) b. 10
a. 20 mg% c. 12
b. 12.5 mg% d. 18 Ref: Ghai 8/e p49, 50; 7/e p28
c. 18 mg% 63. Which of the following cannot be done by 3 years old
d. 15 mg% Ref: Ghai 8/e p172 child: (NEET/DNB Pattern Question)
54. Very severe low birth weight is less than: a. Draw a triangle
a. 1000 gm (FMGE March 2013) b. Draw a circle
b. 1250 gm c. Can arrange 9 cubes
c. 200 gm d. Can go up and down
d. 2500 gm Ref: Nelson 17/e p115 e. Stands on one foot for 5 second
55. Plantar Grashp Reflex disappear by: Ref: Ghai 8/e p51; 7/e p28; Nelson 18/e p49
a. 2 months (FMGE March 2013) 64. A child is able to build blocks of 5 cubes developmental age
b. 3 months is: (NEET/DNB Pattern Question)
c. 4 months a. 12 months
d. 5 months Ref: Nelsons 17/e p1979 b. 15 months
56. Which of the following is component of tetralogy of fallot: c. 18 months
a. PDA (FMGE March 2013) d. 24 months
b. Coarctation of aorta Ref: Internet

Ans. 47. c. 18 months 48. c. Imbecile 49. c. Lower lip... 50. a. 21-hydroxylase...
51. b. Ribavirin 52. c. Influenza virus 53. a. 20 mg% 54. a. 1000 gm
55. c. 4 months 56. None 57. c. Subtle 58. d. Zn deficiency
59. a. 1.6 60. a. CO2 washout 61. c. 4 years 62. b. 10
63. a. Draw a triangle 64. d. 24 months
112 Jaypees Triple A
65. A child is able to say short sentences of 6 words: c. Foreign object being put in the mouth
a. 2 years (NEET/DNB Pattern Question) d. None of above Ref: Nelson 17/e Chap 22.2; Ghai 8/e p58
b. 3 years 75. Maximum growth spurt is seen in girls at time of:
c. 4 years a. Pubarche (NEET/DNB Pattern Question)
d. 5 years Ref: Ghai 8/e p53; Nelson 18/e chap8-9 b. Thelarche
66. Ist permanent teeth of appear: c. Menarche
a. Molor (NEET/DNB Pattern Question) d. Adernarche Ref: Nelson 18/e p60-62; Ghai 8/e p63
b. Premolor 76. What is thelarche: (NEET/DNB Pattern Question)
PRACTICE Qs Pediatrics

c. Incisor a. Pubertal breast enlargement in boys


d. Cannine Ref: Ghai 8/e p11 b. Breast enlargement in pregnancy
67. Milk teeth-Total no. in human being: c. Hormone related breast enlargement in girls
a. 20 (NEET/DNB Pattern Question) d. Post hormonal therapy breast enlargement in postmeno-
b. 28 pausal females Ref: Ghai 8/e p63
c. 32 77. Kwashorkar is diagnosed in growth retarded children along
d. 24 Ref: Nelson 17/e chap. 304; Internet with: (NEET/DNB Pattern Question)
68. Delayed dentition is seen in all/except: a. Edema and mental changes
a. Down syndrome (NEET/DNB Pattern Question) b. Hypopigmentation and anemia
b. Congenital hypothyoridism c. Edema and hypopigmentation
c. Rickets d. Hepatomegaly and anemia
d. None of above Ref: CPDT 18/e p452; Nelson 18/e p47, 73 Ref: Ghai 8/e p99, 100; 7/e p67
69. Nails is formed in which week of IUL: 78. All are seen in Marasmus except:
a. 0 6 week (NEET/DNB Pattern Question) a. Hepatomegaly (NEET/DNB Pattern Question)
b. 10 12 week b. Muscle wasting
c. 14 18 week c. Voracious appetite
d. 20 24 week Ref: Nelson 18/e Chapter 643; Internet d. Weight loss Ref: Ghai 8/e p99; 7/e p67
70. When ICF and ECF of child becomes equal to adult person: 79. Flaky paint appearance of skin is seen in:
a. 1 year a. Dermatitis (NEET/DNB Pattern Question)
b. 2 year (NEET/DNB Pattern Question) b. Pellagra
c. 3 year c. Marasmus
d. 4 year d. Kwashiorkor Ref: Ghai 8/e p99, 100; 7/e p67
e. C Ref: Nelson Chapter 52 80. Exclusive breast feeding is at least till:
71. Upper segment of lover segment ratio in 3 yr age child is: a. 4 month (NEET/DNB Pattern Question)
a. 1.2 (NEET/DNB Pattern Question) b. 6 month
b. 1.3 c. 8 month
c. 1.4 d. 10 month Ref: Ghai 8/e p90; 7/e p60
d. 1.6 Ref: Chhedda 3/e p22; Internet Ans. b. 6 month
72. Short stature, secondary to growth hormone deficiency is 81. Breast milk at room temperature stored for:
associated with: (NEET/DNB Pattern Question) a. 4 hours (NEET/DNB Pattern Question)
a. Normal body proportion b. 8 hours
b. Low birth weight c. 12 hours
c. Normal epiphyseal development d. 24 hours
d. Height age equal to skeletal age Ref: Ghai 8/e p155; 7/e p127
Ref: Ghai 8/e p511; 7/e p474-475 82. A 2-year-old boy has vitamin D resistant rickets.
73. Fragile X-syndrome is characterised by all of the following His investigations revealed serum Calcium-9 mg/dl,
features except: (NEET/DNB Pattern Question) Phosphate-2.4 mg/dl, alkaline phosphatase-1041 IU, normal
a. Long face intact parathyroid hormone and bicarbonate 22 mEq/L.
b. Large ear Which of the following is the most probable diagnosis:
c. Large-nose (AIIMS May 04, 02, Neet/DNB Pattern Question)
d. Large-testis a. Distal renal tubular acidosis
Ref: CPDT 18/e p99; Internet b. Hypophoshatemic rickets
74. Pica refers to: (NEET/DNB Pattern Question) c. Vitamin D dependent rickets
a. IU sucking d. Hypoparathyroidism
b. Thumb sucking Ref: Ghai 8/e p114; 7/e p83; Nelson 17/e p2342

Ans. 65. c. 4 years 66. a. Molor 67. a. 20 68. d. None of above


69. b. 10 12 week 70. a. 1 year 71. b. 1.3 72. a. Normal body proportion
73. c. Large-nose 74. c. Foreign object 75. c. Menarche 76. c. Hormone related...
77. a. Edema and mental... 78. a. Hepatomegaly 79. d. Kwashiorkor 80. b. 6 month
81. b. 8 hours 82. b. Hypoparathyroidism...
Practice Questions 113
83. Wind swept deformity is seen in: 92. Meconium passage in utero leads to which of the
a. Scurvy (NEET/DNB Pattern Question) following: (NEET/DNB Pattern Question)
b. Rickets a. Listeriosis
c. Achondroplasia b. Obstructive Emphysema
d. Osteoporosis Ref: Internet c. Pathological jaundice
84. In 30 years female, with high alkaline phosphatase, low d. Meconium ileus
serum calcium, low serum phosphatase diagnosis is?: Ref: Dutta 4/e p511; Ghai 8/e p170; 7/e p144-145; CPDT 18/e p20, 21
a. Rickets (NEET/DNB Pattern Question) 93. Hyaline membrane seen in the lung is composed of:

PRACTICE Qs Pediatrics
b. Osteomalacia a. Globulin (NEET/DNB Pattern Question)
c. Hypervitaminosis D b. Fibrin
d. Hyperparathyroidism Ref: Davidson 21/e p1121-1122 c. Mucoprotein
85. Selenium deficiency is seen in: d. Polysaccharide Ref: Ghai 7/e p143; Robbins 7/e p715
a. Keshan disease (NEET/DNB Pattern Question) 94. Foetal lung maturity is assessed by:
b. Wilson disease a. L/S ratio (NEET/DNB Pattern Question)
c. Acrodermatitis enteropathica b. Billirubin content of amniotic fluid
d. None of the above c. Ultra sound
Ref: Nelson 17/e Chap 51; Davidson 21/e p129 d. Amniocentesis Ref: Ghai 6/e p167 and 7/e p144; Internet
86. Diarrhoea in a child of 12 month, dose of Zinc is: 95. The dose of betamethasone in prenatal to prevent
a. 1 mg/10 14 day (NEET/DNB Pattern Question) respiratory distress syndrome is:
b. 10 mg/10 14 day a. 6 mg (NEET/DNB Pattern Question)
c. 15 mg/10 14 day b. 12 mg every 24 hours
d. 20 mg/10 14 day Ref: Ghai 8/e p295 c. 6 mg every 12 hours
87. Neonatal period extendsupto: d. 4 mg start Ref: KD Tripathi 7/e p289, 291; Ghai 8/e p170
a. 21 days of life (NEET/DNB Pattern Question) 96. Dose of i.v. adrenaline in term infant is during neonatal
b. 30 days of life resuscitation: (NEET/DNB Pattern Question)
c. 28 days of life a. 0.1 0.3 ml/kg in 1:1000
d. 35 days of life b. 0.3 0.5 ml/kg in 1:1000
Ref: Ghai 8/e p124; 7/e p96 c. 0.1 0.3 ml/kg in 1:10,000
88. A hymenal tag in a new born is best treated by: d. 0.3 0.5 ml/kg in 1:10,000 Ref: Ghai 8/e p132; 7/e p103
a. Steroids (NEET/DNB Pattern Question) 97. One gm of Hb liberates mg of bilirubin:
b. Surgery a. 40 (NEET/DNB Pattern Question)
c. Leaving it alone b. 34
d. None of the above c. 15
Ref: Ghai 6/e p146, 147 d. 55 Ref: Ghai 6/e p169; Internet
89. IUGR is caused by all except: 98. Neonatal Jaundice first time appears in the 2nd week not a
a. Diabetes (NEET/DNB Pattern Question) cause is: (NEET/DNB Pattern Question)
b. Alcohol a. Galatossemia
c. Smoking b. Rh Incompatibility
d. Chronic renal failure c. Hypothyroidism
Ref: Ghai 8/e p156; 7/e p128 d. Breast milk Jaundice Ref: Ghai 8/e p311, 312
90. In asymmetrical IUGR which organ is not affected: 99. Causes of conjugated hyperbilirubinemia is:
a. Subcutaneous fat (NEET/DNB Pattern Question) a. Rotor syndrome (NEET/DNB Pattern Question)
b. Muscle b. Breast milk jaundice
c. Liver c. Crigler najjar
d. Brain d. Gilbert syndrome
Ref: Ghai 8/e p156; 7/e p129; Nelson 18/e p702, 703 Ref: Ghai 8/e p312; Nelson 17/e Chap 352
91. In a child cessation of breathing for 20 sec. With bradycardia 100. Maximum concentration of dextrose that can be given
is: (NEET/DNB Pattern Question) through peripheral vascular line in neonate:
a. Apnea a. 5 (NEET/DNB Pattern Question)
b. Dyspnea b. 10
c. Cheyne stokes respiration c. 12.5
d. None d. 25
Ref: Ghai 8/e p171; 7/e p146 Ref: AIIMS Neonatalogy protocol Hypoglcemia in Newborn

Ans. 83. b. Rickets 84. b. Osteomalacia 85. a. Keshan disease 86. d. 20mg/10 14 day
87. c. 28 days of life 88. c. Leaving it alone 89. None > diabetes 90. d. Brain
91. a. Apnea 92. b. Obstructive... 93. b. Fibrin 94. a. L/S ratio
95. b. 12 mg every 24... 96. c. 0.1 0.3 ml/kg in... 97. b. 34 98. b. Rh Incompatibility
99. a. Rotor syndrome 100. c. 12.5
114 Jaypees Triple A
101. Hyperglycemia in Neonate if blood sugar is above: 110. Coarctation of the aorta is common in which syndrome:
a. 150 mg/dl (NEET/DNB Pattern Question) a. Downs (NEET/DNB Pattern Question)
b. 125 mg/dl b. Turners
c. 180 mg/dl c. Klinefelters
d. 100 mg/dl d. Noonans
Ref: Indian Pediatrics 2008; 45: 29-38; Internet Ref: Ghai 8/e p401; Nelson 18/e p1900
102. The heart lesion not found in Congenital Rubella infection 111. Congestive cardiac failure is diagnosed in an infant by:
is: (NEET/DNB Pattern Question) a. Basal crepts (NEET/DNB Pattern Question)
PRACTICE Qs Pediatrics

a. ASD b. Elevated JVP


b. VSD c. Pedal edema
c. PDA d. Liver enlargement
d. PS Ref: Ghai 8/e p397-399; 7/e p375; Nelson 18/e p1977
Ref: Davidson 21/e p311; Myung K Park 5/e p12; CPDT 18/e p53,54 112. Umbilical cord has: (NEET/DNB Pattern Question)
103. Commonest type of cong. Cyanotic heart disease is: a. 1 vein and 2 arteries
a. ASD (NEET/DNB Pattern Question) b. 2 vein and 2 arteries
b. VSD c. 1 vein and 1 artery
c. TOF d. 2 veins and 1 artery
d. PDA Ref: Ghai 8/e p420; 7/e p408; Nelson 18/e p1882 Ref: Ghai 6/e p674; Internet
104. In atrial septal defect the aorta is: 113. In child, foreign body in lung:
a. Small (NEET/DNB Pattern Question) a. Rigid bronchoscopy (NEET/DNB Pattern Question)
b. Normal b. Chest x-ray
c. Enlarged c. Flexible endoscopy
d. Aneurysmal d. Direct laryngoscopy Ref: Ghai 8/e p369; 7/e p367
Ref: Ghai 8/e p413; 7/e p402; Nelson 18/e p1884 114. Most common cause of stridor shortly after birth:
105. ASD is associated with all except: a. Laryngeal papilloma (NEET/DNB Pattern Question)
a. Infective endocarditis (NEET/DNB Pattern Question) b. Laryngeal web
b. Stroke c. Laryngomalacia
c. Arrhythemia d. Vocal cord palsy Ref: Ghai 8/e p368, 369; 7/e p340
d. Pulmonary hypertension Ref: Ghai 8/e p413, 414 115. Treatment of choice in bronchiolitis is:
106. In patent ductus arteriosus connection is between: a. Ribavirin (NEET/DNB Pattern Question)
(NEET/DNB Pattern Question) b. Amantadine
a. Aorta and coronary artery c. Vidarabine
b. Aorta and pulmonary artery d. Zidovudine
c. Aorta and subclavian artery Ref: Ghai 8/e p381-382; 7/e p358; Nelson 18/e p1776
d. Pulmonary artery and subclavian artery 116. In a child with exercise induced asthma, which is done:
Ref: Ghai 8/e p417, 418; 7/e 405; Nelson 18/e p1891 a. Prophylaxis with steroids
107. Large PDA (Patent ductus arterious) leads to: b. Prophylaxis with Beta, agonist
a. Endocardial valvulitis (NEET/DNB Pattern Question) c. Prophylaxis with theophyline
b. Eisenmenger syndrome d. Breathing exercise (NEET/DNB Pattern Question)
c. CHF Ref: Ghai 8/e p389, 390; 7/e p365-366; CPDT 18/e p1058
d. All of above Ref: Ghai 8/e p417; 7/e p406 117. In which disease, symptoms improve with crying:
108. Tetralogy of fallot is characterized by following except: a. Tetralogy of fallot (NEET/DNB Pattern Question)
w(NEET/DNB Pattern Question) b. Choanal atresia
a. AS c. Bronchial asthma
b. VSD d. All of above Ref: Ghai 8/e p366; 7/e p337
c. Over-riding of aorta 118. A 6-weeks-old boy presenting with palpable abdominal
d. Infundibular constriction Ref: Ghai 8/e p420; 7/e p408 mass in the epigastrium. The clinical diagnosis is (There is
109. All are true regarding tricuspid atresia except: no bile in vomitus): (NEET/DNB Pattern Question)
a. Split S2 (NEET/DNB Pattern Question) a. Duodenal Atresia
b. Patent foramen vole b. Choledochal cyst
c. Pulmonary oligemia in chest X-ray c. Pyloric stenosis
d. Left axis deviation in ECG d. Oesophageal Atresia
Ref: Ghai 8/e p423; 7/e p410 Ref: Bailey 23/e p899; Ghai 8/e p279; 7/e p253; Nelson 18/e p1555

Ans. 101. b. 125 mg/dl 102. a. ASD 103. c. TOF 104. a. Small
105. a. Infective... 106. b. Aorta and... 107. d. All of the above 108. a. AS
109. a. Split S2 110. b. Turners 111. d. Liver enlargement 112. a. 1 vein and 2 arteries
113. c. Flexible endoscopy 114. c. Laryngomalacia 115. a. Ribavirin 116. b. Prophylaxis with...
117. b. Choanal atresia 118. c. Pyloric stenosis
Practice Questions 115
119. Most common cause of Acute Intestinal Obstruction in c. Low lipid
neonates is: (NEET/DNB Pattern Question) d. Platelet activation Ref: ??
a. Jejunal atresia 128. Most common cause of Nephrotic Syndrome in children:
b. Malrotation a. Minimal change disease (NEET/DNB Pattern Question)
c. Duodenal atresia b. IgA nephropathy
d. Acute Intussusception Ref: Internet c. Mesangial GN
120. Commonest cause of intestinal obstruction in children is: d. FSGN
a. Intussusception (NEET/DNB Pattern Question) Ref: Ghai 8/e p477; 7/e p457

PRACTICE Qs Pediatrics
b. Volvulus 129. Which of following is not seen in nephritic syndrome:
c. Hernia a. Edema (NEET/DNB Pattern Question)
d. Adhesions b. Hematuria
Ref: Nelson 18/e p1569; Internet c. HTN
121. Common site of regional enteritis is: d. Hypocholestremia
a. Colon (NEET/DNB Pattern Question) Ref: Internet
b. Rectum 130. All are seen in Henoch scholein purpura except:
c. Distal ileum and colon a. Thrombocytopenia (NEET/DNB Pattern Question)
d. Caecum b. Glomerulonephritis
Ref: Internet; Davidson 21/e p884; c. Arthralgia
Nelson 18/e p1590, 1581; Robbins 7/e p847 d. Abdominal pain
122. Most common cause of cholestatic jaundice in new born is: Ref: Davidson 21/e p114; Nelson 18/e p2178, 2179;
a. Hypoplasia of biliary tract Robbins 7/e p650, 986. 987
b. Neonatal hepatitis (NEET/DNB Pattern Question) 131. Most common cause of haemolytic uremic syndrome:
c. Choledochal cyst a. E. coli (NEET/DNB Pattern Question)
d. Physiological b. Shigella
Ref: CPDT 18/e p644; Internet c. Salmonella
123. Most common cause of portal hypertension in children is: d. Psedononas Ref: Nelson 18/e p2181
a. Extrahepatic compression 132. In shigella dysentery associated haemolytic uremic
b. Budd chairi syndrome (NEET/DNB Pattern Question) syndrome, the false statement is:
c. Veno-occlusive disease a. Leucocytosis (NEET/DNB Pattern Question)
d. Post necrotic b. Neurological abnormalities
Ref: Nelson 18/e p1709; Ghai 8/e p319 c. Hepatic failure
124. Profuse watery diarrhea in an immune compromised child d. Thrombotic angiopathy
is due to: (NEET/DNB Pattern Question) Ref: Nelson 18/e p2181; Harrisons 18/e p970
a. Cryptococcus 133. Which is not a feature of posterior urethral valve:
b. Amoeba a. Palpable bladder (NEET/DNB Pattern Question)
c. Giardia b. Painful stress incontinence
d. Lactose intolerance c. Recurrent UTI
Ref: Nelson 17/e chap 273; Ghai 8/e p291 d. Hydronephrosis Ref: Nelson Chap 540
125. Chronic constipation in children is seen in all A/E: 134. Not true about chronic pyelonephritis in children:
a. Hirschspring disease (NEET/DNB Pattern Question) (NEET/DNB Pattern Question)
b. Jejunal polyp a. Associated with Ureteric reflux
c. Hypothyroidism b. Associated with Intrarenal reflux
d. Stricture c. Associated with renal scarring
Ref: Ghai 8/e p284; 7/e p255 d. Males are more affected than female
126. Unilateral renal agenesis is associated with: Ref: Harrisons 18/e p2388; Nelson 17/e chap 538;
a. Polycystic disease of pancreas www.emedicinemedscape .com
b. Hiatus Hernia (NEET/DNB Pattern Question) 135. Most common cause of convulsion on the First day of life in
c. Single umbilical artery a new born is: (NEET/DNB Pattern Question)
d. Hypogonadism a. Anoxia
Ref: Nelson 17/e p1783; Ghai 6/e p467 b. Head injury
127. Which is seen in nephrotic syndrome: c. Hypoglycemia
a. Low serum calcium (NEET/DNB Pattern Question) d. Hypocalcemia
b. Raised AT-III Ref: Ghai 8/e p553; 7/e p531; Nelson 18/e p712

Ans. 119. c. Duodenal... 120. a. Intussusception 121. c. Distal ileum... 122. b. Neonatal hepatitis
123. a. Extrahepatic... 124. c. Giardia 125. b. Jejunal polyp... 126. c. Single Umbilical..
127. a. Low serum... 128. a. Minimal change.... 129. d. Hypocholestremia 130. a. Thrombocytopenia
131. a. E.coli 132. c. Hepatic failure 133. b. Painful stress... 134. d. Males are more...
135. a. Anoxia
116 Jaypees Triple A
136. Drug of choice in absence seizure: c. Red
a. Carbamazepine (NEET/DNB Pattern Question) d. Purple
b. Phenytoin Ref: Nelson 18/e p531; Ghai 8/e p652; Internet
c. ACTH 146. Mousy odour urine is seen in:
d. Ethosuximide Ref: Ghai 8/e p558, 559; 7/e p532 a. Maple syrup urine (NEET/DNB Pattern Question)
137. Initial drug of choice in a choice in a child with status b. Phenylketonuria
epilepticus: (NEET/DNB Pattern Question) c. Isovalericaciduria
a. Lorazepam d. Cystinuria
PRACTICE Qs Pediatrics

b. Phenobarbitone Ref: Ghai 8/e p652; 7/e 629; Nelson 18/e p529
c. Valproate 147. Not seet in first decade: (NEET/DNB Pattern Question)
d. Phenytoin Ref: Ghai 8/e p554; 7/e p525 a. Retino blastoma
138. The most common causative agent of meningitis in the age b. Rhabdomyosarcoma
group of 6 months-3 year amongst the following is: c. Neuroblastoma
a. Streptococcus pneumonia d. Ameloblastoma
b. H.influenzae (NEET/DNB Pattern Question) Ref: Ghai 8/e p613-620; 7/e p590
c. Staphylococcus 148. Commonest site of extramedullary relapse of ALL:
d. N. gonorrhoeae Ref: Ghai 8/e p563 a. CNS (NEET/DNB Pattern Question)
139. The most common Intracranial tumor in children is: b. Lung
a. Glioma (NEET/DNB Pattern Question) c. Liver
b. Ependymoma d. Testis Ref: Ghai 8/e p604
c. Meningioma 149. All of the following are features of juvenile CML except:
d. Lymphangioma a. Thrombocytopenia (NEET/DNB Pattern Question)
Ref: Ghai 8/e p571; Nelson 18/e p2129-2133 b. Fetal Hb is increased
140. Preventable causes of mental retardation are: c. Philadelphia chromosome is positive
a. Downs (NEET/DNB Pattern Question) d. Lymphadenopathy
b. Phenylketonuria Ref: Ghai 8/e p608; 7/e p586; Nelson 18/e p2122
c. Cretinism 150. 4-year-old child having palpable abdominal mass &
d. Cerebral palsy Ref: Ghai 8/e p585; 7/e p562 hypertension with sweating & diarrhea is due to:
141. Aspirin is associated with: (NEET/DNB Pattern Question) a. Neuroblastoma (NEET/DNB Pattern Question)
a. Reyes Syndrome b. Nephroblastoma
b. Sjogren Syndrome c. PCKD (Polycystic kidney disease)
c. Reiter Syndrome Ref: Ghai 8/e p616; 7/e p590
d. None of the above 151. The most common presentation of a child with Wilms
Ref: Ghai 8/e p570; Nelson 18/e chap 358 tumor is: (NEET/DNB Pattern Question)
142. Decorticate childFalse statement is: a. As asymptomatic abdominal mass
a. Acute Brain injury (NEET/DNB Pattern Question) b. Haematuria
b. Susthalamic, CT & frontal lobe lesion c. Hypertension
c. More dangerous than decerebrate lesion d. Hemoptysis due to pulmonary secondary
d. Flexion of arm & extension of lower limb Ref: Internet Ref: Ghai 8/e p617, 618; 7/e p592; Nelson 18/e p2141
143. All are liver glycogenosis except: 152. Baby with recurrent infection of ear & discharge &
a. Von Girke disease (NEET/DNB Pattern Question) seborrheic dermatitis with hepatosplenomegaly with cystic
b. Hers disease skull lesions. Diagnosis is: (NEET/DNB Pattern Question)
c. Type III glycogenosis a. Hemophagocytic lymphohistiocytosis
d. Pompes disease Ref: Ghai 8/e p657; Nelson 18/e p603, 604 b. Langerhans cell histiocytosis
144. Enzyme deficient in phenylketonuria: c. All
a. Tyrosinase (NEET/DNB Pattern Question) d. Multiple myeloma
b. Phenylalanine hydoroxylase Ref: Ghai 8/e p620; 7/e p595
c. Tyrosine transaminase 153. Highest cure rate is of: (NEET/DNB Pattern Question)
d. Homogentisic oxidase a. Wilms Tumor
Ref: Ghai 8/e p652; 7/e p628 b. Retinoblastoma
145. In phenylketonuria FeCl3 test with urine gives .. color: c. Rhabdomyosarcoma
a. Green (NEET/DNB Pattern Question) d. All
b. Blue Ref: Nelson 17/e p491; Ghai 8/e p616

Ans. 136. a. Carbamazepine 137. a. Lorazepam 138. a. Streptococcus... 139. a. Glioma


140. c. Cretinism 141. a. Reyes syndrome 142. c. More dangerous... 143. d. Pompes disease
144. b. Phenylalanine... 145. a. Green 146. b. Phenylketonuria 147. d. Ameloblastoma
148. a. CNS 149. c. Philadelphia... 150. a. Neuroblastoma 151. a. As asymptomatic...
152. b. Langerhans... 153. b. Retinoblastoma
Practice Questions 117
154. Which of the following haemoglobin (Hb) estimation will 162. What stimulates the gonads in male at 8 week to secrete
be diagnostically helpful in a case of beta thalassesmia testosterone: (NEET/DNB Pattern Question)
trait: (NEET/DNB Pattern Question) a. Inhibin from corpus luteum
a. Hb-F b. GnRH from hypothalamus of baby
b. Hb1-C c. Placental HCG
c. Hb-A2 d. All of above Ref: Nelson 18/e Chap 589
d. Hb-H 163. Duchnnes muscular dystrophy:
Ref: Ghai 8/e p341; 7/e p308; Nelson 18/e p2035, 2036 a. X-linked dominant (NEET/DNB Pattern Question)

PRACTICE Qs Pediatrics
155. In a-thalassemia: (NEET/DNB Pattern Question) b. X-linked recessive
a. Excess a-chain c. Autosomal dominant
b. No a-chain d. Autosomal recessive
c. Excess b-chain Ref: Ghai 7/e p566; Ghai 8/e p595
d. No b-chain Ref: Robbins 7/e p635 164. All of the following are seen in Systemic Juvenile Arthritis,
156. Salmonellosis is most common in: except: (NEET/DNB Pattern Question)
a. Sickle cell anemia (NEET/DNB Pattern Question) a. Rheumatoid Factor + ve
b. Thalassemia b. Hepatosplenomegaly
c. Hemophilia c. High fever with rash
d. Cystic fibrosis d. Elevated E.S.R.
Ref: Ghai 8/e p344; 7/e p311; Harrison 17/e p804 Ref: Ghai 8/e p625, 626; 7/e p600
157. First change of improvement noted after iron therapy is 165. Phocomelia is: (NEET/DNB Pattern Question)
initiated: (NEET/DNB Pattern Question) a. Absence of long bones
a. Decreased irritability b. Absence of brain
b. Retinculcytosis c. Reduplication of bones
c. Increase is serum iron levels d. Absence of heart Ref: Internet
d. Replenishment of iron stores 166. All the following are features of Downs syndrome except:
Ref: Wintrobes hematalogy 12/e p829 a. Brushfields spots in iris
158. Pseudohermaphroditism in a female child is most b. Simian crease (NEET/DNB Pattern Question)
commonly due to: (NEET/DNB Pattern Question) c. Mental retardation
a. 21-hydroxylase deficiency d. Hypertonicity
b. 17-hydroxylase deficiency Ref: Ghai 8/e p638, 639; 7/e p613; Nelson 18/e p508
c. 11-hydroxylase deficiency 167. Down syndrome, All are seen except:
d. 3-hydroxylase deficiency a. t (14 ; 21) (NEET/DNB Pattern Question)
Ref: Nelson 18/e p2360; Internet b. Trisomy 21
159. All of the following can cause precocious puberty in males c. t (11 : 14)
except: (NEET/DNB Pattern Question) d. t (15 : 21)
a. 17 alpha hydroxylase deficiency Ref: Nelson 17/e Chap 81; Internet
b. 21 alpha hydroxylase deficiency 168. Edema of hands & feet in Infants is characterised:
c. 11 b hydroxylase deficiency a. Kilnefelters syndrome (NEET/DNB Pattern Question)
d. None of the above b. Noonam syndrome
Ref: Nelson 18/e p2366; Ghai 8/e p533, 534 c. Turners syndrome
160. 2-year-old child presented with diarrhoea and failure to d. Fragile x syndrome
thrive, Blood examination shows Na = 122, K 6. He is Ref: Ghai 8/e p640; 7/e p616
most probably suffering from: 169. Barr body is absent in female having:
a. Bartter syndrome (NEET/DNB Pattern Question) a. 46 XX genome (Neet/DNB Pattern uestion)
b. 21-a Hydroxylase deficiency b. 45 X0 genome
c. 11-b Hydroxylase deficiency c. 47 XXX
d. 17-a Hydroxylase deficiency d. All of the above Ref: Ghai 8/e p640; Internet
Ref: Harrison 17/e p2345-2346
170. Patau syndrome feature include A/E:
161. Gonads to testes differentiation: a. Cleft lip (NEET/DNB Pattern Question)
a. SRY gene (NEET/DNB Pattern Question) b. Hypotelorism
b. WNT-4 gene c. Holoprosencephaly
c. DAX1 gene d. Rocker bottom foot
d. None Ref: Nelson chapter 589; Ghai 8/e p534 Ref: Davidson 21/e p53; Nelson 17/e Chap 81 Table 81.2

Ans. 154. c. HB-A2 155. b. No a-chain 156. a. Sickle cell... 157. a. Decreased irritability
158. a. 21-hydroxylase... 159. a. 17 alpha... 160. b. 21-a Hydroxylase... 161. a. SRY gene
162. c. Placental HCG 163. b. X-linked recessive 164. a. Rheumatoid... 165. a. Absence of long...
166. d. Hypertonicity 167. c. t(11 : 14) 168. c. Turners syndrome 169. b. 45 X0 genome
170. d. Rocker bottom...
118 Jaypees Triple A
171. Trisomy 13 is seen in: (NEET/DNB Pattern Question) 180. Juvenile idiopathic arthritis includes all the following
a. Edward Syndrome categories except: (NEET Pattern Question)
b. Patau Syndrome a. Psoriatic arthritis
c. Down Syndrome b. Enthesitis related arthritis
d. Turner Syndrome c. Systemic arthritis
Ref: Davidson 21/e p53; Nelson 17/e Chap 81 Table 81.2 d. Reactive arthritis
172. Klinefelter syndrome is dignosed by: Ref: Nelson Textbook of Pediatrics, 19/e p830; Ghai 8/e p625, 626
a. Karyotyping (NEET/DNB Pattern Question) 181. A 7-year-old boy presents with complaints of hyperactivity,
PRACTICE Qs Pediatrics

b. USG abdomen excessive shoulder shrugging, throat clearing and eye


c. Triple test blinking. During examination, he is not cooperative, uses
d. Echocardiography obscene words and runs around in the consultation room.
Ref: Ghai 7/e p615; Ghai 8/e p640 What is the most probable diagnosis?
173. Rubella causes all except: (NEET/DNB Pattern Question) a. Attention deficit hyperactivity disorder
a. Microcephaly b. Tic Disorder (NEET Pattern Question)
b. VSD c. Psychomotor seizures
c. Conduction defect d. Childhood Psychosis
d. All Ref: Nelson Textbook of Pediatrics, 19/e p77
Ref: Nelson 18/e Chap 244; Davidson 21/e p311 182. Which of the following carpal bone will be present in wrist
174. Following are complications of chicken pox except: Xray of a 2 months old child? (NEET Pattern Question)
a. Meningitis (NEET/DNB Pattern Question) a. Capitate
b. Pneumonia b. Scaphoid
c. Enteritis c. Lunate
d. Reyes Syndrome Ref: Ghai 8/e p215; 7/e p186 d. Trapezoid
Ref: Nelsons Textbook of Pediatrics 18/e p48
175. Measles virus is: (NEET/DNB Pattern Question)
a. Paramyxovirus 183. Areola and papilla forming secondary mound in adolescent
b. Orthomyxovirus girls is classified under which stage of sexual maturiy rating
c. Poxvirus (SMR)? (NEET Pattern Question)
d. Picornavirus a. SMR Stage 2
Ref: Ananthanarayan 7/e p517; Ghai 8/e p213; 7/e p185 b. SMR Stage 3
c. SMR Stage 4
176. Glioma of optic nerve is usually:
d. SMR Stage 5
a. Gemistocytic (NEET/DNB Pattern Question)
Ref: Nelson Textbook of Pediatrics, 19/e p651; Ghai 8/e p64
b. Pilocytic
c. Fibrillary 184. Most common cause of congenital hypothyroidism?
d. Lamellar Ref: Nelson 18/e p2608 a. Thyroid dysgenesis (NEET Pattern Question)
b. Dyshormonogenesis
177. GBS in a child treatment: (NEET/DNB Pattern Question)
c. Antithyroid antibodies
a. IV Ig
d. Maternal Hypothyroidism
b. Ventilation
Ref: Nelsons Textbook of Pediatrics. 19/e p1895;
c. Plasmapharesis
Ghai 8/e p516
d. All of above Ref: Ghai 7/e p554; Ghai 8/e p591
185. All the following conditions are associated with pulmonary
178. All are causes of eosinophilia except:
hemorrhage in infants except:
a. Allergic Rhinitis (NEET Pattern Question)
a. CDH (NEET Pattern Question)
b. Trichinosis
b. Exogenous surfactant
c. Corticosteroid therapy
c. PDA
d. Rhematoid arthritis
d. Sepsis
Ref: Nelson Textbook of Pediatrics, 19/e p740;
Ref: Ghai 8/e p417; Nelsons Textbook of Pediatrics, 18/e p599
Ghai 8/e p358)
186. Neonatal hyperglycemia is called when plasma glucose
179. All are true regarding osmotic diarrhoea except:
level is: (NEET Pattern Question)
(NEET Pattern Question) a. > 125 mg/dL
a. Due to highly osmotic substances in lumen b. > 150 mg/dL
b. Fasting may improve symptoms c. > 165 mg/dL
c. Stool reducing substances are present d. > 185 mg/dL
d. Can be mediated by bacterial toxins Ref: Internet
Ref: Nelson Textbook of Pediatrics, 19th edition

Ans. 171. b. Patau syndrome 172. a. Karyotyping 173. c. Conduction defect 174. c. Enteritis
175. a. Paramyxovirus
176. b. Pilocytic 177. d. All of the above 178. c. Corticosteroid...
179. d. Can be mediated... 180. d. Reactive arthritis 181. b. Tic disorder 182. a. Capitate
183. c. SMR Stage 4
184. a. Thyroid dysgenesis 185. a. CDH 186. a. > 125 mg/dL
Practice Questions 119
187. Lutembacher syndrome is: (NEET Pattern Question) 197. Kernicterus associated with: (NEET Pattern Question)
a. Marked cardiomegaly with mitral regurgitation a. Gilbert syndrome
b. Marked cardiomegaly with mitral valve obstruction b. Crigler Najjar syndrome
c. Marked cardiomegaly with TR. c. G6PD deficiency
d. Marked cardiomegaly wit tricuspid stenosis. d. All of the above Ref: Internet
188. Which vitamin supplementation is routinely recommended 198. Sign of hypothyroidism in neonates:
for children? (NEET Pattern Question) a. Prolonged neonatal jaundice
a. Vit. D b. Goiter (NEET Pattern Question)

PRACTICE Qs Pediatrics
b. Vit. B complex c. Arrhythmia
c. Vit. C d. Hyperkinesis Ref: Ghai 8/e p516
d. Vit. E Ref: Ghai 8/e p112 199. Brushfield spots seen in: (NEET Pattern Question)
189. MC site of Langerhans cell histiocytosis: a. Klinefelter syndrome
a. Skin (NEET Pattern Question) b. Turner syndrome
b. Bone c. Down syndrome
c. Lung d. Neurofibromatosis Ref: Ghai 8/e p638; Internet
d. Liver Ref: Ghai 8/e p620 200. Most common cause of meningitis in 2 year old child is:
190. Cyanosis in newborn disappears on crying, diagnosis is: a. Staph. Aureus (NEET Pattern Question)
a. Unilateral choanal atresia (NEET Pattern Question) b. H. influenza
b. Bilateral choanal atresia c. Strep. Pneumonia
c. Enlarge tonsil d. N. goorrhea Ref: Ghai 8/e p563
d. Nasal Obstruction Ref: Ghai 8/e p366 201. Prepubertal volume of testes is: (NEET Pattern Question)
191. Which of the following does not included height? a. < 5 ml
a. Rao and singh index (NEET Pattern Question) b. < 10 ml
b. Dugdales index c. < 15 ml
c. Gomez classification d. 20 ml Ref: Ghai 8/e p63
d. Waterlows classification Ref: Internet 202. The total body iron content of a newborn infant born
192. Not indicate RDS in neonate: (NEET Pattern Question) healthy is: (NEET Pattern Question)
a. Wheezing a. 150 mg
b. Grunting b. 280 mg
c. Retraction c. 450 mg
d. Tachypnea Ref: Ghai 8/e p169 d. 600 mg Ref: Internet
193. In congenital rubella syndrome, MC presentation is: 203. IN 2 year old child to diagnose HIV which test is useful?
a. Cataract (NEET Pattern Question) a. PCR DNA (NEET Pattern Question)
b. PDA b. PCR RNA
c. Deafness c. Elisa
d. Mental retardation Ref: Ghai 8/e p272 d. P24 Ref: Ghai 8/e p234
194. Vit. A requirement in infant: (NEET Pattern Question) 204. Pneumonia is a complication of: (NEET Pattern Question)
a. 350 micro gm a. Measles
b. 500 micro gm b. Chickenpox
c. 700 micro gm c. CMV
d. 900 micro gm Ref: Ghai 8/e p89 d. BV Ref: Ghai 8/e p213, 214
195. A 4 year child having abdominal mass, hematuria and fever 205. Most common source of bilirubin in newborn:
most common cause is: (NEET Pattern Question) a. Platelets (NEET Pattern Question)
a. Neuroblastoma b. RBC
b. Wilms tumour c. WBC
c. Hydronephrosis d. Protein Ref: Internet
d. Bladder tumour Ref: Ghai 8/e p617) 206. Daily water required in child weighing 20 kg:
196. Hypoglycemia in pre-mature infant is due to: a. 625 ml (NEET Pattern Question)
a. Bigger brain than body (NEET Pattern Question) b. 1125 ml
b. Decreased glycogenolysis c. 1550 ml
c. Increased liver function d. 2200 ml
d. Decreases requirement Ref: Internet

Ans. 187. b. Marked... 188. a. Vit. D 189. b. Bone 190. b. Bilateral choanal...
191. c. Gomez... 192. a. Wheezing 193. c. Deafness 194. a. 350 micro gm
195. b. Wilms tumour 196. b. Decreased... 197. d. All of the above 198. a. Prolonged neonatal...
199. c. Down syndrome 200. c. Strep. Pneumonia 201. a. < 5 ml 202. b. 280 mg
203. c. Elisa 204. a. Measles 205. b. RBC 206. c. 1550 ml
120 Jaypees Triple A
207. Trisomy is seen in: (NEET Pattern Question) 211. About Wilms tumour true are all except:
a. Turner syndrome (NEET Pattern Question)
b. Down syndrome a. It is most common abdominal malignancy in child
c. Noonan syndrome b. Abdominal mass is most common symptom
d. Angelmans syndrome Ref: Ghai 8/e p637) c. Hyperploidy is poor prognostic factor
208. In Lesch Nyhan syndrome true is: (NEET Pattern Question) d. Commonly metastasise to brain Ref: Ghai 8/e p617
a. Deficiency of HPRT1 212. Most common location of medulloblastoma?
b. Uric acid accumulate a. Pons (NEET Pattern Question)
PRACTICE Qs Pediatrics

c. Self injured behaviour b. Medulla


d. All of the above Ref: Internet c. Cerebellum
209. Month old child with diarrhea and non bilious vomiting d. Mid brain Ref: Ghai 8/e p613; Davidson 21/e p1216)
and palpable abdomen lump most probable diagnosis is: 213. Mc congenital heart disease? (NEET Pattern Question)
a. Duodenal atresia a. ASD
b. Pyloric stenosis (NEET Pattern Question) b. VSD
c. Primary biliary atresia c. PDA
d. Oesophageal atresia Ref: Internet d. TOF Ref: Ghai 8/e p414)
210. Delayed dentition is characteristic of: 214. Hypertrophic pyloric stenosis-which fluid should be
a. Endemic cretinism (NEET Pattern Question) given? (NEET Pattern Question)
b. Hyperthyroidism a. Hypertonic saline
c. Hyperpituitarism b. Hypotonic saline
d. Turner syndrome Ref: Ghai 8/e p519; Internet c. Glucose
d. Normal saline Ref: Ghai 8/e p280

Ans. 207. b. Down syndrome 208. d. All of the above 209. b. Pyloric stenosis 210. a. Endemic cretinism
211. d. Commonly...
212. c. Cerebellum 213. b. VSD 214. d. Normal saline

Vous aimerez peut-être aussi